X



トップページ数学
1002コメント566KB
面白い問題おしえて〜な 28問目
レス数が1000を超えています。これ以上書き込みはできません。
0001132人目の素数さん
垢版 |
2018/10/29(月) 00:19:23.87ID:59VF2v6C
過去ログ置き場 (1〜15問目)
http://www3.tokai.or.jp/meta/gokudo-/omoshi-log/

まとめwiki
http://www6.atwiki.jp/omoshiro2ch/

1 http://cheese.5ch.net/test/read.cgi/math/970737952/
2 http://natto.5ch.net/test/read.cgi/math/1004839697/
3 http://science.5ch.net/test/read.cgi/math/1026218280/ *
4 http://science.5ch.net/test/read.cgi/math/1044116042/ *
5 http://science.5ch.net/test/read.cgi/math/1049561373/ *
6 http://science.5ch.net/test/read.cgi/math/1057551605/ *
7 http://science2.5ch.net/test/read.cgi/math/1064941085/
8 http://science3.5ch.net/test/read.cgi/math/1074751156/
9 http://science3.5ch.net/test/read.cgi/math/1093676103/
10 http://science4.5ch.net/test/read.cgi/math/1117474512/
11 http://science4.5ch.net/test/read.cgi/math/1134352879/
12 http://science6.5ch.net/test/read.cgi/math/1157580000/
13 http://science6.5ch.net/test/read.cgi/math/1183680000/
14 http://science6.5ch.net/test/read.cgi/math/1209732803/
15 http://science6.5ch.net/test/read.cgi/math/1231110000/
16 http://science6.5ch.net/test/read.cgi/math/1254690000/
17 http://kamome.5ch.net/test/read.cgi/math/1284253640/
18 http://kamome.5ch.net/test/read.cgi/math/1307923546/
19 http://uni.5ch.net/test/read.cgi/math/1320246777/
20 http://wc2014.5ch.net/test/read.cgi/math/1356149858/
21 http://wc2014.5ch.net/test/read.cgi/math/1432255115/
22 http://rio2016.5ch.net/test/read.cgi/math/1464521266/
23 http://rio2016.5ch.net/test/read.cgi/math/1497416499/
24 http://rio2016.5ch.net/test/read.cgi/math/1502016223/
25 http://rio2016.5ch.net/test/read.cgi/math/1502032053/
26 http://rio2016.5ch.net/test/read.cgi/math/1518967270/
27 http://rio2016.5ch.net/test/read.cgi/math/1532793672/
0002132人目の素数さん
垢版 |
2018/10/29(月) 00:23:17.37ID:59VF2v6C
(*) 3〜6問目で「datが存在しません。」が出ます。

削除依頼は不要です。
0003132人目の素数さん
垢版 |
2018/10/29(月) 01:18:29.19ID:2uK9eMpf
削除依頼を出しました
0004イナ ◆/7jUdUKiSM
垢版 |
2018/10/29(月) 16:37:23.99ID:Es1mqcC9
過去ログ27の898つづき。
扇形OAB=3.14cu
AB^2=(2√2-2)^2+2^2
=8-8√2+4+4
=16-8√2
AB=√(16-8√2)
=2√(4-2√2)
△OAB=(1/2)・2√(4-2√2)・√{(2√2)^2-(4-2√2)}
=√(4-2√2)・√(4+2√2)
=√(16-8)
=2√2
三日月形AB=3.14-2√2
(=3.14-2.8284……
≒0.312)
ABを延長、半直線AB上にP(遠いほう)から垂線PCを下ろす。
ABの中点をNとすると、
ON=2△OAB/AB
=2・2√2/2√(4-2√2)
=√(4+2√2)
扇形PABの高さPC=ON+PH
=√(4+2√2)+PH
扇形PAB=(1/2)AB・PC-三日月形AB
=√(4-2√2)・{√(4+2√2)+PH}
=√8+PH√(4-2√2)
=√8+√(8-OH^2)(4-2√2)
= cu
(考え中)
OP//BQ(外側のQ)
中心角∠AOB=45°
円周角∠APB=22.5°
NB=√(4-2√2)
△OPQ∽△BQP
√は消えるはず……
0005 【末吉】
垢版 |
2018/10/30(火) 00:26:22.58ID:rGErkV8r
>>4
図描いたら円Oは一辺4pの正方形にすっぽり入るから半径2pかもしれない。
OP=2p
OM=MA=1p
なんで√が要るのかすらももうわからない。
円Aの半径は1pのようだ。小学生向けか。
とりあえず前スレのはなんだったんだという感じで、仕切りなおし。
0006 【だん吉】
垢版 |
2018/10/30(火) 00:55:37.66ID:rGErkV8r
>>5円の中に一辺4pの正方形だった。あってる。
0007132人目の素数さん
垢版 |
2018/10/30(火) 01:50:09.86ID:bXAGzjkG
〔前スレ.898〕 (再掲)

これも中学の入試問題

図1のように一辺4cmの正方形にちょうど入る大きさの円Oがある。
図2のように円Oの周上に点Aがあり、OAの中点をMとする。
点Aを中心として点Mを通る円を描き、円Aとする。
円Oの周上に点B, Pが、円Aの周上に点Qがあり、次の条件を満たしている。
 ・∠AOB = 45°
 ・BQと円Aは接している。
 ・OPとBQは平行
このとき、直線AP, BP, 円Oの短い方の弧ABで囲まれた面積として考えられるものをすべて答えなさい。
円周率は 3.14 とする。

図1 http://i.imgur.com/uYNULrq.jpg
図2 http://i.imgur.com/s7n55LS.jpg
0009イナ ◆/7jUdUKiSM
垢版 |
2018/10/30(火) 03:00:18.86ID:rGErkV8r
>>6
>>7-8我々は我々ながらよく気づいた。公立行って日本語勉強したほうがいいと思わせる問題だった。
0010イナ ◆/7jUdUKiSM
垢版 |
2018/10/30(火) 12:45:59.96ID:rGErkV8r
>>9
弧AB=円Oの円周/8
=2・3.14・2/8
=1.57p
扇形OAB=3.14・2^2/8
=1.57cu
AからOBに垂線を引いてできる、弧ABを斜辺とする直角三角形について、三平方の定理より、
AB^2=(√2)^2+(2-√2)^2
=8-4√2
AB=2√(2-√2)
=1.5307338
≒1.53p
△OAB=OB×(AからOBに引いた垂線)×(1/2)
=2×√2×(1/2)
=√2
≒1.41cu
三日月形AB≒1.57-1.41
=O.16cu
4つのPの弦ABからの距離を求め、
AB/2=√(2-√2)
≒0.7653669を掛け、
三日月形AB=O.16を足すと、
4つの扇形PABの値が出ると思う。
0012132人目の素数さん
垢版 |
2018/10/30(火) 16:57:56.36ID:bXAGzjkG
Memo.

cos(22.5゚) = (1/2)√(2+√2) = 0.923879532511
sin(22.5゚) = (1/2)√(2-√2) = 0.382683432365

O = (0,0)
円O: xx + yy = 8,
A = (-(√8)sin(22.5゚),-(√8)cos(22.5゚) ) = (-√(4-√8),-√(4+√8) )
B = ( (√8)sin(22.5゚),-(√8)cos(22.5゚) ) = ( √(4-√8),-√(4+√8) )

円A: {x + √(4-√8)}^2 + {y + √(4+√8)}^2 = AM^2 = 2,

点Bをとおる円Aの接線は
 y = ±m{x -√(4-√8)} - √(4+√8)
 m = 1/√(7-4√2) = 0.862856209461
接点Qは
 x(Q) = -(1 -1/√2)^(3/2) = -0.1585126677811
 y(Q) = -3.68384840 と -1.542403459
0013イナ ◆/7jUdUKiSM
垢版 |
2018/10/30(火) 21:45:13.29ID:rGErkV8r
>>10Pが2つとして、
ABの中点をNとすると、
△PAB=NB・(ON±√2)
NB=√(2-√2)
ON^2=OB^2-NB^2
=2^2-(2-√2)
=2+√2
△PAB=√(2-√2)・{√(2+√2)±√2}
=√2±√(4-2√2)
=2.4966057
または0.3318213
三日月形AB=扇形OAB-△OAB
=1.57-√2
=0.1557865
∴扇形PAB=2.4966057+0.1557865
=2.6523922cu(2.65cu)
または扇形PAB=0.3318213+0.1557865
=0.4876078cu(0.488cu)
0014132人目の素数さん
垢版 |
2018/10/30(火) 23:06:38.55ID:sol3nzL/
まだ違う。
てかもう答え出てるんだから何故それで検算してみてから書き込まないの?
0015イナ ◆/7jUdUKiSM
垢版 |
2018/10/30(火) 23:38:37.99ID:rGErkV8r
>>14検算はしました。といっても電卓だし。
答えが出てるとは思ってない。
せめて答えは√がなくなると思ったんだけど。
もちろんsinやcosも小学生にそこまで求めないと思うし。
面積だから単位はcuだと思うけど、求める図形が2つか4つか、いくつあるかもわからない。
>>13
0016132人目の素数さん
垢版 |
2018/10/30(火) 23:45:12.35ID:xWKyl2U0
>>15

> >>14検算はしました。といっても電卓だし。
> 答えが出てるとは思ってない。

出てる答えの何が間違ってると思うん?
0017132人目の素数さん
垢版 |
2018/10/31(水) 01:11:26.66ID:MZfORzCN
Memo.

cos(22.5゚) = √{[1+cos(45゚)]/2} = 0.9238795325
sin(22.5゚) = √{[1-cos(45゚)]/2} = 0.3826834324

O = (0,0)
円O: x^2 + y^2 = (2r)^2,
A = (-2r・sin(22.5゚),-2r・cos(22.5゚) ) = (x(A),y(A))
B = ( 2r・sin(22.5゚),-2r・cos(22.5゚) ) = (x(B),y(B))

AB = x(B) - x(A) = 4r sin(22.5゚) = 2√(2-√2) r = 1.53073373 r^2,
y(A) = y(B) = -√(2+√2) r,

円A: {x - x(A)}^2 + {y - y(A)}^2 = AM^2 = r^2,

点Bから円Aに曳いた接線は
 y = ±m{x -x(B)} + y(B),
 m = 1/√(7-4√2) = 0.862856
接点Qは
 x(Q) = -(1/4)(2-√2)^(3/2) r = -0.1120854 r

OP//BQ より
 y(P) = ±{2m/√(1+mm)} r = ±(1/√2)√(2+√2) r = ±1.306563 r,
 y(P1) - y(B) = (1/2)(2+√2)√(2+√2) r,
 y(P2) - y(B) = (1/2)(2-√2)√(2+√2) r,

△ABP1 = (1/2)AB{y(P1)-y(B)} = (√2 +1)r^2,
△ABP2 = (1/2)AB{y(P2)-y(B)} = (√2 -1)r^2,

(三日月型AB) = (π/2 - √2)r^2 = 0.15658276 r^2,

∴ S = (π/2 ± 1)r^2,
0018BLACKX ◆SvoRwjQrNc
垢版 |
2018/10/31(水) 03:27:59.82ID:5UKLbe5p
https://imgur.com/a/pQmEjCF
点Pから点Qに団地の路地を毎秒1ずつ進む。
1)緑の団地脇を抜ける最短経路の長さはいくつか。
2)A君家からB君家まで最短で何秒かかるか。

点Pスレに貼っても無反応だったので
0019132人目の素数さん
垢版 |
2018/10/31(水) 04:30:04.94ID:yZNMRVFG
>>18
団地一棟の一辺は√5。
1)
P → Q : 15
P → A → B → Q : 2 + 5√5 + 4 = 6 + 5√5
P → A → C → Q : 問題外
P → A → D → Q : 2 + 4√5 + 3 = 5 + 4√5
P → B → C → Q : 問題外
P → B → D → Q : 問題外
P → C → D → Q : 4 + 4√5 + 3 = 7 + 4√5
より5+4√5。
2)題意がP → A → B → Qと進むという意味なら6 + 5√5。
ーーーー
ABとPQが逆?
1)も緑の団地脇のみを抜けてPからQへは移動できないし。
なんでもいいけど。
0020BLACKX ◆SvoRwjQrNc
垢版 |
2018/10/31(水) 05:56:19.70ID:PJ+dgrvi
>>19
あってる。ありがと。
そうね、PABQの時間だねごめん
ABが逆だと思ったのは外接を点CDでやったからだと思う
俺想定してたのがC1〜C7点でやったから関係なかったねごめん
初めからC点つけとけば良かったな
0021イナ ◆/7jUdUKiSM
垢版 |
2018/10/31(水) 09:53:18.49ID:aaz9J1TG
>>15
>>13>>17の三日月形ABの値が少し違うのは3.14とπの違いかな。
問題に3.14を使えとある。だからここは>>13のほうが正しい。実際より少し小さい値になる。

問題文によると、
>>7一辺4pの正方形にちょうど入るってことなんで、
OP=2
OM=MB=1
ただし、図を優先すると値は変わる。
Pが4つか2つか。2つずつ一致するとみて答えを求める。
4つのPの弦ABからの距離を求め、
AB/2=√(2-√2)を掛け、
三日月形ABの面積を足す方針。
もしもPが2つずつ一致するなら、
ABの中点をNとして、
△OAB=NB・(ON±√2)
扇形OAB=NB・(ON±√2)+三日月形AB
NB=√(2-√2)
ON=√(OB^2-NB^2)
=2^2-(2-√2)
=√(2+√2)
扇形OAB=√(2-√2)√(2+√2)±√(2-√2)√2
=√2±√(4-2√2)
√は外せないか。
Pは4つあるかもしれない。
0022132人目の素数さん
垢版 |
2018/10/31(水) 10:08:52.50ID:iQsFQdVw
出てる答えがπr^2/2 + r^2だから
>>15
>答えが出てるとは思ってない。
てか?
まぁじゃ好きにすればいいけど。
いつになったら正解にたどり着くん?
ホントに東大卒?
0023132人目の素数さん
垢版 |
2018/10/31(水) 10:12:40.56ID:PPhF82WW
>>22
>πr^2/2 + r^2
おっとコレ大きい方の答えね。小さい方の答えはπr^2/2 - r^2。
出てる答えは条件みたすPは4ヶ所、面積は2通り。
まぁ頑張って下さいませ。
0024132人目の素数さん
垢版 |
2018/10/31(水) 12:26:50.70ID:DLpd368O
ちがった。現在上がってる答えはπr^2/8 ± r^2/4だった。
r = 2 なら π/2 ± 1 = 2.57、0.57。
0025イナ ◆/7jUdUKiSM
垢版 |
2018/10/31(水) 20:00:52.00ID:aaz9J1TG
>>21題意の文章のとおりだと、>>13であってると思うんだけど、図を優先すると円Oの半径が2√2となり、値は変わる。面積は2倍になる。
Pが2つとして、
ABの中点をNとすると、
△PAB=NB・(ON±2)
NB=√(4-2√2)
ON^2=OB^2-NB^2
=(2√2)^2-(4-2√2)
=4+2√2
△PAB=√(4-2√2)・{√(4+2√2)±2}
=√8±2√(4-2√2)
=2√2±2√(4-2√2)
=2.828427±2.1647844
=4.9932114
または0.6636426
三日月形AB=扇形OAB-△OAB
=3.14-2√2
=0.311572875
∴扇形PAB=4.9932114+0.311572875
=5.3047842cu(5.30cu)
または扇形PAB=0.6636426+0.311572875
=0.9752154cu(0.975cu)
0026132人目の素数さん
垢版 |
2018/10/31(水) 21:48:18.01ID:S59Tt0Nc
一応オリジナルなんだけどもしかして有名問題だったりするのかしらと一抹の不安を感じながら投稿

実数から実数への連続関数fは有界であり、任意の実数xに対して
f(x+√2)=(f(x)+f(x+1))/2
を満たす。この時、fは定数関数であることを示せ。
0027132人目の素数さん
垢版 |
2018/11/01(木) 00:37:19.95ID:B/Irv09c
分かすれ447の888より未解決

fを実係数n次多項式、s_0,s_1,...,s_nを相異なる実数とすると
f(x+s_0),f(x+s_1),f(x+s_2),...,f(x+s_n)は一次独立であることを示せ
0028132人目の素数さん
垢版 |
2018/11/01(木) 01:42:46.87ID:x6bVfA2R
>>27
f(x) = Σc[j]x^j とおき Σ[i] a[i] f(x+s[i]) = 0 とする。
Σa[i] c[j] (x + s[i])^j = 0 である。
n-k次の係数は
Σa[i] c[n] C[n k] s[i]^k
+Σa[i] c[n-1] C[n-1 k-1] s[i]^(k-1)
+Σa[i] c[n-2] C[n-2 k-2] s[i]^(k-2)
……
+Σa[i] c[n-k] C[n-k 0] s[i]^(k-k)
でこれが0であるから帰納的に
Σa[i] s[i]^k = 0 (k=0,1,…,n)
である。
ここでVandelmonde(s[i]^k)の行列式は零でないからa[i] = 0である。
0029132人目の素数さん
垢版 |
2018/11/01(木) 02:18:37.45ID:GatmQtrC
>>25
もし>>13があってたら
△PAB=√2±√(4-2√2)
三日月AB = π/2-√2
で答えが
π/2±√(4-2√2)
になるやん?
中学受験の答えがこんなんになるはずないでしょ?
半径が2√2でも答え2倍になるだけだからありえへんでしょ?
やり直し。
0030sage
垢版 |
2018/11/01(木) 04:00:36.26ID:xVnRbBm5
>>27
nについての帰納法で。
n=1 のとき (略)

n-1 に対して成立したとする。
 n次の多項式f(x)に対し、因数定理より
 f(x+) - f(x) = g(x),
 g(x) は 高々n-1次の多項式で、係数はf(x)の場合と同様。
いま
 Σ[k=0,n] c_k f(x+s_k) = 0,
とする。x^n の係数を比べて
 Σ[k=0,n-1] c_k + c_n = 0,
だから
 0 = Σ[k=0,n] c_k f(x+s_k)
 = Σ[k=0,n-1] c_k {f(x+s_k) - f(x+s_n)}
 = Σ[k=0,n-1] c_k (s_k-s_n)g(x+s_k)
帰納法の仮定により g(x+s_k) は1次独立。(k<n)
∴ c_k (s_k - s_n) = 0,  (k<n)
題意により s_k - s_n ≠ 0, (k<n)
∴ c_k = 0   (k<n)
∴ c_n = -Σ[k=0,n-1] c_k = 0,
∴ f(x+s_k) も1次独立。(0≦k≦n)
0031132人目の素数さん
垢版 |
2018/11/01(木) 04:11:28.52ID:GatmQtrC
>>30
>f(x+) - f(x) = g(x),

これあかんやろ?
f(x+) - f(x)は凾ナくくれるけど竸2以上の項があるからくくったg(x)にも凾ェのこるからg(x)は凾ノ依存する。
つまり正確には
f(x+) - f(x) = g(,x)
になる。
すると
>= Σ[k=0,n-1] c_k (s_k-s_n)g(x+s_k)
のところは
Σ[k=0,n-1] c_k (s_k-s_n)g(s_k-s_n,x+s_k)
になりg(s_k-s_n,x+s_k)は単純に同一のn-1次式をシフトしただけの組ではない。
0032132人目の素数さん
垢版 |
2018/11/01(木) 06:01:46.13ID:T2UjLCSb
以前に出題された「体積が最大になる表面積1の多面体」(面白いスレ26:420〜)について
計算でそれらしい解を出していたのだけど、今回それらを視覚化してみたので投下しておきます。

内接球との接点から頂点までを結んだ線を追加していますが、稜を挟んで向かい合う三角形はそれぞれ対称であるはず。

8面体 http://imgur.com/wYhPa1z.gif
9面体 http://imgur.com/yIbytVb.gif
10面体 http://imgur.com/rESWAMh.gif
11面体 http://imgur.com/airhHDk.gif
12面体 http://imgur.com/Cb0Go9K.gif
13面体 http://imgur.com/G7QDmpg.gif
14面体 http://imgur.com/G6jsj9s.gif
15面体 http://imgur.com/EYjweqv.gif
16面体 http://imgur.com/imr1D2Q.gif

17〜19面体 略

20面体を作ってみたところ、12枚ある五角形が野球ボールの縫い目のように整列しているような形が出てきました。

20面体 http://imgur.com/006j3bX.gif
0033132人目の素数さん
垢版 |
2018/11/01(木) 06:08:04.77ID:T2UjLCSb
なお、>>32 の各図形について最大であるかどうかの証明はしていません。
何回か試行して他に良い解がなく、かつ対称性が高いものを採用しています。
0034132人目の素数さん
垢版 |
2018/11/01(木) 06:31:27.51ID:IlhZTSyF
  ★★★For those who fight red dogs, weather manipulation is a daily routine. Kn▲owing weather is always with malice, we can reduce the risks.★★★▲

  この掲示板(万有サロン)▲に優秀な書き込みをして、総額148万円の賞金をゲットしよう!(*^^)v
▲ ▲ http://jbbs.livedoor.jp/study/3729/ →リンクが不良なら、検索窓に入れる!
0035イナ ◆/7jUdUKiSM
垢版 |
2018/11/01(木) 12:26:31.61ID:BwjvOPT2
>>29一瞬√2が消えるのにまた出てくるジレンマ。

>>25
問題文の「正方形に入る」は、「正方形が入る」なのかな?

いずれにしろ半径が√2倍になったら面積は2倍。出題者の考えを聴こう。これ以上やってもおもんない問題になる。
0036132人目の素数さん
垢版 |
2018/11/01(木) 12:52:25.04ID:YxMVM+RV
@方程式 x⁴+y⁴=6z⁴+12w⁴ は, (x,y,z,w)=(0,0,0,0)以外の有理数解を持たないことを示せ.

AXをk+m次正定値エルミート行列とせよ.
X=[[ A, C], [C^*, B] ]
とブロック行列表示せよ.
ここで, Aはk次行列, Bはm次行列, Cはk×m行列であり,
C^*はCの随伴行列である.
このとき,
det(X) ≦ det(A) det(B)
を示せ.
0040132人目の素数さん
垢版 |
2018/11/03(土) 02:50:14.57ID:Ha92ty6K
広場に3つの扉がある。
扉の見た目はいずれも同じ。

ひとつは「一瞬で天国に行ける」扉
ひとつは「1日の間ずっと広場から出られない」扉
ひとつは「2日の間ずっと広場から出られない」扉

扉は、開けた瞬間に効果があらわれる。

扉の効果があらわれると、すぐに扉は閉じられ位置がランダムにシャッフルされる。
そのため、前回の位置にある扉が今回も同じとは限らない。

さて、この広場を訪れた者は平均何日で天国にたどり着けるだろうか?
0041132人目の素数さん
垢版 |
2018/11/03(土) 03:06:50.27ID:6u03sBH6
平均 A 日で天国に行けるとすると
 A = 1/3・0 + 1/3・(A + 1) + 1/3・(A + 2)。
 ∴ A = 3。
0042132人目の素数さん
垢版 |
2018/11/03(土) 05:56:24.41ID:/E6xXixt
〔問題2926〕
mを正の整数とする。
θを 0≦θ≦π/2 の範囲で動かすとき、次の関数の値域を求めよ。
 f_m(θ) = √{1 -(sinθ)^m} + √{1 -(cosθ)^m}.

http://suseum.jp/gq/question/2926
0043132人目の素数さん
垢版 |
2018/11/03(土) 05:57:53.47ID:/E6xXixt
〔問題2931〕
ΔABCにおいて、
 辺ABを f:(1-f), g:(1-g) に内分する点をそれぞれ X,Y
 辺BCを f:(1-f), g:(1-g) に内分する点をそれぞれ Z,W
 辺CAを f:(1-f), g:(1-g) に内分する点をそれぞれ U,V
とする。(0<f≠g<1)
このとき ΔXZU と ΔYWV が相似であれば、ΔABCは正三角形であることを示せ。

http://suseum.jp/gq/question/2931
0044132人目の素数さん
垢版 |
2018/11/03(土) 06:00:39.16ID:/E6xXixt
〔問題2937〕
僊BCで AB=7,AC=4,また辺BC上の1点Dについて AD=7/2 である。
BD、CD がともに正の整数であるものとして、BDの長さを求めよ。

http://suseum.jp/gq/question/2937
0045132人目の素数さん
垢版 |
2018/11/03(土) 06:03:25.27ID:/E6xXixt
〔問題2938〕
3個のサイコロA,B,Cをこの順で1度づつ振る。
いちばん小さい出目が2のとき、いちばん大きい出目が4である確率はいくらか?

http://suseum.jp/gq/question/2938
0046132人目の素数さん
垢版 |
2018/11/03(土) 08:27:56.87ID:zSMa/Wom
>>41
エレガントな解だなぁ。

早速、シミュレーションしてみました。
> door <-function(){
+ stay=x=sample(0:2,1)
+ while(x!=0){
+ x=sample(0:2,1)
+ stay=append(stay,x)
+ }
+ sum(stay)
+ }
> re=replicate(1e3,mean(replicate(1e3,door())))
> summary(re)
Min. 1st Qu. Median Mean 3rd Qu. Max.
2.606 2.914 3.003 3.001 3.083 3.427
0047132人目の素数さん
垢版 |
2018/11/03(土) 09:19:53.97ID:zSMa/Wom
サイコロをふって1の目がでたら終了。 終了までにでた目の総和の期待値はいくらか?
0049132人目の素数さん
垢版 |
2018/11/03(土) 11:35:59.53ID:zSMa/Wom
>>45
3/15=1/5

x=[[a,b,c]|a<-[1..6],b<-[a..6],c<-[b..6],minimum[a,b,c]==2]
length x --15
length $ elemIndices 4 (map maximum x) -- 3
0050132人目の素数さん
垢版 |
2018/11/03(土) 11:39:29.08ID:zSMa/Wom
>>49
これは間違い、組み合わせじゃなくて順列にしなくちゃいけなかった。

x=[[a,b,c]|a<-[1..6],b<-[1..6],c<-[1..6],minimum[a,b,c]==2]
length x -- 61
length $ elemIndices 4 (map maximum x) --12

12/61
0051132人目の素数さん
垢版 |
2018/11/03(土) 12:08:51.34ID:zSMa/Wom
>>47
サイコロをふって1の目がでたら終了。 

(1)終了までにでた目の総和の期待値はいくらか?

(2)総和が50以上になる確率はいくらか?

(2)はどうやって解けばいいんだろ?

場合分けして1〜49まで場合分けして余事象でだすしかないのだろうか?
0052132人目の素数さん
垢版 |
2018/11/03(土) 15:16:07.91ID:OVkXWZOI
>>51
そうだと思うけど…
SageMath:
,var x
f = (x/6)/(1-(x^2+x^3+x^4+x^5+x^6)/6)
1-f.taylor(x,0,49).subs(x=1)

2887816213848518927/28430288029929701376
0.101575341438975
0053132人目の素数さん
垢版 |
2018/11/03(土) 15:31:30.50ID:OVkXWZOI
>>51-52
おおよそなら、1回1以外の目が出るごとに平均して総和が4大きくなるから、
総和が50以上になるのは12〜13回1以外の目がでればいい。
(5/6)^12 = 0.112156654784615
(5/6)^13 = 0.0934638789871792
近似としてはまあまあか?
0055132人目の素数さん
垢版 |
2018/11/03(土) 17:47:45.86ID:OVkXWZOI
>>54
なるほど! そうすればできるね。
,var n
R.<x> = CC['x']
g = (x/6)/(1-(x^2+x^3+x^4+x^5+x^6)/6)/(1-x) #x^nの係数は総和がn以下になる確率
pfd = g.partial_fraction_decomposition()
p = sum(c.numerator()/c.denominator().subs(x=0)*(-1/c.denominator().subs(x=0))^n for c in pfd[1]) #pは総和がn以下になる確率

sage: p
-0.924602908258674*e^(-0.0450727249412852*n)
- (0.0141317451913899 + 0.0178673357104935*I)*e^(-(0.329151925064039 - 1.18446726809051*I)*n)
- (0.0141317451913899 - 0.0178673357104935*I)*e^(-(0.329151925064039 + 1.18446726809051*I)*n)
- 0.0172163389039878*e^(-(0.361410021965218 - 3.14159265358979*I)*n)
- (0.0149586312272785 + 0.00913725707805078*I)*e^(-(0.363486436096737 - 2.18509474953750*I)*n)
- (0.0149586312272785 - 0.00913725707805078*I)*e^(-(0.363486436096737 + 2.18509474953750*I)*n)
+ 1.00000000000000

sage: 1-p.subs(n=49)
0.101575341438976 + 5.79026428787119e-24*I
0056132人目の素数さん
垢版 |
2018/11/03(土) 18:27:17.22ID:zSMa/Wom
能力的についていけない、コードと議論なんだけどRでのシミュレーション解(1000回の頻度の1000回平均値)と一致しております。

> dice = function(){
+ total=x=sample(6,1)
+ while(x!=1){
+ x=sample(6,1)
+ total=total+x
+ }
+ total
+ }
> re50=replicate(1e3,mean(replicate(1e3,dice()>=50)))
> summary(re50)
Min. 1st Qu. Median Mean 3rd Qu. Max.
0.0740 0.0950 0.1010 0.1013 0.1070 0.1350
0057132人目の素数さん
垢版 |
2018/11/03(土) 21:23:52.08ID:OVkXWZOI
>>56
生成関数をTaylor展開で1を代入は場合分けしているのも同然だし、nが変わるごとn回微分するのはコストがかかる。
部分分数分解すればn乗ですむ、というのが>>54の指摘。
生成関数の部分分数分解は基本ともいえるのにでなかったのが恥ずかしい。
0058132人目の素数さん
垢版 |
2018/11/04(日) 00:25:20.63ID:1wP06nNi
結局有理関数のテイラー展開の係数だからn項間関係の漸化式とけばいいんだね。

Prelude Data.List Data.Ratio> let ns n d = map head $ iterate(¥p -> (++[0] )$ tail $ zipWith (-) (p ++ (repeat 0)) $ map ((*) $ (head p)/(head d)) d) n
Prelude Data.List Data.Ratio> fromRational $ (!!49) $ ns [0,1%6] [6%1,-6%1,-1%1,0,0,0,0,1%1]
0.8984246585610248
Prelude Data.List Data.Ratio> 1 - it
0.10157534143897518
0059132人目の素数さん
垢版 |
2018/11/04(日) 01:08:35.11ID:1wP06nNi
訂正
Prelude Data.List Data.Ratio> let ns n d = map ((/(head d)).head) $ iterate(¥p -> tail $ zipWith (-) (p ++ (repeat 0)) $ map ((*) $ (head p)/(head d)) d) n
Prelude Data.List Data.Ratio> fromRational $ (!!49) $ ns [0,1%1] [6%1,-6%1,-1%1,0,0,0,0,1%1]
0.8984246585610248
Prelude Data.List Data.Ratio> 1 - it
0.10157534143897518
0060132人目の素数さん
垢版 |
2018/11/04(日) 04:19:27.45ID:RDBCTf5Y
答えが1/4じゃなくて10/49なのはトランプ問題
では、答えが4/25じゃなくて20/61なのは何問題?
0062132人目の素数さん
垢版 |
2018/11/04(日) 08:18:37.88ID:VDxltAIF
>>59
>51,56です。
いつも簡潔なHaskellのコードをありがとうございます。
自分にはか初見and/or失念のコマンドを調べながら勉強してます。
0064132人目の素数さん
垢版 |
2018/11/04(日) 13:20:58.52ID:t5LZe7yy
>>57
すみません。
そもそもテイラー展開が出てくる理屈からしてわからないものです。
0066132人目の素数さん
垢版 |
2018/11/04(日) 16:33:37.60ID:5kr3GpiZ
答え載せてるんだから、コレよりいい解答を見つけよが題意じゃね?
答えだけ書いてどうする?
しかも十分性成り立ってないし。
0067132人目の素数さん
垢版 |
2018/11/04(日) 16:41:52.33ID:lTCeMsqQ
>>51
総和の分布ってパラメータ1/21の指数分布になるみたいだな。

グラフにするとそんな感じだけど証明はわからんのであしからず。
0068132人目の素数さん
垢版 |
2018/11/04(日) 16:56:27.49ID:2EqmTiCY
>>44
△ABC∽△DACになるようにしたら相似比2:1だからBC=8, CD=2 となってBD=6。
それしかないかを調べるにはどうするのがよいだろうか?
0069132人目の素数さん
垢版 |
2018/11/04(日) 17:03:14.33ID:2EqmTiCY
>>64
生成関数のべき級数展開をx^49の項まで求めているだけです。
ここでの生成関数はx^nの係数が総和がnになる確率です。
0070132人目の素数さん
垢版 |
2018/11/04(日) 17:34:30.63ID:tTiGqsss
>>44

AB=7, AC=4, AD=7/2,

BC < AB + AC = 7 + 4 = 11,

BC = BH + CH
= √(AB^2-AH^2) + √(AC^2-AH^2)
≧ √(AB^2-AD^2) + √(AC^2-AD^2)
= (7√3 + √15)/2
= 7.99867

∴ 8 ≦ BC ≦ 10
0072イナ ◆/7jUdUKiSM
垢版 |
2018/11/04(日) 18:41:40.54ID:eJosweG2
>>65△ABCを描き、BC上にAD=7/2なるDをとると、
x=BDの条件は、
AB-AD<BD<AB+AC
7-7/2<x<7+4
よってBDは4、5、6、7、8、9、10のいずれか。
AからBCに垂線AHを引くと、
AC^2-CH^2=AB^2-BH^2
=AD^2-DH^2
4^2-(y+z)^2=7^2-(x+y)^2
=(7/2)^2-y^2
16-(y+z)^2=49-(x+y)^2
=49/4-y^2
xとyについて解くと、
196-4(x^2+2xy+y^2)=49-4y^2
196-4x^2-8xy=49

8xy=147-4x^2
y=147/32-2=83/32
yとzについて解くと、

BD=4のとき、y=DH、z=CH-yとすると、
DC=DH+HC=y+z=83/16+z
=83/16+CH-y
――中略―――――
BD=4、5、6はいずれもNG
――中略―――――
x=BD=7のとき、
49/4-y^2=7^2-(7-y)^2
=49-(49-14y+y^2)
49/4=14y
y=7/8
CH=√{16-(49・15/64)}
={√(1024-735)}/8
=(√289)/8
=17/8
y=DH=√{(7/2)^2-(7√15/8)^2}
=7/8
BH=√{7^2-(7√15/8)^2}
=49/8
BD=BH+DH=49/8+7/8=56/8=7
CD=CH-DH=17/8-7/8=10/8=5/4
BD=7はNG
x=BD=8、9、10は未調査。
この中にある可能性がある。
0073132人目の素数さん
垢版 |
2018/11/04(日) 18:49:12.73ID:LooLpWav
そもそも答え出すだけなら

(x^2+3.5^2-7^2)/7x=-(y^2+3.5^2-4)^2/7y
3.5<x<10.5, 0.5<y<7.5

の整数解出すだけだから問題としては大して難しいわけでもない。
エレガントなやつを求められてる。
0075イナ ◆/7jUdUKiSM
垢版 |
2018/11/04(日) 19:45:58.67ID:eJosweG2
>>72
BDが整数のときCDも整数になったらそれが答えなんだが、BD=8、9、10でもならなんだ。
しらみ潰しに調べてしらみを潰しきった。
∴解なし
または計算間違い。
おそらく同じ図を使ったときAH辺り同じ数字を使った可能性がある。
0076132人目の素数さん
垢版 |
2018/11/04(日) 22:35:51.00ID:zevnpesP
>>75
AB=7,AC=4,AD=7/2,BD=x,CD=y
cos(∠ADB)+cos(∠ADC)=0→(x^2+49/4-49)/(7x)+(y^2+49/4-16)/(7y)=0
正整数解は(x,y)=(6,2)
0077イナ ◆/7jUdUKiSM
垢版 |
2018/11/04(日) 23:05:37.18ID:eJosweG2
>>75見落とし、6を。
x=BD=6のとき、
AB^2-BH^2=AD^2-DH^2=AC^2-CH^2
7^2-(x+y)^2=(7/2)^2-y^2=4^2-z^2
49-x^2-2xy-y^2=49/4-y^2=16-z^2
xとyについて解くと、
49-x^2-2xy-y^2=49/4-y^2
(3/4)49-x^2-2xy=0
y=(3/8x)49-x/2
x=6を代入し、
y=49/16-3
=1/16
yとzについて解くと、
(7/2)^2-y^2=4^2-z^2
49/4-y^2=16-z^2
z^2=16-49/4+y^2
z^2=15/4+y^2
y=1/16を代入すると、
z^2=15/4+1/256
=(15・64+1)/256
=(960+1)/256
=961/256
z=31/16
CD=y+z=1/16+31/16=2
BDもCDも整数ゆえOK。
0079132人目の素数さん
垢版 |
2018/11/05(月) 15:04:22.76ID:gJu+KUPZ
数列{a_n}は
a_1=1
a_(3n+1)=a_(2n+1)
a_(3n-1)=a_(2n-1)
a_(3n)=-a_n
を満たす。この時、 lim_(n→∞) (1/n)Σ_(k=1,n)a_k を求めよ
0080132人目の素数さん
垢版 |
2018/11/05(月) 16:07:54.33ID:B1F8UTQM
>>70

BC が決まると、ピタゴラスの定理より BH,CH が出る。
 2BH - BC = BH - CH = (BH^2 - CH^2)/(BH+CH) = (AB^2 - AC^2)/BC = 33/BC,
 BH = (BC + 33/BC)/2,
 CH = BC - BH = (BC - 33/BC)/2,
さらに
 y = DH = √{BH^2 + AD^2 - AB^2} = √(BH^2 - 147/4),
 x = BD = BH - DH = BH - √{BH^2 + AD^2 - AB^2} = BH - √(BH^2 - 147/4),
または
 x = BD = BC - CH - √(CH^2 + AD^2 - AC^2) = BC - CH - √(CH^2 - 15/4),
これに BC = 8,9,10 を入れる。
0081132人目の素数さん
垢版 |
2018/11/05(月) 16:55:39.33ID:NWPSgxHY
サイコロを1000回ふったとき123456の順に並ぶめがある確率は?

(1000-6+1)/6^6=995/46656= 0.0213263

であってる?
0082132人目の素数さん
垢版 |
2018/11/05(月) 17:05:06.59ID:NWPSgxHY
>>81
10万回のシミュレーションで
> diseq = function(x) grepl("123456",paste(sample(6,x,rep=T),collapse=''))
> re=mean(replicate(1e5,diseq(k)))
> summary(re)
Min. 1st Qu. Median Mean 3rd Qu. Max.
0.02179 0.02179 0.02179 0.02179 0.02179 0.02179
なので多分、あってると思う。
0083132人目の素数さん
垢版 |
2018/11/05(月) 17:49:29.47ID:wfCkOOVj
>>81

24961325273729411157493296049923162050180048167808600668865194878033743728420689
84376099408174097463441629168520294258369368829106815003461566130708953839682920
95221171948291726439658114424799498019097661179266489639765057526270978013345104
76524747032016146895691394753020822407944413722991044460400808243936692906887421
58562789397002085900222149015685484765540355084031630923512566224026716368839141
57709132547009630669030748477906517799741669954712570078185561021427430325179405
72821546368625756251314713494685242945606761774980997529512510098234243941523221
41377931716188773349134288985450150950313965433089387066776103030489204172673462
73213105601812900585824575190324664251243051466881215047349049321238316686313754
951248352723962319494352745433967465925847447405514305001/6^1000
≒0.017620460571654540349...
0084132人目の素数さん
垢版 |
2018/11/05(月) 18:18:13.85ID:B1F8UTQM
>>81
「123456」を2つ以上含む場合を無視すれば合ってる。

「123456」の前後5回以内に「123456」はないから短時間の負相関があるが、それを無視すると
 1 - {1 - 1/(6^6)}^(1000-6+1) = 0.021100729
0085132人目の素数さん
垢版 |
2018/11/05(月) 18:29:06.48ID:6Utw5VZV
p[n]=n回目までに123456の並びが無い確率

p[n]=「n-1回目までに123456の並びが無い確率」-「n-1回目までに123456の並びが無く、n回目で123456が現れる確率」
=p[n-1]-p[n-6]/(6^6)

p[0]=p[1]=p[2]=p[3]=p[4]=p[5]=1

求める確率は 1-p[1000]

あとは任せた
0086132人目の素数さん
垢版 |
2018/11/05(月) 19:02:20.70ID:gJu+KUPZ
g[n]はn回目までに123456の並びがある確率
f[n]はn回目までに123456が出現してないが直前が12345で終わってる確率
e[n]はn回目までに123456が出現してないが直前が1234で終わってる確率

b[n]はn回目までに123456が出現してないが直前が1で終わってる確率
a[n]はn回目の時点で上のどれにも当てはまらない確率
とすると、
a[0]=1, b[0]=c[0]=d[0]=e[0]=f[0]=g[0]=0,
a[n+1]=(5/6)a[n]+(2/3)(b[n]+c[n]+d[n]+e[n]+f[n]),
b[n+1]=(1/6)(a[n]+b[n]+c[n]+d[n]+e[n]+f[n]),
c[n+1]=(1/6)b[n],
d[n+1]=(1/6)c[n],
e[n+1]=(1/6)d[n],
f[n+1]=(1/6)e[n],
g[n+1]=g[n]+(1/6)f[n]
となるから、あとはがんばる(他力本願)
>>85 も同じになるんかねこれ
0087132人目の素数さん
垢版 |
2018/11/05(月) 20:53:19.32ID:wfCkOOVj
>>83 
変換行列間違えました。

29894670002765580717622018953762664878384906585883227072416001286367327613872462
36176982687935042063489589508768499096701653359496110507628202771697652461481898
15867083862983975849719649919755556602395081603445217810191450771872978378492822
20929434295896536747523490245609702844253652551469152963247478735449528154607738
94897010212340239653386134088594267247751133592603591616944486751154360658915673
06079634567518636788329151870647053752023096491715169208527344676777484044463919
37957902455021334206833507250332593780127396266576899688383930527654264882692836
52659851277089138926126233125349416055818289864095231536195189335702592341232170
57986389166894334399077869160455544669456052004648395254763440350805452857924718
777798944034868387931340048957755897293189482452979797088/6^1000
≒0.021102960211841870224...
0090132人目の素数さん
垢版 |
2018/11/05(月) 22:15:57.52ID:1RAsBANL
>81と>87の差異はどういう違いでしょうか?
123456を1個含むか複数かの違いでしょうか?
複数許容なら数値の大きい方でいいのでしょうか?
0091132人目の素数さん
垢版 |
2018/11/05(月) 22:38:47.29ID:Pcec+Aw3
どかーん!
(⌒⌒⌒)
 ||

/ ̄ ̄ ̄ ̄ ̄\
| ・ U      |
| |ι        |つ
U||  ̄ ̄ ||
   ̄      ̄
呼んだ?
0092132人目の素数さん
垢版 |
2018/11/05(月) 22:53:10.54ID:wfCkOOVj
>>90
81の計算は、当選確率1/6^6のクジを995回引いたときの当選回数の期待値に等しい。
84の計算は、当選確率1/6^6のクジを995回引いたとき、少なくとも一回当たりくじを引く確率に等しい。

「目的の出目が六回続けて出る」という事が達成される確率が1/6^6。
普通のクジなら、外れを引くとクジ一個分を無駄にする。
しかし、この問題の場合は、いわば途中まで成功していた分のクジも無駄になることになる。
一回の失敗で、無駄になるクジの数が1枚の場合もあれば、複数の場合もある。
チャレンジできる回数が995回固定ということはない。

さらに、失敗したとしても、もし、1を引いていたら、新たなチャレンジの第一歩を
踏み出していたことになるが、それ以外の目で失敗したら、第0歩から
スタートすることになる。単純に「失敗」と言っても、内容が異なることもある。

本来はこのような機微に関わる問題で、厳密な値は、シンプルな式では表せない。

87や86は、6^1000通りある全てのパターンを想定している。
0093132人目の素数さん
垢版 |
2018/11/05(月) 22:59:51.87ID:hlCe+j6H
チャレンジできる回数が995回固定されないかも
ちょうど1000回使い切ることもありうる
0094132人目の素数さん
垢版 |
2018/11/05(月) 23:21:44.43ID:1RAsBANL
>>92
解説ありがとうございました。

>失敗したとしても、もし、1を引いていたら、新たなチャレンジの第一歩を踏み出していたことになる

この理解が私には欠けていました。
0095132人目の素数さん
垢版 |
2018/11/06(火) 02:48:39.54ID:jOazYBXJ
P1st Q1st even
[1,] 0 0 1
[2,] 4 5 6
[3,] 26 27 13
[4,] 84 83 23
[5,] 203 197 35
[6,] 413 398 50
[7,] 751 722 67
[8,] 1259 1210 87
[9,] 1986 1910 109
[10,] 2986 2875 134

完全追尾型多項式が完成しました

宝の個数を2で固定します

P1st ={12n^4+28n^3-42n^2-52n-3(-1)^n+51}/48

Q1st ={12n^4+20n^3-18n^2-20n-3(-1)^n+3}/48

■Wolframに入力すると既約分数表示になるので御注意

P1st/Q1st

={8(n-1){(n-2)n-6}/{2n(n+2)(6n^2-2n-5)-3(-1)^n+3}}+1
0096132人目の素数さん
垢版 |
2018/11/06(火) 03:09:24.93ID:FZJllfOU
>>85
任されたんぢゃ、生姜ねぇ…

線形漸化式は
 p[n] = p[n-1] - p[n-6] /(6^6),

特性多項式は
 t^6 - t^5 + (1/6)^6
= (t-α) (t-β) {tt -2Re(γ_1) t + |γ_1|^2} {tt -2 Re(γ_2) t + |γ_2|^2},

特性根は
α = 0.9999785642321302281427595561300279367871
  = 1 - (1/6)^6 - 5・(1/6)^12 - 40・(1/6)^18 - …

β = 0.11947305512892524659941083415872186721

γ_1 = |γ_1| e^(i θ_1)
 |γ_1| = 0.117113316705063892011642575051190099053
 θ_1 = 2.5253513177722176449

γ_2 = |γ_2| e^(i θ_2)
 |γ_2| = 0.11436934616195511830934529716995273057
 θ_2 = 1.279751470687185368

p[n] = 1.00000000689325α^(n-5) + c_0 β^n + c_1 |γ_1|^n cos(n θ_1+ d_1) + c_2 |γ_2|^n cos(n θ_2 + d_2)
   = 1.00000000689325α^(n-5)        (n>>1)
  
∵ 0.114 < |β|,|γ_1|,|γ_2| < 0.120 なので

1 - p[1000] = 1 - 1.00000000689325α^995 = 0.021102960211842


1.00000000689325 = 1 + 15 (1/6)^12 + …
0097132人目の素数さん
垢版 |
2018/11/06(火) 03:39:44.90ID:FZJllfOU
>>84

いったん「123456」が完成すると次の5回はデッド・タイムになるわけか。

GM計数管(放射線測定器)の分解時間、不感時間みたいなものかな?
0099132人目の素数さん
垢版 |
2018/11/06(火) 05:12:24.35ID:DN0vL5hu
haskell先生の答え
*Main> let ps = map (!!6) $ iterate (¥x->(tail x) ++ [x!!5 + 6%(6^6) - (sum $ take 6 x)/6^6]) $ [1%1,1%1,1%1,1%1,1%1,0,0,0,0,0,0]
*Main> fromRational $ ps !! 999
2.110296021184187e-2
0100132人目の素数さん
垢版 |
2018/11/06(火) 05:17:27.31ID:Er8xgC3V
>>98
あった
*Main> [(x,y)| x<-[4.0,4.5..10.0],y<-[1.0,1.5..7.0],y*(x^2+(3.5)^2-7^2)== -x*(y^2+(3.5)^2-4^2)]
[(4.5,4.5),(6.0,2.0)]
0101132人目の素数さん
垢版 |
2018/11/06(火) 05:50:54.52ID:FZJllfOU
>>86
g[n] の漸化式は
g[n] - g[n-1] = (1/6)f[n-1] = (1/6)^2 e[n-2] = (1/6)^3 d[n-3] = (1/6)^4 c[n-4] = (1/6)^5 b[n-5]
  = (1/6)^6 (1-g[n-6])
となります。したがって p[n] = 1 - g[n] の漸化式は
 p[n] = p[n-1] - (1/6)^6 p[n-6],
これは >>85 と同じです。

Memo.
 a[n] = p[n] - (1/6)p[n-1] - (1/6)^2 p[n-2] - (1/6)^3 p[n-3] - (1/6)^4 p[n-4] - (1/6)^5 p[n-5],
 b[n] = (1/6) p[n-1],
 c[n] = (1/6)^2 p[n-2],
 d[n] = (1/6)^3 p[n-3],
 e[n] = (1/6)^4 p[n-4],
 f[n] = (1/6)^5 p[n-5],
 g[n] = 1 - p[n],
0102132人目の素数さん
垢版 |
2018/11/06(火) 06:21:01.93ID:L5OqW8l+
Haskell 先生に聞いてみました。

Prelude Data.List Data.Ratio> let ps = map head $ iterate (¥x->(tail x) ++ [x!!5 - 1%(6^6) * x!!0]) [1%1,1%1,1%1,1%1,1%1,1%1-1%(6^6)]
Prelude Data.List Data.Ratio> fromRational $ 1%1 - (ps !! 999)
2.110296021184187e-2

確かにこっちの方がいいね。
0103132人目の素数さん
垢版 |
2018/11/06(火) 08:00:46.22ID:9NNsjRpE
>>102
こんな短いコードで算出できるとは驚きです。
計算原理がさっぱりわかりません。
個々のコマンドはなんとかわかります。
1-1/(6^6)は何の数値でしょうか?
0105132人目の素数さん
垢版 |
2018/11/06(火) 08:57:44.56ID:9NNsjRpE
>>102
ようやくコードの意味が理解できたのでRに移植。
実数計算なので誤差がでます。

f = function(N){
p=numeric()
p[1]=p[2]=p[3]=p[4]=p[5]=p[6]=1
for(n in 7:(N+1)){
p[n]=p[n-1]-p[n-6]/(6^6)
}
1-p[N+1]
}
> f(1000)
[1] 0.02110296
0107132人目の素数さん
垢版 |
2018/11/06(火) 12:59:18.08ID:poLu8oOg
>>103-106
もうわかってるみたいだけど参考までに。
>>102はHaskellで漸化式解くときの定石みたいなやつです。
簡単な例ではFibonacci数列を計算するとき
Prelude> let f x = if x <= 2 then 1 else (f $ x-1) + (f $x-2)
Prelude> [f x|x<-[1..30]]
でも
Prelude> let g = map head $ iterate (¥x-> (tail x) ++ [sum x]) [1,1]
Prelude> take 30 g
でもできますが、やってみると速度が段違いです。
なので速度優先のときは後者使います。
しかし後者のほうが優れてるわけではないんですよ。
計算そのものより理論が正しいかどうかとかの議論をしたい時とか、理論の正しさを明らかにしたい時とかなら可読性を優先して前者のほうがよい時もあります。
実際Haskellはその記述に厳しい “一貫性” を課してるので “書きにくいけど読みやすい” のが売りですからねぇ?
計算の数値だけが目的ならHaskell使う意味ありません。
TPO考えて使い分けないとです。
0108132人目の素数さん
垢版 |
2018/11/06(火) 15:40:04.34ID:cDO4b4Dm
コインを1000回投げた。連続して表がでる確率が最も高いのは何回連続するときか?
0109132人目の素数さん
垢版 |
2018/11/06(火) 17:36:12.86ID:/OX7Wfwz
0<|2x^2-y^3|<100√|y|
を満たす整数の組(x,y)が無限に存在することを示せ
0110132人目の素数さん
垢版 |
2018/11/06(火) 17:39:00.56ID:FZJllfOU
まとめ(?)

ε = 1/(6^6) とおく。

・「123456]を2つ以上含む場合を無視 >>81
 1 - p[n] ≒ (n-5)ε

・複数あり、相関を無視  >>84
 1 - p[n] = 1 - (1-ε)^{n-5} = (n-5)ε - (1/2)(n-5)(n-6)ε^2 + (1/6)(n-5)(n-6)(n-7)ε^3 - …

・複数あり、相関あり  >>96
 α = 1 -ε -5ε^2 -40ε^3 -385ε^4 -4095ε^5 - …
 1 - p[n] = 1 - (1 + 15ε^2 + …)α^{n-5} = (n-5)ε - (1/2)(n-10)(n-11)ε^2 + …

Python? Haskell?
プログラムの作れないオジサンは Excel で1000行 使ってますよ。。。
0111132人目の素数さん
垢版 |
2018/11/06(火) 19:13:07.74ID:0/M2gc6l
>>108
日本語の微妙なニュアンスは、それで合ってるの?

100回のコイントスで連続表の最大回数は
5回がもっとも起こりやすく確率は26% らしいけど
0112132人目の素数さん
垢版 |
2018/11/06(火) 19:32:20.35ID:3z562f7u
>>108
1000回コインを振って、11連以上、10連以上、...、7連以上の表が出る確率はそれぞれ、
0.215431673
0.385449752
0.624240992
0.861144809
0.981783332
なので、最大連続数が、10連、9連、8連、7連となる確率は、それぞれ、
0.170018079
0.23879124
0.236903817
0.120638523
8連と9連がほぼ等しいが、9連となる確率が最も高い。

参考
https://rio2016.5ch.net/test/read.cgi/math/1540218853/462-465
0113132人目の素数さん
垢版 |
2018/11/06(火) 20:00:33.66ID:cDO4b4Dm
>>112
100回で5
1000回で9
10000回で12
10万回で15
1000万回で18になった。
0115132人目の素数さん
垢版 |
2018/11/06(火) 20:07:29.69ID:FZJllfOU
a = 2^(1/6) は無理数だから、ディリクレの定理より
 |p/q - a| < 1/q^2

(「ディオファントス近似」とかいうらしい。)

x = q^3,y = pp とおくと
0 < | 2xx - y^3 | = | 2q^6 - p^6 | < ?

う〜む、近似が足りぬ…
0116132人目の素数さん
垢版 |
2018/11/06(火) 20:14:11.95ID:FZJllfOU
>>115

では、ニュートン・ラフソン法を使おう。

 f(x) = (x^6 -2) / x^(5/2),
として
 x ' = x - f(x) / f '(x) = x - 2x(x^6 -2)/(7x^6 +10) = x(5x^6 +14)/(7x^6 +10),
を繰り返す。このとき
 x '- a = (x-a)^3 * (5x^4 +8ax^3 +9aaxx +8aaax +5a^4)/(7x^6 +10)
∴ | x '- a | < C |x-a|^3,
さて…
0117132人目の素数さん
垢版 |
2018/11/06(火) 20:32:32.09ID:cDO4b4Dm
>>113
100万回で18の間違い。
ちなみに1000万回で22回(確率は0.2474)だった。
0119132人目の素数さん
垢版 |
2018/11/06(火) 22:06:22.80ID:0/M2gc6l
ある道路では、30分以内に車が通る確率は99.9%である。
では、10分以内に車が通る確率は?
0121132人目の素数さん
垢版 |
2018/11/06(火) 23:36:10.05ID:08uZxk9P
>>115
それを満たすp q が無限にあったとしてもそれらが平方数、立方数になってくれてる保証なんかないでしょ?
0122132人目の素数さん
垢版 |
2018/11/07(水) 00:32:00.28ID:p6NUZQ5G
>>105
分数表示したいのでPythonに移植した。

from fractions import Fraction

def dice126(N):
P=list()
for n in range(6):
P.append(1)
P.append(1-1/(6**6))
for n in range(7,N+1):
P.append(P[n-1]-P[n-6]/(6**6))
return(1-P[N])

def dice123456(N):
print(Fraction(dice126(N)))
print(" = " + str(dice126(N)))

dice123456(1000)
0123132人目の素数さん
垢版 |
2018/11/07(水) 00:44:06.23ID:5PMwby1T
>>116 (補足)
 a = 2^(1/6) として
 0 < (5x^4 +8ax^3 +9aaxx +8aaax +5a^4)/(7x^6 +10) ≦ C,
 C = 2.706458005831039532180100595416
 等号成立は x = 0.903918268122918428596803223653869
0125132人目の素数さん
垢版 |
2018/11/07(水) 04:43:16.35ID:5PMwby1T
>>96 (補足)
 p[0] = p[1] = p[2] = p[3] = p[4] = p[5] = 1
より
 p[n] = 1.00000000689307114563713652919α^(n-5) + c_0 β^n + ……
  = (1 + 15ε^2 + 220ε^3 + 320ε^4 +…)α^(n-5) + c_0 β^n + ……

 c_0 = -0.02813048468
 c_1 = 0.03983999218
 d_1 = -0.51407117920
 c_2 = -0.05049060128
 d_2 = 1.43835771780
0126132人目の素数さん
垢版 |
2018/11/07(水) 09:16:36.43ID:5PMwby1T
>>110 (補足)

ε = 1/(6^6) とする。

p[n] ≒ (1 +15ε^2 +220ε^3+ 3060ε^4 + …) α^(n-5)
  = (1 +15ε^2 +220ε^3+ 3060ε^4 + …)(1 -ε -5ε^2 -40ε^3 -385ε^4 -4095ε^5 -…)^(n-5)
  = 1 -(n-5)ε +(1/2!)(n-10)(n-11)ε^2 -(1/3!)(n-15)(n-16)(n-17)ε^3 +(1/4!)(n-20)(n-21)(n-22)(n-23)ε^4 - …
0127132人目の素数さん
垢版 |
2018/11/07(水) 17:50:14.73ID:fQSBmb6Q
今んとこ解かれてないのは >>26 >>36 >>42 >>43 >>79 >>109 かね
真ん中二つはリンク先に答えあるみたいだしあれだけども
0128132人目の素数さん
垢版 |
2018/11/07(水) 19:51:09.21ID:5PMwby1T
>>110 (補足)

ε = 1/(6^6) とする。

α = 1 -ε -5ε^2 -40ε^3 -385ε^4 -4095ε^5 -46376ε^6 -548340ε^7 - …

p[n] = Σ[k=0,∞] C[n-1-5k,k] (-ε)^k,
0129132人目の素数さん
垢版 |
2018/11/07(水) 21:09:24.11ID:p6NUZQ5G
某シリツ医大の裏口入学調査委員会が裏口入学は高々10%と報告したとする。

その結果の検証に100人を調査したら4人続けて裏口入学生であった、という。
この検証から裏口入学率が10%であるか否かを有意水準1%で検定せよ。
0130132人目の素数さん
垢版 |
2018/11/07(水) 21:51:21.02ID:mEYJ0uIZ
>>129
>その結果の検証に100人を調査したら4人続けて裏口入学生であった、という。

これは100件の検証結果の中に
・1度だけ4連続裏口入学があった
・複数回4連続裏口入学があった
・1度だけ4以上連続裏口入学があった
・複数回4以上連続裏口入学があった
のどれを意味していますか?
0131132人目の素数さん
垢版 |
2018/11/07(水) 22:38:16.40ID:p6NUZQ5G
>>130
情報がないとして全ての場合を含む、
1回以上、4以上の連続裏口入学があった場合を考える。
4連続が1回でもいいし、
4連続が2回の後に5連続が1回でもいいとする。

想定したのは
表のでる確率が0.1のコインを100回投げて表が4回以上連続する確率が1%未満かという問題。
0134132人目の素数さん
垢版 |
2018/11/08(木) 02:34:17.78ID:45SX77TX
>>109 >>116

(x, y) = (q^3, pp) とおくと、求めるものは
0 < | 2q^6 - p^6 | < 100・p,
を満たす整数 (p, q)

そこで 2^(1/6) = a の近似分数の列 p/q = t を次の漸化式で定める。
 t ' = t - 2 t (t^6 - 2)/(7 t^6 + 10) = t (5 t^6 + 14)/(7 t^6 + 10),

(p,q) の漸化式は
 p ' = p (5p^6 + 14q^6),
 q ' = q (7p^6 + 10q^6),

(p,q) = (1,1) のときは成立するが

(p,q) = (19, 17) のときは 2xx -y^3 = 2・17^6 - 19^6 = 1229257 > 100*19

(p,q) = (10889952049, 9701846569) のときは 2xx -y^3 = 2q^6 - p^6 = 2.31865949E+54 > 100・p

(p,q) = (217953260587942275546675683149407795232019596416934847340158868299331811, 194174280472305108606358058802927185430436427469916728412097502845028473)
のときも絶望的でござる。
0136132人目の素数さん
垢版 |
2018/11/08(木) 04:07:51.53ID:pDHe6HSd
>>134
てかその方針そのものが無理なんじゃないの?
もし
>0 < | 2q^6 - p^6 | < 100・p,
>を満たす整数 (p, q)
が無限にあったら
0 < 2q^6 - p^6 < 100p 又は -100p < 2q^6 - p^6 < 0
になるけど前者なら t = 100p/q^6、b=(p/q)^(-5/6)とおいて
p/q < 2^(1/6) < (p^6/q^6 + 100/q^6)^(1/6) = p/q + tb/6 - 5tb/72 (bt) + 55tb/1296 (bt)^2 + ……
だけど誤差項がO(q^(-5))なので
>この系は、トゥエ・ジーゲル・ロスの定理が、代数的数の有理数での近似の下界は 2 を超えて 2 + ε への改善はできないという意味で、最良であることを示している。
に矛盾してしまう。
https://ja.wikipedia.org/wiki/%E3%83%87%E3%82%A3%E3%83%AA%E3%82%AF%E3%83%AC%E3%81%AE%E3%83%87%E3%82%A3%E3%82%AA%E3%83%95%E3%82%A1%E3%83%B3%E3%83%88%E3%82%B9%E8%BF%91%E4%BC%BC%E5%AE%9A%E7%90%86
後者でも同様。
0137132人目の素数さん
垢版 |
2018/11/09(金) 05:17:13.44ID:pvdoV3Z4
>>110 >>126 >>128

ε = 1/(6^6) とする。

p[n] = (1 +15ε^2 +220ε^3 +3060ε^4 +42504ε^5 +593775ε^6 +8347680ε^7 + … )α^(n-5)

α は t^6 - t^5 +ε = 0 の根

α = 1 -ε -5ε^2 -40ε^3 -385ε^4 -4095ε^5 -46376ε^6 -548340ε^7 -6690585ε^8 - …
0139132人目の素数さん
垢版 |
2018/11/09(金) 14:55:14.01ID:ds1M8gYh
>>79
分からないけど、{a_n}の母関数の関数等式ができたので一応書いとく

f(x)=Σ[n≧1] a_nx^n

g(x)=Σ[n≧1] a_(2n-1)x^(2n-1)

h1(x)=Σ[n≧1] a_(3n-2)x^(3n-2)

h2(x)=Σ[n≧1] a_(3n-1)x^(3n-1)

h3(x)=Σ[n≧1] a_(3n)x^(3n)

とおく。これらは区間 (-1,1) 上で絶対収束する。
まず明らかに
 h1(x)+h2(x)+h3(x)=f(x) …@
 (f(x)-f(-x))/2=g(x) …A
さらに漸化式より
 xh1(x^2)=g(x^3) …B
 h2(x^2)=xg(x^3) …C
 h3(x)=-f(x^3) …D

@に x^2 を代入して x 倍
 xh1(x^2)+xh2(x^2)+xh3(x^2)=xf(x^2)
BCDより
 g(x^3)+x^2g(x^3)-xf(x^6)=xf(x^2)
Aより
 (1+x^2)(f(x^3)-f(-x^3))/2-xf(x^6)=xf(x^2)
整理して
 (1+x^2)(f(x^3)-f(-x^3))=2x(f(x^2)+f(x^6))

うーん…
0142132人目の素数さん
垢版 |
2018/11/09(金) 19:33:26.08ID:ak/GsOoT
しらんの?わかスレでこれ答えでんやろって適当な問題連発してるやつ。
とくに彼がだしてる確率系は殆どとけない。
(というか持ってる答えあるなら出してくれといって出したことないのでそう推定している。)
>>79はいかにも彼が好きそうな形。
本人解けたつもりで出してるだけの可能性あり。
0144132人目の素数さん
垢版 |
2018/11/09(金) 22:58:54.45ID:5AnUTlVm
コインを100回投げて表が連続した最大数が5のとき、表がでる確率の95%信頼区間を求めよ。

近似解計算で

lower upper
0.2487456 0.6386493

になったけど、自信がない。
0145132人目の素数さん
垢版 |
2018/11/10(土) 01:10:06.43ID:TW6tyhOr
>>138
>t^6 - t^5 +ε = 0 の根は
>
>α = 1 - Σ[k=1,∞] c(5, k) ε^k,

これどうやって証明するんですか?
Link先にも載ってないですけど?
0147132人目の素数さん
垢版 |
2018/11/10(土) 02:17:14.74ID:BjsJwiKs
>>79は関数等式が作れたので満足ということにしよう

>>26
これも分からないけど考えたことを書いてみる。

区間 [0,√2] で関数の値を決めれば、
等式 f(x+√2)=(f(x)+f(x+1))/2 を満たすように実数全体に一意に拡張することができる。
したがって、[0,√2] 上で定数であることを示すことが必要かつ十分。

[0,√2] 上で定数でないと仮定して、有界でないことを導く感じかなあ。
x を大きくすると平均化されて収束しそうなので、逆に x を -∞ の方に持ってくと有界でなくなりそう。
0149132人目の素数さん
垢版 |
2018/11/10(土) 03:12:05.12ID:P9RJEHjc
諦め早いなあ じゃヒント
>>79
S(n) = Σ_(k=1,n) a_(2k-1) とおくと
S(3n) = Σ_(k=1,n) a_(6k-5) + a_(6k-3) + a_(6k-1)
= Σ_(k=1,n) a_(4k-3) - a_(2k-1) + a_(4k-1)
= S(2n) - S(n)
0150132人目の素数さん
垢版 |
2018/11/10(土) 03:46:00.82ID:P9RJEHjc
ちなみに言っとくと
>>142 人違いです 書き込んだことあるのはこの面白スレだけなので

あとついでにもう一問
整数 N に対し、rad(N) を N の互いに異なる素因数の積と定める。
正の整数 n に対して (1+√2)^n を展開した時の √2 の係数を a_n とおくと、
n を奇数の中から適切に選んで rad(a_n)/a_n を任意に小さくできることを示せ。
0152132人目の素数さん
垢版 |
2018/11/10(土) 05:09:34.22ID:0LaPCkg7
>>150

a_n = {(1+√2)^n - (1-√2)^n} / (2√2),

{(1+√2)^m + (1-√2)^m}/2 は自然数。

∴ a_{pq} は a_p および a_q で割り切れる。

さて、どうするか?
0153132人目の素数さん
垢版 |
2018/11/10(土) 05:35:01.43ID:EuCYu9xA
>>150
こっちの方はできたかな?

RをQ[√2]の整数環とする。
p≡3.5 (mod 8)である素数をとる。
x^2 - 2 = 0は mod p で解を持たないからQ/Zのpの拡大次数は2でpRはRの素イデアル。
とくにa+b√2∈p^iR ⇔ a∈p^iZ かつ b∈p^iZ である。
ここで n をR/p^2Rの乗法群の位数とするとき(1+√2)^n ≡ 1 (mod p^2R)であるからa≡1 (mod p^2) かつ b≡0 (mod p^2)である。
とくにこのとき rad b ≦ b/p であるから rad b / b ≦ 1/p となる。
p≡3.5 (mod 8)である素数は無数にあるから主張は示された。
0154132人目の素数さん
垢版 |
2018/11/10(土) 05:45:34.35ID:EuCYu9xA
あれ?素数取り直す必要ないか。
R/3^iRの乗法群の位数をnとすれば(1+√2)^n = a + b√2 とおくとき同様にしてb ≡ 0 (mod 3^i)だから
b / rad b ≦ 1/3^(i-1)でいいのか。
0156132人目の素数さん
垢版 |
2018/11/10(土) 05:59:55.44ID:EuCYu9xA
ありゃ?とすると代数的整数論のテクニック使う必要すらないや。
(1+√2)^(8・3^(i-1)) = a[i] + b[i]√2 とおいて a[i] ≡ 1 (mod 3^i)、b[i] ≡ 0 (mod 3^i) を帰納法で示せばいいだけだ。
0158132人目の素数さん
垢版 |
2018/11/10(土) 06:03:33.77ID:0LaPCkg7
>>152

n = (2^r) -1 のとき a_n は 2n-1 で割り切れるらしい。

といっても平方因子じゃないが…

a_7 = 13^2,
a_15 = 5^2・29・269,
0159132人目の素数さん
垢版 |
2018/11/10(土) 06:31:54.82ID:EuCYu9xA
>>155
とりあえず代数的整数論つかえば n が奇数もクリアできた。
p ≡ 3、5 (mod 8)にとっておけば p^2 ≡ 9 (mod 16)なのでRの乗法群の位数は16で割り切れない。
とくに 1+√2 + pR がある数の8乗であれば1+√2 + pRの位数は奇数である。
よって 方程式 x^8 - (1+√2) が R/pRで完全分解する素数pをとればよい。
そのような素数はチェボタレフ密度定理により無限にある。
0160132人目の素数さん
垢版 |
2018/11/10(土) 06:33:49.21ID:EuCYu9xA
かいたあとに気づく。orz。これも初等的にいける。けど、もういいや。これで。
0162132人目の素数さん
垢版 |
2018/11/10(土) 07:34:42.59ID:0LaPCkg7
>>158
どうやれば平方因子が(無限に)出てくるか、という問題らしいけど、サパーリです。
0163132人目の素数さん
垢版 |
2018/11/10(土) 11:22:16.94ID:QJ6NJqU7
コインを100回投げて表が連続した最大数が5のとき、
表がでる確率の期待値と最頻値および95%信頼区間を求めよ。
0164132人目の素数さん
垢版 |
2018/11/10(土) 12:31:29.30ID:8Fs/tUZA
>>159
それは題意とは別のことの証明みたい
nを奇数に制限した時に数列 a_n に素因数が無限に出現することの証明 これもこれですごいんだけど
0165132人目の素数さん
垢版 |
2018/11/10(土) 13:53:36.51ID:mkbHRdk3
>>155
初等的に。
(1+√2)^n = a[n] + b[n]√2 (a[n],b[n]∈Z) として b[3・5^i] ≡ 0 (mod 5^(i+1))をしめす。
i=0のとき(1+√2)^3 = 7+5√2より成立。
i=kで成立するとしてi=k+1のとき
b[3・5^(i+1)] = 5a[3・5^i]^4 b[3・5^i] + 10a[3・5^i]^2 b[3・5^i]^3 + b[3・5^i]^5
だから成立。
これを用いて
rad b[3・5^i]/b[3・5^i] ≦ 1/5^i。
0167132人目の素数さん
垢版 |
2018/11/10(土) 19:05:10.64ID:0LaPCkg7
>>150 >>165

n=3・5^i のとき
 a_n = {(1+√2)^n - (1-√2)^n}/(2√2) ≡ 0,  (mod 5^(i+1))
(略証)
iについての帰納法で
i=0、n=3 のとき
 a_3 = {(1+√2)^3 - (1-√2)^3}/(2√2) = 5 ≡ 0,  (mod 5)

m で成立するとして n=5m のとき
a_{5m} / a_m = {(1+√2)^(5m) - (1-√2)^(5m)} / {(1+√2)^m - (1-√2)^m}
= {(1+√2)^(4m) + (1-√2)^(4m)} + (-1)^m・{(1+√2)^(2m) + (1-√2)^(2m)} +1
= {64(a_m)^2 + 32(-1)^m}^2 +2} + (-1)^m・{8(a_m)^2 + 2(-1)^m} +1
= 64(a_m)^4 + 40(-1)^m・(a_m)^2 + 5
≡ 0 (mod 5),     (← a_m≡0)
だから n=5m でも成立。

ここで
(1+√2)^(2m) + (1-√2)^(2m) = {(1+√2)^m - (1-√2)^m}^2 + 2(-1)^m
 = 8(a_m)^2 + 2(-1)^m,
(1+√2)^(4m) + (1-√2)^(4m) = {(1+√2)^(2m) + (1-√2)^(2m)}^2 - 2
 = {8(a_m)^2 + 2(-1)^m}^2 - 2
 = 64(a_m)^2 + 32(-1)^m (a_m)^2 + 2,
を使った。
0168132人目の素数さん
垢版 |
2018/11/10(土) 22:23:10.80ID:P9RJEHjc
>>165 >>167
正解 実は自分もこのくらい初等的な証明の存在は投稿してから気づいた
>>166
しょうがないなあ

>>149 の続き
a_n の絶対値は全て1であるから n>m の時
|S(n)-S(m)| = |Σ_(k=m+1,n) a_(2k-1)|
≦ Σ_(k=m+1,n) |a_(2k-1)|
= n-m.
したがって、一般の n,m≧1 について
|S(n)-S(m)| ≦ |n-m|
が成り立つ。これと >>149 の式を組み合わせると…?
0170132人目の素数さん
垢版 |
2018/11/11(日) 00:17:10.76ID:6OpEPnNJ
あ、いや、なるほど!わかったかも!
でも偶数項もなんとかせねば!
0172132人目の素数さん
垢版 |
2018/11/11(日) 02:26:17.27ID:sLf3laj9
>>167 のようにおくと

a_{5m} = 64(a_m)^5 + 40(-1)^m・(a_m)^3 + 5 a_m,

(略証)
mについての帰納法による。
m=1 のとき
 a_1 = 1,a_5 = 29 だから成立。

m 以下で成立すれば…
a_{m+1} - a_{m-1} = 2a_m = 2a,
a_{m+1}a_{m-1} = (a_m)^2 + (-1)^m = aa + (-1)^m,
から
a_{m+1}^3 - a_{m-1}^3 = 2a[(2a)^2 +3{aa+(-1)^m}] = 14a^3 +6(-1)^m a,
a_{m+1}^5 - a_{m-1}^5 = 2a[(2a)^4 +5(4aa){aa+(-1)^m} +5{aa+(-1)^m}^2] = 82a^5 +60(-1)^m a^3 +10a,
が出る。また、
64(a_{m+1})^5 + 40(-1)^{m+1}・(a_{m+1})^3 + 5a_{m+1} - a_{5(m-1)}
 = 64(a_{m+1}^5 - a_{m-1}^5) - 40(-1)^m・(a_{m+1}^3 - a_{m-1}^3) + 5(a_{m+1} - a_{m-1})
 = 64{82a^5 + 60(-1)^m・a^3 + 10a} - 40{14(-1)^m a^3 + 6a} + 10a
 = 82{64a^5 + 40(-1)^m・a^3 + 5a}
 = 82 a_{5m},   (← 帰納法の仮定)
以上により
a_{5(m+1)} = 82a_{5m} + a_{5(m-1)} = 64(a_{m+1})^5 + 40(-1)^{m+1}・(a_{m+1})^3 + 5a_{m+1},
∴ m+1 でも成立する。
0173132人目の素数さん
垢版 |
2018/11/11(日) 03:18:51.59ID:/I0SuFdi
>>149
できたかも。
まず>>149を一般化して
S[3N] = S[2N] - S[N]
S[3N-1] = S[2N-1] - S[N]
S[3N+1] = S[2N+1] - S[N]
さらにT[N] = Σ[n≦2N, n:evev] a[n]とおいて
T[3N] = S[2N] - T[N]。
T[3N+1] = S[2N+1] - T[N]。
T[3N-1] = S[2N-1] - T[N]。
まず|S[N]|、|T[N]| ≦ N^(3/4)(log (3*N)))^2を示す。
f(x) = x^(3/4)(log (x+1))^2とおくときx≧1において多分
f(3x)≦f(2x) + f(x)、
f(3x+1)≦f(2x+1) + f(x)、
f(3x-1)≦f(2x-1) + f(x)
である。(∵パソコンでグラフ描いてみた)
よって成立。
よって
lim[n→∞](S[N]+T[N])/N = 0
である。
0176132人目の素数さん
垢版 |
2018/11/11(日) 05:41:25.27ID:/oKi5paQ
>>26が一応できたけど、証明が長くなった。
もし模範解答が短いなら書くだけ損なので、あまり書きたくないw
0177132人目の素数さん
垢版 |
2018/11/11(日) 05:44:17.33ID:/oKi5paQ
>>79は2種類の証明ができて、lim_(n→∞) (1/n)Σ_(k=1,n)a_k=0 が証明できた。
1つ目の方法は、正の実数xに対して S(x)=Σ(1≦k≦x) a(2k−1) と置いてから、
>>149の類似品を作って、それを展開しまくってたくさんのS(x)の和にしたあとに、
その和を適当に区切ってからそれぞれ評価して、
limsup_x S(x)/x と liminf_x S(x)/x を考える方法。
簡単だけど計算がごちゃごちゃしてて、8レスくらいになった。
計算ミスしてる可能性もある。

2つ目の方法は、>>149の類似品を使いながら、素数定理の簡単な証明
ttps://people.mpim-bonn.mpg.de/zagier/files/doi/10.2307/2975232/fulltext.pdf
と同じやり方を使う方法。実はこっちの方が先にできた。これは7レスくらい。
0179132人目の素数さん
垢版 |
2018/11/11(日) 10:22:49.91ID:6OpEPnNJ
そうなんだよね〜
ディリクレ級数はまず真っ先に思いつくのはつくんだけど、母関数と違って関数等式作れなくてs=1近傍での振る舞いが確定できなかった。
どうやったんですか?
関数等式作れました?
0180132人目の素数さん
垢版 |
2018/11/11(日) 13:14:33.63ID:FVh8W5vn
>>177
んーなんかできてるっぽいね やや略式ではあるけど一応答え載せときます
(これからも答え複雑になりそうな時はこんくらい省いても問題ないんじゃないかな まあでもこれは個人の感覚か)

>>149 を使って、例えば
S(9n) = S(6n)-S(3n) = S(6n)-S(2n)+S(n) = S(4n)-2S(2n)+S(n)
のように、S(3N) を S(2N)-S(N) に置き換える操作を繰り返していく時、各辺を
Σ_(c∈C) σ(c)S(cn) (Cは正整数の有限集合、各cに対してσ(c)は整数)
とおいた時の Σ_(c∈C) |σ(c)c| の値は操作により増加しないことがわかる。
(このことは、操作を適用する項だけに着目して、その項の操作前後の変化を考えればわかる)
(上の例では 9 ≧ 6+3 ≧ 6+2+1 ≧ 4+2*2+1)
したがって、a>b>0 かつaとbの偶奇が一致しない時
S((3^a)n) = S((2^a)n) - S((3^b)n) + (それ以外)
と展開することができることから、limsup |S(n)/n|=μ とおくと >>168 から
(3^a)μ ≦ |2^a-3^b| + limsup((それ以外)/n)
≦|2^a-3^b| + (3^a-2^a-3^b)μ.
よって、μ ≦ |2^a-3^b|/(2^a+3^b).
ディリクレのディオファントス近似定理より
|mlog_3(2) - (1/2)log_3(2) - m'| (m,m'は十分大きい正の整数) は任意に小さくすることができるので、
a=2m-1, b=2m' とおけば a,b は条件を満たし、|2^a-3^b| も (2^a+3^b) と比べて任意に小さくなる。
ゆえに、μ=0.
さらに >>173 の T(n) について limsup|T(n)/n|=ν とおくと、
3ν = limsup|T(3n)/n| ≦ limsup|S(2n)/n| + limsup|T(n)/n| = ν
から、ν=0. 以上より、示された。
0181132人目の素数さん
垢版 |
2018/11/11(日) 14:15:47.83ID:FVh8W5vn
よければ >>177 の手法ももうちょい詳しく聞いてみたいな

ここからはやや余談。コラッツの問題は
f(n)=n/2 (if 2|n), 3n+1 (otherwise)
とおいた時 f の合成による値の挙動を問う問題でご存知の通りまだ未解決なんだけど、
kを正の整数として『(1に到達するまでの操作の回数)mod k』で自然数を分類した時に何か言えないか?
を考えることができるのではと思い、感触をつかむためまず手始めに
g(n)=n/3 (if 3|n), 2n+1に最も近い3の倍数 (otherwize)
とした時にできる同類の問題を考えてみた、というのが >>79 の問題。
どんな自然数もgの合成でいずれ1に到達することは簡単にわかるけど、
それでもこの問題の(思いつく限り簡単な)解は >>180 のようにやや込み入ったものになっていて、以外、という印象。
本来のコラッツ数列で同類の問題を考えた時にどうなるかは、少なくとも自分には未解決です

あと実は >>26>>79 の解からも着想を得てできた問題で、要は
「動き方が制限されている関数(実→実関数の連続性、整数→整数関数のリプシッツ連続性、等)に
非有理的な”漸化式”(f(x),f(x+1),f(x+√2)間、S(n),S(2n),S(3n)間、等)を設けた時の挙動」
を問う問題を他にも作ってみたい、という感じでできたものでした まあ解法は若干違うものになったんだけど…

>>26 の想定解は >>180 と同じぐらいかそれ以下の分量なんだけど、
他の方法も見てみたいし、もしお時間あれば概略だけでも是非書いてみてくださいな
0184132人目の素数さん
垢版 |
2018/11/11(日) 14:33:02.52ID:/oKi5paQ
>>79の解答。

正の実数xに対して S(x)=Σ(1≦k≦x) a(2k−1) と置く。
ただし、0<x<1のときは S(x)=0 と定義する。

η(x)=S(3x)−S(2x)+S(x) (x>0)

と置くと、>>149と同じような計算をして、η(x)は有界であることが示せる。
α=limsup_(x→∞)S(x)/x, β=liminf_(x→∞)S(x)/x と置くと、
−1≦β≦α≦1 である。α=β=0 を示したい。
S(3x)=S(2x)−S(x)+η(x) をxで割ってlimsup_xを取ると、
3α≦2α−β となるので、α+β≦0 となる。また、liminf_xを取ると
3β≧2β−α となるので、α+β≧0 となる。よって、α+β=0 となる。
β≦αにより0=α+β≦2αとなるので、0≦αとなる。
0185132人目の素数さん
垢版 |
2018/11/11(日) 14:36:34.89ID:/oKi5paQ
次に、S(3x)=S(2x)−S(x)+η(x) において、x>0をx/3>0で置き換えると
S(x)=S((2/3)x)−S(x/3)+η(x/3) となるので、n≧1とx>0に対して、帰納的に

S(x)=S((2/3)^n x)−Σ(k=0〜n−1)S((2/3)^k x/3)+Σ(k=0〜n−1)η((2/3)^k x/3)

が成り立つ。特にx>1のとき、n_x=[log_{3/2}x]+1と置けば、
0 < (2/3)^{n_x} x < 1 となるので、S((2/3)^{n_x} x)=0 であり、

S(x)=−Σ(k=0〜n_x−1)S((2/3)^k x/3)+Σ(k=0〜n_x−1)η((2/3)^k x/3)

となる。この式では、Σの部分は n_x−1 までの和なので、
x に依存して和の項数が増えることに注意。
0186132人目の素数さん
垢版 |
2018/11/11(日) 14:40:11.70ID:/oKi5paQ
ηが有界であることから、x のオーダーとして

Σ(k=0〜n_x−1)η((2/3)^k x/3)=O(n_x)=O([log_{3/2}x]+1)=O(log x) (x→∞)

である。よって、

S(x)=−Σ(k=0〜n_x−1)S((2/3)^k x/3)+O(log x)

である。次に、s(x)=S(x)/x (x>0) と置く。−1≦s(x)≦1である。上の式をxで割って

s(x)=−Σ(k=0〜n_x−1) (2/3)^k (1/3) s((2/3)^k x/3)+o(1)

となる(o(1)の部分は、こだわるならO((log x)/x)と書いた方が精密だが、ここではo(1)で十分)。
ここで、δ>1を任意に取って固定する。Σの部分を

Σ(k=0〜n_x−1) = Σ(k:(2/3)^k x/3<δ)+Σ(k:(2/3)^k x/3≧δ)

と分解する。
0187132人目の素数さん
垢版 |
2018/11/11(日) 14:42:57.29ID:/oKi5paQ
Σ(k:(2/3)^k x/3<δ) (2/3)^k (1/3) s((2/3)^k x/3)
≧Σ(k:(2/3)^k x/3<δ) (2/3)^k (1/3)(−1)
=Σ(log_{3/2}(x/(3δ))<k≦n_x−1) (2/3)^k (1/3)(−1)
≧Σ([log_{3/2}(x/(3δ))]<k) (2/3)^k (1/3)(−1)
=(−1/3)(2/3)^{ 1+[log_{3/2}(x/(3δ))] } * 1/(1−2/3)
=o(1)

である。
0188132人目の素数さん
垢版 |
2018/11/11(日) 14:45:18.55ID:/oKi5paQ
また、

Σ(k:(2/3)^k x/3≧δ) (2/3)^k (1/3) s((2/3)^k x/3)
≧Σ(k:(2/3)^k x/3≧δ) (2/3)^k (1/3) inf(t≧δ)s(t)
=Σ(1≦k≦log_{3/2}(x/(3δ))) (2/3)^k (1/3) inf(t≧δ)s(t)
=Σ(1≦k≦[log_{3/2}(x/(3δ))]) (2/3)^k (1/3) inf(t≧δ)s(t)
=(1/3)(2/3)(1−(2/3)^{ [log_{3/2}(x/(3δ))] })/(1−2/3) * inf(t≧δ)s(t)
=(1/3)(2/3)(1−o(1))/(1−2/3) * inf(t≧δ)s(t)
=(2/3)(1-o(1)) * inf(t≧δ)s(t)
=(2/3)inf(t≧δ)s(t)−(2/3)o(1)inf(t≧δ)s(t)
=(2/3)inf(t≧δ)s(t)+o(1)

である。
0189132人目の素数さん
垢版 |
2018/11/11(日) 14:48:06.02ID:/oKi5paQ
これらの不等式を

s(x)=−Σ(k=0〜n_x−1) (2/3)^k (1/3) s((2/3)^k x/3)+o(1)

と合わせて、

s(x) ≦ o(1)−(2/3)inf(t≧δ)s(t)

となるので、limsup_x を取って α≦−(2/3)inf(t≧δ)s(t) となる。
δ>1は任意だから、δ→∞として、α≦−(2/3)β となる。
よって、3α+2β≦0 となるので、α+β=0によりα≦0となる。
一方でα≧0だったから、α=0となる。よって、β=0となる。
よって、lim(x→∞) S(x)/x=0 である。
0190132人目の素数さん
垢版 |
2018/11/11(日) 14:51:36.06ID:/oKi5paQ
次に、正の実数xに対して T(x)=Σ_(1≦k≦x)a_k と置く。
ただし、0<x<1 のときは T(x)=0 と定義する。
ν(x)=T(3x)+T(x)−2S(x) (x>0) と置くと、ν(x) は有界であることが示せる。
α=limsup_(x→∞) T(x)/x, β=liminf_(x→∞) T(x)/x と置く。
−1≦β≦α≦1である。T(3x)=−T(x)+2S(x)+ν(x) をxで割って
limsup_x を取れば、3α=−β となる。また、liminf_x を取れば、3β=−α となる。
よって、α=β=0となるので、lim_(x→∞) T(x)/x=0 となる。
よって、lim_(n→∞) (1/n)Σ_(k=1,n)a_k=0 である。
0191132人目の素数さん
垢版 |
2018/11/11(日) 14:55:49.03ID:/oKi5paQ
次は素数定理のやり方。

1以上の実数xに対して S(x)=Σ(1≦k≦x) a(2k−1) と置く。
η(x)=S(3x)−S(2x)+S(x) (x≧1) と置くと、η(x)は有界であることが示せる。
次に、Re(z)>1を満たす複素数zに対して

f(z)=∫(1,∞) S(x)/x^{1+z}dx

と置くと、f(z)はRe(z)>1の範囲の正則関数である。変数変換で

f(z)=∫(1/3,∞) S(3x)/(3x)^{1+z}3dx

としてから S(3x)=S(2x)−S(x)+η(x) を使って変形すれば、
面倒くさいので詳細は省略するが、ある具体的なg(z)に対して

(1−(2^z−1)/3^z)f(z)=g(z)

という形になって、しかもg(z)はRe(z)>0の範囲の正則関数になることが示せる。
0192132人目の素数さん
垢版 |
2018/11/11(日) 14:59:42.91ID:/oKi5paQ
次に、Re(z)≧1のとき (1−(2^z−1)/3^z)≠0 となることが示せる(自明ではないが)ので、

h(z)=g(z)/(1−(2^z−1)/3^z)

が Re(z)≧1 の範囲で定義できて、Re(z)>1の範囲ではh(z)は正則である。

また、Re(z)=1上の各点zに対して、(1−(2^z−1)/3^z)≠0 であるから、
zごとに、zを含む十分小さな開円盤B(円の半径はzに依存する)が存在して、
各点 w∈B で (1−(2^w−1)/3^w)≠0 である。
よって、B上でも h(w)=g(w)/(1−(2^w−1)/3^w) が定義できて、B上でh(w)は正則である。
よって、Re(z)≧1という範囲を包含するある連結開集合Uが存在して、
z∈U のとき h(z)=g(z)/(1−(2^z−1)/3^z) が定義できて、
hはU上の正則関数である。また、Re(z)>1のときは f(z)=h(z) である。
よって、fはU上の正則関数に解析接続される。
0193132人目の素数さん
垢版 |
2018/11/11(日) 15:02:30.04ID:/oKi5paQ
s(x)=S(x)/x (x≧1)と置けば、sは有界であり、変数変換により

f(z)=∫(0,∞) s(e^x)e^{−(z−1)x}dx (Re(z)>1)

となるので、fがU上の正則関数に解析接続されることから、
>>177のpdfの analytic theorem により、∫(0,∞) s(e^x)dx が存在する。
変数変換して、∫(1,∞) S(t)/t^2dt が存在する。
0194132人目の素数さん
垢版 |
2018/11/11(日) 15:05:12.24ID:/oKi5paQ
ここで、>>177のpdfの(VI)と似たような計算をする。λ>1を満たす実数λを任意に取る。
∫(1,∞) S(t)/t^2dt が存在することから、lim_(x→∞)∫(x,λx)S(t)/t^2dt=0 である。
また、1≦x≦t≦λxのとき|S(x)−S(t)|≦[t]−[x] なので、

∫(x,λx)S(t)/t^2dt≧∫(x,λx)(S(x)+[x]−[t])/t^2dt
=(S(x)+[x])(1/x)(1−1/λ)−∫(x,λx)[t]/t^2dt
=(S(x)+[x])(1/x)(1−1/λ)−∫(x,λx)(t−{t})/t^2dt
=(S(x)+[x])(1/x)(1−1/λ)−logλ+∫(x,λx){t}/t^2dt
≧(S(x)+[x])(1/x)(1−1/λ)−logλ

となる。limsup_(x→∞)として、0≧(limsup_x S(x)/x+1)(1−1/λ)−logλ
となるので、limsup_x S(x)/x ≦ (logλ)/(1−1/λ)−1 となる。
λ>1は任意だから、λ↓1として、limsup_x S(x)/x ≦ 1−1=0 となる。
0195132人目の素数さん
垢版 |
2018/11/11(日) 15:07:14.14ID:/oKi5paQ
次に、N(x)=−S(x) と置くと、∫(1,∞) N(t)/t^2dt が存在して、
1≦x≦yのとき|N(y)−N(x)|≦[y]−[x]である。よって、
>>194の計算をN(x)に置き換えても成立して、limsup_x N(x)/x≦0 となる。
よって、liminf_x S(x)/x≧0 となる。よって、

0≦liminf_x S(x)/x≦limsup_x S(x)/x≦0

となったので、lim_(x→∞) S(x)/x=0 である。
あとは>>190と同じようにして、lim_(n→∞) (1/n)Σ_(k=1,n)a_k=0 が示せる。
0196132人目の素数さん
垢版 |
2018/11/11(日) 16:01:29.89ID:oRKvGZPH
η有界なの?
計算機で実験したら極めてゆっくりではあるけど(nが6桁くらいで3桁くらいになる)なんかlogオーダーぐらいで揺れてそうだったけど。
多分揺れても高々logオーダーなのでxで割った時点で大丈夫なんだけど。
実験するとηもだけどS本体もlogオーダーの何乗かでは抑えられてそうなんだけどなぁ。
0197132人目の素数さん
垢版 |
2018/11/11(日) 16:05:08.09ID:oRKvGZPH
いや、ごめん。間違えました。ηは計算してない。
実験ではS本体がlog程度しか予想できなかった。
0198132人目の素数さん
垢版 |
2018/11/11(日) 16:32:13.28ID:/oKi5paQ
ηが有界の証明書いた方がよかったかな。

0<x<1の範囲ではη(x)は有界。x≧1のときは、|η(x)|≦3が成り立つことを示す。
まず、n≦x<n+1を満たす正整数nが取れるので、S(x)=S(n)となる。
また、2n≦2x<2n+2なので、2n+r≦2x<2n+r+1を満たすr=0,1が取れて、
S(2x)=S(2n+r)となる。また、3n≦3x<3n+3なので、3n+r'≦3x<3n+r'+1
を満たすr'=0,1,2が取れて、S(3x)=S(3n+r')となる。

S(2n+r)=S(2n)+Σ(k=2n+1〜2n+r) a_{2k−1}
S(3n+r')=S(3n)+Σ(k=3n+1〜3n+r') a_{2k−1}

なので、

η(x)=S(3x)−S(2x)+S(x)
=S(3n)−S(2n)+S(n)+Σ(k=3n+1〜3n+r') a_{2k−1}−Σ(k=2n+1〜2n+r) a_{2k−1}
=0+Σ(k=3n+1〜3n+r') a_{2k−1}−Σ(k=2n+1〜2n+r) a_{2k−1}

よって|η(x)|≦r+r'≦1+2=3
0199132人目の素数さん
垢版 |
2018/11/11(日) 16:34:10.74ID:FVh8W5vn
前者の手法だけど、
>>189
k=0 の項を足し忘れてないかい?修正して計算し直したら
s(x) ≦ o(1) - inf(t≧δ)s(t)
になって、limsup_x とってから δ を ∞ に飛ばしても
α ≦ - β
にしかならなくて、ここから結論を同じように導くのは無理な気がする…

後者の素数定理のやり方はあってるっぽいので正解にします
analytic theorem については存じ上げなかったんだけどまあこんなうまいこと成り立ってくれるんだねえ
|S(x)-S(t)|≦|[x]-[t]| の >>194 での使われ方がうまいと思いました ほんとお疲れ様でした

>>196
有界になるはずだよ
>>149 から x>0 が整数の時は μ(x)=0 になるし、
x が 1 以下動いた時の S(x),S(2x),S(3x) の値の変化はそれぞれ 1,2,3 以下だから μ(x) の変化も6以下になる
0201132人目の素数さん
垢版 |
2018/11/11(日) 16:43:02.44ID:FVh8W5vn
およ ログ進んでた すまんち
>>196 >>197
その様子を見ると、S(n) の最良のオーダーはだいたい n^(1/2 + ε) ぐらいにはなるのかねえ
まあ自分は証明できそうもないけど
0202132人目の素数さん
垢版 |
2018/11/11(日) 16:51:08.50ID:/oKi5paQ
剰余項つき素数定理の証明と同じようにすると、
(1/n)Σ_(k=1,n)a_k が 0 に収束する具体的なオーダーが
求められるかもしれない。最良のオーダーとして求まるわけではないし、
あまり詳しくないので何とも言えないが。

もし任意のε>0に対して n^(1/2 + ε) のオーダーで抑えられるなら、
リーマン予想のS(x)バージョンになってるので、とても面白いw
0203132人目の素数さん
垢版 |
2018/11/11(日) 17:58:42.98ID:oRKvGZPH
>>196
ですね。S(x)=S([x])で整数値のとこで0なんだからほぼ自明。
それにしても

>1以上の実数xに対して S(x)=Σ(1≦k≦x) a(2k−1) と置く。


コレがうまい。
言われてみれば当たり前なんだけど。
私も池原の定理の類使うのは第一勘だったんだけど

f(s)=Σa[n]n^(-s)

を考えて失敗してこの方針捨てちゃったんだよね〜
頭硬いorz
0204132人目の素数さん
垢版 |
2018/11/11(日) 23:22:48.02ID:FVh8W5vn
>>26 ももう答え書いていいかな

M = sup_x |f(x)| とおく。
実数 x と整数 n に対して y_n = x - n*√2 と定めると、
f(x+1) - f(x) = Σ_(k=0,2n) 2^(-2n) * (2n)C(k) * ( f(y_(2n)+1+k) - f(y_(2n)+k) )
= 2^(-2n) * Σ_(k=0,2n+1) ( (2n)C(k-1) - (2n)C(k) ) * f(y_(2n)+k)
となるから、
|f(x+1)-f(x)| ≦ 2^(-2n) * Σ_(k=0,2n+1) | (2n)C(k-1) - (2n)C(k) | * M
= M * 2^(-2n) * 2 * Σ_(k=0,n) (2n)C(k) - (2n)C(k-1)
= M * 2^(-2n) * 4 * (2n)C(n).
n は任意のであったから、n→∞ として f(x)=f(x+1) を得る。
これを元の式に代入することで f(x)=f(x+√2) も得られるが、
以上から f は稠密な集合 {a+b√2 | a,bは整数} 上で一定であるから、f は定数関数である。
0206132人目の素数さん
垢版 |
2018/11/11(日) 23:34:22.25ID:sLf3laj9
>>172
 a_{n+1} = 2a_n + a_{n-1},   (a_2 = 2)
ならば
a_[n+2} = 6a_n - a_{n-2},   (a_1 + a_3 = 6)
 a_{n+3} = 14a_n + a_{n-3},   (a_2 + a_4 = 14)
a_{n+4} = 34a_n - a_{n-4},   (a_3 + a_5 = 34)
a_{n+5} = 82a_n + a_{n-5},   (a_4 + a_6 = 82)

(略証)
 a_{n+1} -2a_n - a_{n-1} = d_n,
とおくと
a_{n+2} -6a_n + a_{n-2} = d_{n+1} +2d_n - d_{n-1},
 a_{n+3} -14a_n - a_{n-3} = d_{n+2} +2d_{n+1} +5d_n -2d_{n-1} +d_{n-2},
a_{n+4} -34a_n + a_{n-4} = d_{n+3} +2d_{n+2} +5d_{n+1} +12d_n -5d_{n-1} +2d_{n-2} -d_{n-3},
a_{n+5} -82a_n - a_{n-5} = d_{n+4} +2d_{n+3} +5d_{n+2} +12d_{n+1} +29d_n -12d_{n-1} +5d_{n-2} -2d_{n-3} +d_{n-4},
0207132人目の素数さん
垢版 |
2018/11/11(日) 23:40:24.39ID:/oKi5paQ
>>204
そんなに簡単に終わるのかw

こっちは自分の証明の簡略化を考えてみたが、1行も短くならない (^o^)
しかも、>>177のpdfの手法を使った別証明も見つかったw
まさか>>26でも>>177が使えるとは思わんかった。
0209132人目の素数さん
垢版 |
2018/11/12(月) 00:08:20.31ID:nrLLMPLB
>>208
いま、長い方の証明から書き起こしてます(例のごとく、計算ミスしてるかもしれん)。
>>177の方針はそのあとになるんで、ちょっと時間かかりますw
0210132人目の素数さん
垢版 |
2018/11/12(月) 00:15:17.48ID:Q11gTSDz
>>207 いや普通に気になる よければ概要だけでも聞きたいわ

やや余談 >>201 の予想だけど、だんだん成り立たない気がしてきた
仮に任意の ε>0 について |S(x)|=O(x^(α+ε)) が成り立つと仮定すると、
Re(z)>α の範囲でf(z)が絶対収束するから >>191 の等式が成り立つことになるんだけど、
z が 3^z-2^z+1=0 を満たす場合、g(z)=0 も成り立たなければならなくなる。
例えば z=0.603312...+47.8074...i とか z=0.734188...+169.407...i が 3^z-2^z+1 の根になるらしいんだけど、
仮に α=1/2 ととれるなら、これらが全て g の根にもなる必要がある。
こんなうまいこと成り立ってくれるとはあまり思えない…
0211132人目の素数さん
垢版 |
2018/11/12(月) 08:44:17.10ID:TKDy5P8X
>>204

スターリングの公式で
 2^(-2n)・C(2n,n) = 2^(-2n)・(2n)!/(n!)^2 〜 1/√(πn),   (n→∞)
かしらん…
0212132人目の素数さん
垢版 |
2018/11/12(月) 17:14:52.91ID:Q11gTSDz
>>109 ももう答え出しちゃおうか

ペル方程式 p^2 - 2q^2 = -1 には解が無数に存在するので、この解を用いて
x=q(p^2+9p+19),
y=p^2+6p+4
と定めれば
2x^2 - y^3 = 27(2p+11)
となり、p が十分大きければ 0<|2x^2-y^3|<100√|y| が成り立つ。
0215132人目の素数さん
垢版 |
2018/11/12(月) 18:20:45.73ID:Q11gTSDz
>>213
この問題考える前にまず |x^2-y^3| を小さくできないかって考えてて、
x,y をnをパラメータとした多項式で表して良いのが作れるかなって考えたんだけど、
いつぞやのメーソン・ストーサーズの定理から、多項式 f,g について
deg(f^2-g^3) > (1/2)deg(g)
が成り立つことがわかるため、これは断念。
でも、例えば無限個のnについてh(n)が平方数になるようなあるhについてなら
deg(hf^2-g^3) ≦ (1/2)deg(g)
が成り立ってくれるのでは?と思って、状況を簡単にするために
(mx^2+1)(mx^2+ax+b)^2 - (mx^2+cx+d)^3
が x についての一次以下の自明でない式になるように
整数係数 m,a,b,c,d についての方程式を立てて解いていった、というのが見つかったきっかけ。
ただこの場合、最後に残った m,a についての方程式が確か
81m=a^2
だったから、どう頑張っても m が平方数にしかならなくて、h(n)が無限回平方数になるという目的は断念。
副産物として m=1 として a,b,c,d を定めていってできたのが >>212 で使われた
(x^2+1)(x^2+9x+19)^2 - (x^2+6x+4)^3 = 27(2x+11)
という式。これでも h にあたる x^2+1 が無限回(平方数×2)になってくれるからまあいいか、と。
ちなみに
8(2y^2-x^3) = (4y)^2-(2x)^3
でもあるから、Y^2-X^3 < 800√|X| を満たす(X,Y)も無限に存在する事になって、
無事最初の目的も果たされることになったとかそんな感じです
0216132人目の素数さん
垢版 |
2018/11/12(月) 18:37:16.39ID:Q11gTSDz
ただまあ後で調べてみたら Hall's conjecture というのがあるらしくて
https://en.wikipedia.org/wiki/Hall%27s_conjecture
この問題を考える過程で Danilov さんが既に同等のことを証明していたらしいことがわかって、やや萎え(?)
(式は自分のの方がより簡単になってるから意味無くはないと信じたいけどまあその辺はどうでも)
おそらくこれがその論文↓
http://www.mathnet.ru/php/archive.phtml?wshow=paper&;jrnid=mzm&paperid=6024&option_lang=eng
ロシア語で書かれてるけど実質2ページしかないからグーグル翻訳にちょっとずつ入れてって何とかなるレベルかと
英語版もあるらしいけど有料みたいなのでまあいいやと
0217132人目の素数さん
垢版 |
2018/11/12(月) 19:55:19.13ID:nrLLMPLB
素数定理の方針を使った>>26の証明を書きます。
示したいのは次の定理(I)で、>>26はこの定理の特殊な場合になります。

定理(I) m≧2とする。a_1,…,a_m は正の実数で、ある異なるa_uとa_vがQ上一次独立とする。
λ_1,…,λ_mは正の実数で、Σ(k=1〜m)λ_k=1を満たすとする。
f:R→Rは連続かつ有界で、f(x)=Σ(k=1〜m)λ_k f(x−a_k) (x∈R) が成り立つとする。
このとき、fは定数関数である。
0218132人目の素数さん
垢版 |
2018/11/12(月) 19:58:30.09ID:nrLLMPLB
補題1 m≧2とする。a_1,…,a_m は正の実数とする。
λ_1,…,λ_mは正の実数で、Σ(k=1〜m)λ_k=1を満たすとする。
写像 f:R→R は f(x)=Σ(k=1〜m)λ_k f(x−a_k) (x∈R) を満たすとする。
このとき、limsup(x→−∞)f(x)=sup(x∈R)f(x), liminf(x→−∞)f(x)=inf(x∈R)f(x) である。
また、fが局所ルベーグ可積分なら、Σ(k=1〜m)λ_k ∫(x−a_k,x)f は
xに依存しない定数である。よって、xに依存しない定数 α∈R を

α = (Σ(k=1〜m)λ_ka_k)^{−1} Σ(k=1〜m)λ_k ∫(x−a_k,x)f

として定義できるが、実は
α≦limsup(x→−∞)f(x), α≦limsup(x→+∞)f(x),
α≧liminf(x→−∞)f(x), α≧liminf(x→+∞)f(x) が成り立つ。
0219132人目の素数さん
垢版 |
2018/11/12(月) 20:02:26.03ID:nrLLMPLB
証明 帰納法により、x∈Rとn≧1に対して

f(x)=Σ(k_1,…,k_n∈[1,m]) λ_{k_1}…λ_{k_n}f(x−a_{k_1}−a_{k_2}−…−a_{k_n})

が成り立つ。c=limsup(x→−∞)f(x) と置く。sup(x∈R)f(x)≦c を示す。
c=+∞のときは明らか。c<+∞のときは、c<rを満たす実数rを任意に取る。
limsup(x→−∞)f(x)=c<r だから、あるδが存在して、t<δのとき f(t)<r が成り立つ。
a=min{a_1,…,a_m}>0 と置く。x∈Rを任意に取る。xに依存した十分大きなn≧1を取れば、
x−na<δ となるので、k_1,…,k_n∈[1,m] に対して
x−a_{k_1}−a_{k_2}−…−a_{k_n}≦x−a−a−…−a=x−na<δ である。よって、

f(x)=Σ(k_1,…,k_n∈[1,m]) λ_{k_1}…λ_{k_n}f(x−a_{k_1}−a_{k_2}−…−a_{k_n})
≦Σ(k_1,…,k_n∈[1,m]) λ_{k_1}…λ_{k_n} r = (Σ(k=1〜m)λ_k)^n * r = r

となる。
0220132人目の素数さん
垢版 |
2018/11/12(月) 20:04:45.72ID:nrLLMPLB
証明の続き x∈R は任意だったから、sup(x∈R)f(x)≦r となる。
c<rは任意だったから、r↓cとして、sup(x∈R)f(x)≦c となる。
次に、sup(x∈R)f(x)≧c を示したいが、これは明らか。
よって、sup(x∈R)f(x)=c すなわち sup(x∈R)f(x)=limsup(x→−∞)f(x) が成り立つ。
fを(-f)で置き換えれば、(-f)自体が補題1の仮定を満たすので、同じ議論によって
sup(x∈R)(−f)(x)=limsup(x→−∞)(−f)(x) が成り立つ。
よって inf(x∈R)f(x)=liminf(x→−∞)f(x) が成り立つ。
0221132人目の素数さん
垢版 |
2018/11/12(月) 20:07:19.81ID:nrLLMPLB
証明の続き 次に、fは局所ルベーグ可積分とする。x≦yのとき

∫(x,y)f(t)dt=∫(x,y)Σ(k=1〜m)λ_k f(t−a_k)dt
=Σ(k=1〜m)λ_k ∫(x,y)f(t−a_k)dt
=Σ(k=1〜m)λ_k ∫(x−a_k,y−a_k)f(t)dt
=Σ(k=1〜m)λ_k (∫(x−a_k,x)+∫(x,y)+∫(y,y−a_k))f(t)dt
=∫(x,y)f(t)dt+Σ(k=1〜m)λ_k (∫(x−a_k,x)+∫(y,y−a_k))f(t)dt

なので、Σ(k=1〜m)λ_k ∫(x−a_k,x)f =Σ(k=1〜m)λ_k ∫(y−a_k,y)f
となる。よって、Σ(k=1〜m)λ_k ∫(x−a_k,x)f はxに依存しない定数である。
0222132人目の素数さん
垢版 |
2018/11/12(月) 20:11:12.83ID:nrLLMPLB
証明の続き 次に、α = (Σ(k=1〜m)λ_ka_k)^{−1} Σ(k=1〜m)λ_k ∫(x−a_k,x)f
と置く。α≦limsup(x→+∞)f(x) かつ α≦limsup(x→−∞)f(x) を示す。
どちらもほぼ同じ議論なので、前者のみ示す。b=max{a_1,…,a_m}と置いておく。
c=limsup(x→+∞)f(x)と置く。α≦cを示せばよい。c=+∞なら、明らか。
c<+∞のときは、c<rを満たす実数rを任意に取る。
limsup(x→+∞)f(x)=c<rより、あるδが存在して、x≧δのときf(x)<rが成り立つ。
δ+b>δ+b−a_k≧δ (1≦k≦m) であるから、

α=(Σ(k=1〜m)λ_ka_k)^{−1} Σ(k=1〜m)λ_k ∫(δ+b−a_k,δ+b)f ≦ r

となる。すなわち、α≦rとなる。c<rは任意だったから、r↓cとして、確かにα≦cとなる。
次に、α≧liminf(x→+∞)f(x) かつ α≧liminf(x→−∞)f(x) を示す。
補題1のfとαに対して(-f)と(−α)を考えれば、補題1の条件が成り立つので、
同じ議論ができて−α≦limsup(x→+∞)(−f)(x) かつ −α≦limsup(x→−∞)(−f)(x)
となる。よって、α≧liminf(x→+∞)f(x), α≧liminf(x→−∞)f(x) である。□
0223132人目の素数さん
垢版 |
2018/11/12(月) 20:17:32.43ID:nrLLMPLB
定理1 m≧2とする。a_1,…,a_m は正の実数で、ある異なるa_uとa_vがQ上一次独立とする。
λ_1,…,λ_mは正の実数で、Σ(k=1〜m)λ_k=1を満たすとする。f:R→RはR上でリプシッツ連続で、
f(x)=Σ(k=1〜m)λ_k f(x−a_k) (x∈R) が成り立つとする。このとき、fは定数関数である。

証明 あるL≧0が存在して、任意のx,y∈Rに対して|f(x)−f(y)|≦L|x−y|である。
特にfは連続なので、局所ルベーグ可積分である。よって、補題1により、
limsup(x→−∞)f(x)=sup(x∈R)f(x) かつ liminf(x→−∞)f(x)=inf(x∈R)f(x) であり、
xに依存しない定数αが

α=(Σ(k=1〜m)λ_ka_k)^{−1} Σ(k=1〜m)λ_k ∫(x−a_k,x)f

として定義できる。実は sup(x∈R)f(x)<+∞ が成り立つ。これを背理法で示す。
0224132人目の素数さん
垢版 |
2018/11/12(月) 20:21:03.19ID:nrLLMPLB
証明の続き もしsup(x∈R)f(x)=+∞ならば、任意のN≧1に対して、
あるt∈Rが存在して、N<f(t)が成り立つ。y_1=t と置くと、N<f(y_1)である。
また、f(y_1)=Σ(k=1〜m)λ_k f(y_1−a_k) である。
よって、あるkに対して N<f(y_1−a_k) である。そこで、y_2=y_1−a_k と置く、
これを帰納的に繰り返すと、点列 {y_i}_i が定義できて、
N<f(y_i), y_1=t, y_i−y_{i+1}∈{a_1,…,a_m} となる。
特に、y_iは狭義単調減少で、y_i→−∞ となる。よって、x∈(−∞,y_1]を任意に取ると、
y_{i+1}<x≦y_iを満たすiが取れる。b=max{a_1,…,a_m}と置けば

|f(x)−f(y_i)|≦L|x−y_i|≦L|y_{i+1}−y_i|≦Lb

なので、f(x)≧f(y_i)−Lb≧N−Lbとなる。これがx∈(−∞,y_1]のとき成り立つので、

α=(Σ(k=1〜m)λ_ka_k)^{−1} Σ(k=1〜m)λ_k ∫(y_1−a_k,y_1)f ≧ N−Lb

となる。すなわち、α≧N−Lb となる。N≧1は任意だったから、α=+∞となって矛盾する。
よって、sup(x∈R)f(x)<+∞である。
0225132人目の素数さん
垢版 |
2018/11/12(月) 20:28:30.09ID:nrLLMPLB
証明の続き fを(−f)で置き換えれば、(−f)自体が定理1の条件を満たすので、
同じ議論によって sup(x∈R)(−f)(x)<+∞ である。
よって、inf(x∈R)f(x)>−∞である。よって、fは有界である。
次に、g=f−α と置く。このとき、gは有界である。
また、|g(x)−g(y)|≦L|x−y| (x,y∈R) である。
また、g=f−α とαの定義により

Σ(k=1〜m)λ_k ∫(x−a_k,x)g = 0 (x∈R)

である。特に Σ(k=1〜m)λ_k ∫(−a_k,0)g = 0 である。
また、g(x)=Σ(k=1〜m)λ_kg(x−a_k) (x∈R) が成り立つ。
xを−xで置き換えて、g(−x)=Σ(k=1〜m)λ_kg(−x−a_k) (x∈R) も成り立つ。
ここで、Re(z)>0 を満たす複素数zに対して

G(z)=∫(0,∞)g(−x)e^{−zx}dx

が定義できて、G(z)はRe(z)>0の範囲で正則である。
0226132人目の素数さん
垢版 |
2018/11/12(月) 20:30:52.77ID:nrLLMPLB
証明の続き

G(z)=∫(0,∞)g(−x)e^{−zx}dx=∫(0,∞)e^{−zx}Σ(k=1〜m)λ_kg(−x−a_k)dx
=Σ(k=1〜m)λ_k ∫(0,∞)e^{−zx}g(−(x+a_k))dx
=Σ(k=1〜m)λ_k ∫(a_k,∞)e^{−z(x−a_k)}g(−x)dx
=Σ(k=1〜m)λ_k ∫(a_k,0)e^{−z(x−a_k)}g(−x)dx+G(z)Σ(k=1〜m)λ_ke^{a_kz}

であるから、

(1−Σ(k=1〜m)λ_ke^{a_kz})G(z)=Σ(k=1〜m)λ_k∫(a_k,0)e^{−z(x−a_k)}g(−x)dx

となる。右辺をH(z)と置き、u(z)=(1−Σ(k=1〜m)λ_ke^{a_kz}) と置けば、
H(z)とu(z)はC全体で定義可能な正則関数であり、Re(z)>0 のとき u(z)G(z)=H(z) が成り立つ。
0227132人目の素数さん
垢版 |
2018/11/12(月) 20:36:53.57ID:nrLLMPLB
証明の続き 

H(0)=Σ(k=1〜m)λ_k ∫(a_k,0)g(−x)dx=−Σ(k=1〜m)λ_k ∫(−a_k,0)g(x)dx=0

である。また、u(0)=0 だが、lim(z→0)u(z)/z=u'(0)=−Σ(k=1〜m)λ_ka_k ≠ 0 である。
よって、z=0を含むある開円盤 B_0 とある正則関数 H_1:B_0→C とある正則関数
u_1:B_0→C−{0} が存在して、w∈B_0のとき H(w)=wH_1(w), u(w)=wu_1(w) と表せる。
次に、Re(z)=0 かつ z≠0 のとき u(z)≠0 が成り立つことが示せる
(自明ではないが、省略する)。よって、Re(z)=0 かつ z≠0 のとき、
その z を含むある開円盤 B_z の上で u(w)≠0 である。
0228132人目の素数さん
垢版 |
2018/11/12(月) 20:39:28.98ID:nrLLMPLB
証明の続き 

U={z∈C|Re(z)>0}∪(∪_{Re(z)=0}B_z)

と置けば、Uは連結開集合である。G_1:U→C を

G_1(w)=G(w) (Re(w)>0)
G_1(w)=H(w)/u(w) (w∈∪_{Re(z)=0, z≠0}B_z)
G_1(w)=H_1(w)/u_1(w) (w∈B_0)

と定義すると、G_1 は well-defined である(自明ではないが、省略する)。
また、G_1はU上の正則関数である。また、Re(z)>0 のときは G_1(z)=G(z) である。
よって、G(z)はU上に解析接続される。
0229132人目の素数さん
垢版 |
2018/11/12(月) 20:42:28.44ID:nrLLMPLB
証明の続き まとめると、gは有界であり、Re(z)>0 のとき
G(z)=∫(0,∞)g(−x)e^{−zx}dx であり、GはU上に解析接続されるので、
analytic theorem により、∫(0,∞)g(−y)dy が存在する。
すなわち、∫(−∞,0)g(y)dy が存在する。そこで、ε>0を任意に取る。
∫(−∞,0)g(y)dy が存在することから、lim(x→−∞)∫(x,x+ε)g(y)dy=0 である。
また、x≦y≦x+εのとき|g(x)−g(y)|≦L|x−y|=L(y−x)であるから、

∫(x,x+ε)g(y)dy≧∫(x,x+ε)(g(x)−L(y−x))dy
=εg(x)−L∫(x,x+ε)(y−x)dy=εg(x)−L∫(0,ε) y dy=εg(x)−Lε^2/2

となる。limsup(x→−∞) を取って、0≧εlimsup(x→−∞)g(x)−Lε^2/2 となるので、
limsup(x→−∞)g(x)≦Lε/2 となる。ε>0は任意だったから、
limsup(x→−∞)g(x)≦0 となる。
0230132人目の素数さん
垢版 |
2018/11/12(月) 20:44:54.14ID:nrLLMPLB
証明の続き 次に、gのかわりに(−g)を考えると、∫(−∞,0)(−g)(y)dy が存在し、
|(−g)(x)−(−g)(y)|≦L|x−y| (x,y∈R) であるから、同じ議論によって
limsup(x→−∞)(−g)(x)≦0 となる。すなわち、liminf(x→−∞)g(x)≧0 となる。
以上より、lim(x→−∞)g(x)=0 となる。g=f−αだったから、lim(x→−∞)f(x)=αとなる。
limsup(x→−∞)f(x)=sup(x∈R)f(x) かつ liminf(x→−∞)f(x)=inf(x∈R)f(x) だったから、

sup(x∈R)f(x)=limsup(x→−∞)f(x)=lim(x→−∞)f(x)=α,
inf(x∈R)f(x)=liminf(x→−∞)f(x)=lim(x→−∞)f(x)=α

である。任意のx∈Rに対して inf(x∈R)f(x)≦f(x)≦sup(x∈R)f(x) であるから、
α≦f(x)≦α となり、よって f(x)=α (x∈R) となり、fは定数関数である。□
0231132人目の素数さん
垢版 |
2018/11/12(月) 20:47:54.18ID:nrLLMPLB
定理(I)の証明 補題1から、α=(Σ(k=1〜m)λ_ka_k)^{−1}Σ(k=1〜m)λ_k ∫(x−a_k,x)f
がxに依存しない定数として定義できる。F(x)=∫(0,x)(f(t)−α)dt と置けば、

F(x)−Σ(k=1〜m)λ_kF(x−a_k)=Σ(k=1〜m)λ_k(F(x)−F(x−a_k))
=Σ(k=1〜m)λ_k ∫(x−a_k,x)(f(t)−α)dt
=Σ(k=1〜m)λ_k ∫(x−a_k,x)f(t)dt−αΣ(k=1〜m)λ_ka_k=0

となるので、F(x)=Σ(k=1〜m)λ_kF(x−a_k) (x∈R)となる。
また、L=sup(t∈R)|f(t)−α| と置けば、fが有界であることから L<+∞であり、
|F(y)−F(x)|=|∫(x,y)(f−α)|≦L|x−y| (x,y∈R) である。よって、
FはR上でリプシッツ連続である。よって、定理1から、Fは定数関数である。
Fは各点で微分可能なので、F'=0すなわち(f−α)=0である。
よって、fは定数関数である。□
0232132人目の素数さん
垢版 |
2018/11/12(月) 20:51:00.49ID:nrLLMPLB
やっと終わった('A`)

長い方の証明も途中までは同じで、素数定理の手法を使っている部分が
別のやり方に変わっていきます。需要があったら書きます。
途中まで同じなので、やや省略できて+10レスくらいになります。
0233132人目の素数さん
垢版 |
2018/11/12(月) 21:24:55.86ID:Q11gTSDz
一般の場合に証明しちまったのか…つよ…そりゃ長くもなるわ
書き起こすの大変だったろうに、気軽に頼んじゃってごめんよ お疲れ様ですほんとに

f を積分した関数 F の挙動について考えたのはうまいなあと思いました
f と同じ等式が成り立つし、単に連続だったのをリプシッツ連続まで強くできるんだもんなあ
(まあ積分した関数も有界であることの証明は必要になったけど何とかなってたし)
α は何か f にとって重要な意味を持つ値なんだろうね おおまかに f の"平均"ということになるのかしら
あとやっぱ analytic theorem つええなあ…

自分も一般的な場合の証明考えてみようかな…
個人的には、もう一つ見てみたいのとお腹いっぱいなのとの半々かなあ
もし既に書き起こしてるなら折角だし書いてみてもいいと思うけど、任せる!
0235132人目の素数さん
垢版 |
2018/11/12(月) 22:55:53.30ID:nrLLMPLB
ちなみに、もはや遊びですが次の定理も成り立ちます。

定理2 m≧2とする。a_1,…,a_m は正の実数で、ある異なるa_uとa_vがQ上一次独立とする。
λ_1,…,λ_mは正の実数で、Σ(k=1〜m)λ_k=1を満たすとする。
f:R→R は L^∞ 関数で、f(x)=Σ(k=1〜m)λ_k f(x−a_k) a.e.x∈R が成り立つとする。
このとき、fはa.e.で定数である。
0236132人目の素数さん
垢版 |
2018/11/13(火) 00:18:02.67ID:BEOLFZw6
>>234 >>235
ありがとう気楽にやってくれい てかそこまで一般化できるのか…
あと次みたいな拡張も考えられそうだ 例のごとく自分には証明できそうもないが

(予想)
実数上の有界連続関数全体からなるベクトル空間を V とおく。
V 上の線形作用素 S:V→V が次を全て満たすとする:
・任意の a∈R に対して定まる平行移動作用素 T=T_a:V→V ; (Tf)(x)=f(x+a) について、T と S は可換。
・f∈V について、f(x)≧0 for ∀x∈R ならば (Sf)(x)≧0 for ∀x∈R.
・f∈V が周期関数の時、(Sf=f)⇔(fは定数関数)
この時、f∈V が Sf=f を満たすならば f は定数関数である。□

作用素論とかに強い人いたら、もしよければ挑戦してみてくれ!!(他力本願)
0238132人目の素数さん
垢版 |
2018/11/13(火) 03:35:05.71ID:f/KMc4Mc
あ、失礼しました。
――-
there is a positive constant C such that for any integers x and y for which y2 ≠ x3,

|y^{2}-x^{3}|>C|x|^(1/2).
――
が Hall’ conjecture で
――
for any ε > 0, there is some constant c(ε) depending on ε such that for any integers x and y for which y2 ≠ x3,

|y^{2}-x^{3}|>C|x|^(1/2-ε).
――-
がその弱型ですね。
どうやって ABC予想から示すんだろう?
0239132人目の素数さん
垢版 |
2018/11/13(火) 05:00:13.35ID:BEOLFZw6
でけたわ 作用素に課される条件が若干強くなったけどまあこれで十分と思われ 証明はまた時間ある時に

定理3
実数上の有界連続関数全体からなるベクトル空間を V とおく。
V 上の線形作用素 S:V→V が次を全て満たすとする:
・任意の a∈R に対して定まる平行移動作用素 T=T_a:V→V ; (Tf)(x)=f(x+a) について、T と S は可換。
・f∈V について、f(x)≧0 for ∀x∈R ならば (Sf)(x)≧0 for ∀x∈R.
・f∈V が定数関数の時、Sf=f.
・任意の a>0 について次が成り立つ:「f∈V が f(x)>0 for ∀x∈R-(aZ) を満たせば (Sf)(0)>0 も満たす」
この時、Sf=f を満たす f∈V は定数関数のみである。□
0240132人目の素数さん
垢版 |
2018/11/13(火) 12:23:04.40ID:oFVtyBzR
固有の番号の書かれたカードが何枚あり、
その枚数は1000枚以下であることはわかっているが、その数を推定したい。
調査員が無作為に10枚選んで番号を記録して元に戻した。
別の調査員が無作為に20枚選んで番号を記録した。
二人の調査員の記録した番号を照合すると3人分の番号が一致していた。
この情報からカード枚数の期待値を求めよ。
0241132人目の素数さん
垢版 |
2018/11/13(火) 13:03:18.44ID:FBLhmoMV
>>240
>二人の調査員の記録した番号を照合すると3人分の番号が一致していた。

三枚一致した?
0243132人目の素数さん
垢版 |
2018/11/13(火) 14:44:52.93ID:FBLhmoMV
>>240
とりあえず”3枚一致した”と解釈して

C:3枚一致するという事象、N:カードの枚数、p[n] = P(N=n)として
E(N|C) = Σ n P(N=n|C) = Σ n P(N=n&C) / P(C) = Σ n p[n] P(C|N=n) / (Σ p[n] P(C|N=n))、
P(C) = Σp[n]P(C|N=n)、
P(C|N=n) = C[10,3]C[n-10,17]/C[n,20] (n≧27)
.      0 (otherwise)、
により
E(N|C) = Σn p[n] C[10,3]C[n-10,17]/C[n,20] / (Σ p[n] C[10,3]C[n-10,17]/C[n,20])

ここまでは定義どうり。
しかし問題文にp[n]が与えられてないし、p[n]に依存すると思われるのでここで詰まる。
一様分布だとしてHaskell先生に聞く。

Prelude Data.Ratio> let c m n = div (product [m-n+1..m]) (product [1..n])
Prelude Data.Ratio> let pc n = ((c 10 3)*(c (n-10) 17))%(c n 20)
Prelude Data.Ratio> (sum [(fromInteger n)*(pc n)|n<-[27..1000]])/(sum[pc n|n<-[27..1000]])
21375 % 143
Prelude Data.Ratio> fromRational it
149.47552447552448

キレイに求まるんかな?
0244132人目の素数さん
垢版 |
2018/11/13(火) 14:55:08.32ID:oFVtyBzR
>>243

正解です。

最頻値が20*10/3の66なのに期待値との乖離が大きくてびっくりした。
Rでも
[1] 149.4755244755245

95%信頼区間をhighest densitye intervalで算出したら32 〜 411

確率分布をグラフにしたらこんな感じ。

http://i.imgur.com/paJnhr9.png
0245132人目の素数さん
垢版 |
2018/11/13(火) 19:37:40.91ID:SF2cgR9a
>>239
ひえー!
予定していた長い方の証明は>>239には通用しないので、
書く意味がなくなりましたね(^o^)

>>235の証明だけ書いておきます。実は>>231とほぼ同じです。
0246132人目の素数さん
垢版 |
2018/11/13(火) 19:42:09.32ID:SF2cgR9a
>>235の証明 補題1の積分計算と同じ計算をすると、
α=(Σ(k=1〜m)λ_ka_k)^{−1}Σ(k=1〜m)λ_k∫(x−a_k,x)f
がxに依存しない定数として定義できる(a.e.x の意味ではなく、任意のxに依存しない)。
F(x)=∫(0,x)(f(t)−α)dt (x∈R) と置く。微積分学の基本定理から、
Fはa.e.で微分可能で F'=(f−α) a.e. である。
次に、αが任意のxに依存しないことから、>>231と同じ計算で
F(x)=Σ(k=1〜m)λ_kF(x−a_k) (x∈R) となる。
また、L=|α|+||f||_∞ と置けば、L<+∞であり、
|F(y)−F(x)|=|∫(x,y)(f−α)|≦L|x−y| (x,y∈R) である。よって、
FはR上でリプシッツ連続である。よって、定理1から、Fは定数関数である。
よって、FはR全体で微分可能で、R全体で F'=0 である。
一方で、F'=(f−α) a.e. だったから、(f−α)=0 a.e. となる。
よって、f=α a.e. である。□
0248132人目の素数さん
垢版 |
2018/11/14(水) 02:25:34.49ID:uakH23jG
〔補題〕
ペル方程式 pp - 2qq = -1 には解が無数に存在する。

(略証)
(p_1, q_1) = (1, 1) は一つの解である。

(p, q) が解ならば
 p ' + q'√2 = (1+√2)^2・(p+q√2),
 p ' - q'√2 = (1-√2)^2・(p-q√2),
(どちらでも同じこと)とおくと、
 p ' = 3p + 4q,
 q ' = 2p + 3q,
も解である。(漸化式)

これより解が無数に存在する。
 p_{2n+1} = {(1+√2)^(2n+1) + (1-√2)^(2n+1)}/2,
 q_{2n+1} = {(1+√2)^(2n+1) - (1-√2)^(2n+1)}/(2√2),

∴ p > (27*11-2C)/(C-27*2) をみたすpも無数に存在する。(C>27*2 のとき)
0249132人目の素数さん
垢版 |
2018/11/14(水) 04:52:59.01ID:A5DUEUEt
位数9852554225504584574の群を分類せよ
0250132人目の素数さん
垢版 |
2018/11/14(水) 05:55:42.54ID:XxUkRupx
>>249
9852554225504584574の素因数分解は2×83×59352736298220389である。
http://www.wolframalpha.com/input/?i=prime+factorization+9852554225504584574
p = 2、q = 83、r = 59352736298220389 とおき、それぞれの sylow group P,Q.R を選び、その共役の個数をl、m、n とおく。
このとき m = [G:N(Q)] = 1,p,r,pr、m ≡ 1 (mod q) である。
https://en.wikipedia.org/wiki/Sylow_theorems
しかしこれを満たすのは m=1 のみである。
特に Q は正規部分群である。
同様にして R も正規部分群である。
よってQR = Hは位数qrの部分群であり、同様にしてRはHの正規部分群である。
よってHはRのQによるInner semidirect productである。
https://en.wikipedia.org/wiki/Semidirect_product
ここでAut Rは位数 r-1 の巡回群であり、(r-1,q) = 1であるからQ→Aut Hは自明であるものしかない。
よってHはQとRの直積であり、位数qrの巡回群である。
同様にしてGはHのPによるInner semidirect productである。
ここでAut Hは位数q-1の巡回群と位数r-1の巡回群の直積であり、P→Aut Hは4つある。
以上によりGは
C[2]×C[83]×C[59352736298220389]、
D[83]×C[59352736298220389]、
C[83]×D[59352736298220389]、
<x,y,z|x^2、y^83、z^ 59352736298220389、xyxy、xzxz>
の4つのいずれかに同型である。
0251132人目の素数さん
垢版 |
2018/11/14(水) 21:17:34.40ID:uBKcGx1c
池の鯉を網で56匹すくいました。
すくった56匹に目印をつけ、池にもどしました。
次の日に鯉45匹をすくったところ、36匹に目印がついていました。
池の鯉はおよそ何匹ですか。
95%信頼区間も合わせて述べなさい。
0252132人目の素数さん
垢版 |
2018/11/14(水) 22:13:33.36ID:qZaeulAo
>>36 の1つ目が本当にわからん…
mod p での非自明な整数解がいつでも存在することは示せたから、
剰余を使った証明はどうやら無理っぽいということまでしかわからん…
0253132人目の素数さん
垢版 |
2018/11/15(木) 00:31:58.54ID:f0/aOcVm
次の等式が成立することを示せ。eはネイピア数である。
1/((1π)^2+1)+1/((2π)^2+1)+1/((3π)^2+1)+……=1/(e^2-1)
0254132人目の素数さん
垢版 |
2018/11/15(木) 00:38:45.17ID:0YjszKFG
>>252
忘れてた。
これムズイよね?
多分すべてのp進数体では解を持つけど大域的には解がないHasse principle の反例になるやつだと思うんだけど、じゃどうせいと言われると手が止まるよね?
0255132人目の素数さん
垢版 |
2018/11/15(木) 02:26:40.72ID:0h25dXYt
>>239 の定理3だけど、証明に致命的な間違いが見つかってやり直したが、それでもどう頑張っても修正無理そうだ…
代わりに、 S の条件をさらに強めたバージョンに変更して(主張がだいぶしょぼくなったけど)今度こそ証明したので報告。
安々と定理なんて言うもんじゃないな…

実数上の有界連続関数全体からなるベクトル空間を V とおく。
V 上の線形作用素 S:V→V が次を全て満たすとする:
(i)任意の a∈R に対して定まる平行移動作用素 T=T_a:V→V ; (Tf)(x)=f(x+a) について、T と S は可換。
(ii)f∈V が定数関数の時、Sf=f.
(iii)任意の a>0 について次が成り立つ:「f∈V が f(x)>0 for ∀x∈R-(aZ) を満たせば (Sf)(0)>0.」
(iv)次を満たす H>0 が存在する:「f∈V が f(x)=0 for ∀x∈[-H,H] を満たせば (Sf)(0)=0.」
(定理3.1)このような状況の時、Sf=f を満たす f∈V は定数関数のみである。

使う補題とすんごい大雑把な証明の概略
(補題1)f∈V が Sf=f を満たし、ある a>0 について f(x+a)=f(x) for ∀x∈R を満たせば、f は定数。
(∵)f(x) が f の最小値をとるような x の集合を L、 x∈L⇒x+a∈L を満たす a の集合を G とおくと、
G は閉集合で加法に関して群をなし、∪_(x∈L) {a∈R| x+a∈L でない} = R-G が言える。
R の強リンデレフ性から L の点列 {x_n} であって ∪_(n=1,∞) {a∈R| x_n+a∈L でない} = R-G を満たすものがとれて、
アスコリ=アルツェラの定理から F(x) = Σ_(n=1,∞) 2^(-n)_f(x+x_n) は連続で S 不変、
かつ ∀x∈R-G で F(x)>0 となるから (iii) より G=R となるしかない。□
0256132人目の素数さん
垢版 |
2018/11/15(木) 02:39:58.41ID:CcvRy6JY
>>253
Σ[k≧1] 1/((kπ)^2 + 1)
=Σ[k≧1i≧1] (-1(kπ)^2)^i
=Σ[i≧1] ζ(2i)(-1π^2)^i
=Σ[i≧1] (-1)^(i+1)B[2i](2π)^(2i)/2/(2i)!(-1π^2)^i
=Σ[i≧1] (-1)B[2i]2^(2i)/2/(2i)!
= (1/2)(coth 1 - 1)
=1/(e^2-1)
0257132人目の素数さん
垢版 |
2018/11/15(木) 03:57:25.66ID:0h25dXYt
>>255 の続き
(補題2)f∈V が Sf=f を満たし、lim(x→∞)f(x) と lim(x→-∞) f(x) が存在するならば、f は定数。
(∵)g(x)=f(x+1)-f(x) とおいて sup_x g(x) > 0 と仮定すると、g(x) が g の最大値になるような最小の x_0 と最大の x_1 について、
G(x):=g(x+x_0)+g(x+x_1) は S 不変かつ G(x)<G(0) for∀x≠0 となり、(iii) と矛盾。
これらの議論が -g にも適用できることから g が定数とわbゥるので、補題bPより f も定数。□

(補題3)f∈V が Sf=f を満たすならば、F(x):=∫_(0,1) f(x+t)dt も SF=F を満たす。
(∵)関数列 f_n(x):=(1/n)Σ_(k=1,n) f(x+k/n) のコンパクト一様収束性から従う。□
0258132人目の素数さん
垢版 |
2018/11/15(木) 04:33:13.19ID:BIkI04V5
>>253

オイラー積表示
 sinh(x) = x Π[k=1,∞] {1 + (x/kπ)^2},
より
 log|sinh(x)| = log|x| + Σ[k=1,∞] log{1 + (x/kπ)^2},
 coth(x) = cosh(x)/sinh(x) = ( log|sinh(x)| ) ' = 1/x + 2Σ[k=1,∞] x/{(kπ)^2 + x^2},
に x=1 を入れる…
0259132人目の素数さん
垢版 |
2018/11/15(木) 04:41:27.91ID:0h25dXYt
>>257 の続き 一番面倒な補題

(補題4)f∈V が1以下のリプシッツ係数を持ち Sf=f を満たすならば、lim_(x→∞) f(x) が存在する。
(∵)a:=liminf_(x→∞) f(x) < b:=limsup_(x→∞) f(x) であると仮定する。
各 r∈R に対して C(r)=lim_(ε→+0) limsup_(x→∞, f(x)<a+ε) f(x+r)-a とおくと、
アスコリアルツェラから C は有界かつ1以下のリプシッツ係数を持つことがわかる。
またこれより、各 r∈R に対して、非有界な上昇列 {x_n} であって n→∞ の時 f(x_n)-a→0、f(x_n+r)-a→C(r) を満たすものがとれるが
アスコリアルツェラから部分列 {x'_n} であって T_(x'_n)f-a が [-1,1] 上で一様収束するものがとれる。(Tの定義は(i)参照)
この部分列 {x''_n} をとって T_(x''_n)f-a が [-2,2] 上で一様収束するものがとれて、更に部分列 {x'''_n} をとって T_(x'''_n)f-a が [-3,3] 上で一様収束するものがとれて、…
と部分列を帰納的に定義できることから、{x_n} の部分列 {y_n} であって T_(y_n)f-a がコンパクト一様収束するものがとれる。
この関数列の収束先である F_r は 0≦F_r(x)≦C(x) for∀x かつ F_r(r)=C(r) を満たし、なおかつ S 不変。これが任意の r に定義できることから、
C(x) にコンパクト一様収束する V の関数列 {g_n} であって g_n(x)≧0 for∀x かつ (Sg_n)(0)=0 を満たすものが構成できるので、(SC)(0)=0.
r_1,r_2∈R について C(r_1)=C(r_2)=0 ⇒ C(r_1+r_2)=0 がわかるので、C の零点集合は (1) R の加法に関する離散部分群かその部分集合、(2) R 全体、
もしくは (3)0以上の実数全体か0以下の実数全体のどちらかの部分集合 X であって原点から離れるにつれ徐々に稠密になっていく集合、のどれかになる。
(続く)
0260132人目の素数さん
垢版 |
2018/11/15(木) 04:45:42.16ID:0h25dXYt
>>259 の続き

(1) の場合は (iii) と矛盾。(2) の場合は、十分小さい ε>0 と十分大きい実数 b_1<a_1<a_2<a_3<b_3 s.t.
(x≧b_1 かつ f(x)<a+ε ならば ∀r∈[-H,H] について f(x+y)<(a+b)/2) ∧ f(b_1),f(b_3)>(a+3b)/4
∧ f(a_2) = min_(t∈[b_1,b_3]) f(t) < a+ε ∧ a_1=min{t∈[b_1,b_3]|f(t)=f(a_2)} ∧ a_3=max{t∈[b_1,b_3]|f(t)=f(a_2)}
がとれるので、(T_(a_1)+T_(a_3))f(x)>2f(a_2) (for∀x∈[-H,0)∪(0,H]) から S(T_(a_1)+T_(a_3))f(0)>2f(a_2) となるが、これは f(a_1)+f(a_3)=2f(a_2) と矛盾。
これらから (3) の場合のみが残る。集合 X が伸びている方向を σ∈{+,-} とおくと、Xはσの方向に徐々に稠密になっていくから lim_(x→σ∞) C(x)=0.

これまでと全く同様の議論を -f に対して行い(勿論aとbも逆になる)、C にあたる関数を D とおく。
もし D の零点集合が伸びる方向が σ と逆ならば (C+D)(x)>0 for∀x≠0 より S(C+D)(0)>0 となるが、これは SC(0)=SD(0)=0 と矛盾。
よって、D は C と同じ方向に伸びるので、lim_(x→σ∞) D(x)=0. したがって、M>0, ε>0 であって
(M<x かつ f(x)<a+ε ならば、∀r∈[H,3H] について f(x)<(2a+b)/3) ∧ (M<x かつ f(x)<b+ε ならば、∀r∈[H,3H] について f(x)>(a+2b)/3)
を満たすものが存在。これより、M<b'_1<a'_1<a'_2<a'_3<b'_3 を
f(b_1),f(b_3) > b-ε ∧ f(a_2) = min_(t∈[b_1,b_3]) f(t) < a+ε ∧ a_1=min{t∈[b_1,b_3]|f(t)=f(a_2)} ∧ a_3=max{t∈[b_1,b_3]|f(t)=f(a_2)}
を満たすように定めれば、(1') |b'_1-a'_1|≦H または |b'_3-a'_3|≦H と (2') b'_1+H<a'_1 かつ a'_3<b'_3-H の二つの場合に分けてそれぞれ今までとほぼ同様に矛盾を示せる。□
0261132人目の素数さん
垢版 |
2018/11/15(木) 05:14:14.61ID:0h25dXYt
(定理3.1)f∈V が Sf=f を満たすならば、f は定数。
(∵)補題3から F(x):=∫_(0,1) f(x+t)dt も S 不変かつリプシッツ連続であるが、
補題4(と対称性)から lim(x→∞) F(x) と lim(x→-∞) F(x) が存在。
補題2から F は定数であるから、f(x)=f(x+1). これと補題1から f は定数。□

(系1)正の数からなる無限列 λ_1, λ_2,… の総和は1であり、
有界な実数列 a_1, a_2,… のうちQ上一次独立な二つ組が存在すると仮定する。
この時、実数上の有界連続関数 f で
f(x) = Σ_(n=1,∞) λ_nf(x+a_n) (for∀x)
を満たすものは定数関数に限られる。(証明略)

(系2)a<b を実数、φ を区間 [a,b] 上の連続関数であって
φ(t)≧0 (for∀t∈[a,b]), ∫_(a,b)φ(t)dt=1
を満たすものとする。この時、実数上の有界連続関数 f で
f(x) = ∫_(a,b)φ(t)f(x+t)dt (for∀x)
を満たすものは定数関数に限られる。(証明略)
0262132人目の素数さん
垢版 |
2018/11/15(木) 05:34:37.06ID:0h25dXYt
ちなみに、例えばこんなことはまだ示せていません。和をとったりしてる範囲が非有界なので

(予想1)実数上の有界連続関数 f で
f(x) = Σ_(n=1,∞) f(x+√n)*2^(-n) (for∀x)
を満たすものは定数関数に限られる。また、
f(x) = (1/√π)∫_(-∞,∞) f(x+t)e^(-t^2) dt (for∀x)
を満たすものも定数関数に限られる。

(予想2)S の条件のうち (iv) を
(iv)’ 関数列 f_n∈V (n=1,2,…) が一様有界かつ 0∈V にコンパクト一様収束するならば lim_(n→∞) (Sf_n)(0)=0.
まで緩めても、定理3.1と同様の主張が成り立つ。
0263132人目の素数さん
垢版 |
2018/11/15(木) 05:51:08.71ID:0h25dXYt
>>260 訂正
誤:
したがって、M>0, ε>0 であって
(M<x かつ f(x)<a+ε ならば、∀r∈[H,3H] について f(x)<(2a+b)/3) ∧ (M<x かつ f(x)<b+ε ならば、∀r∈[H,3H] について f(x)>(a+2b)/3)
を満たすものが存在。

正:
したがって、M>0, ε>0, m>3H であって
(M<x かつ f(x)<a+ε ならば、∀r∈[m-2H,m+2H] について f(x)<(2a+b)/3) ∧ (M<x かつ f(x)<b+ε ならば、∀r∈[m-2H,m+2H] について f(x)>(a+2b)/3)
を満たすものが存在。
0264132人目の素数さん
垢版 |
2018/11/15(木) 11:10:28.14ID:0YjszKFG
>>252

> >>36 の1つ目が本当にわからん…
> mod p での非自明な整数解がいつでも存在することは示せたから、

どうやって示すんですか?
0265132人目の素数さん
垢版 |
2018/11/15(木) 14:20:30.06ID:7EQ5hn9e
>>264
S(p,a) = Σ_(x,y,z,w=1〜p) e^(2πi(x^4+y^4-6z^4-12w^4)a/p),
S(p) = Σ_(a=1〜p) S(p,a)
とおくと、もし x^4+y^4-6z^4-12w^4=0 の解が (0,0,0,0) しか無ければ S(p)=p となるはず。
一方 S(p,p)=p^4 であり、Aがpで割りきれない時は確か
|Σ_(x=1〜p) e^(2πi(x^4)A/p)|≦3√p
が Vinogradov(1954) の "The Method of Trigonometrical Sums in the Theory of Numbers" から言えたはずだから、
S(p) ≧ p^4 - (p-1)(3√p)^4 = p^4 -
81p^3 + 81p^2.
以上から p≦80 がわかるから、あとはこれらの p についてパソコンか何かで調べ上げればOK
0266132人目の素数さん
垢版 |
2018/11/15(木) 14:41:42.62ID:7EQ5hn9e
>>265
もし x^4+y^4-6z^4-12w^4≡0 (mod p) の解が x≡y≡z≡w≡0 (mod p) しか無ければ
だった スマン
0268132人目の素数さん
垢版 |
2018/11/15(木) 17:24:26.51ID:0h25dXYt
>>262(予想2)普通にできたわ…何でこれ気づかなかったんや…

(∵)S の条件を変えたときに補題1,2,3は同じ議論で通用するから、補題4だけ証明すればよい。
まず g(x)=f(x+1)-f(x) とおき、この g に対して >>259 と同様に C を定めると、
(1)の場合に矛盾が生じるところまで同じ議論が成り立つ。
(2),(3) のいずれの場合も、σ∈{+,-} であって lim_(x→σ∞)C(x)=0 を満たすものが存在。
したがって、a = liminf(x→∞)g(x) < 0 と仮定すると、任意の N>0 について
M∈R, ε>0 s.t. C(r)<|a/6| for∀x∈[M,M+N] がとれるから、
十分大きい全ての x∈R s.t. g(x)<a+ε が g(x+r)<a/2 for∀r∈[M,M+N] を満たすことがわかる。
このような x について、
f(x+M+N)-f(x+M) = g(x+M)+g(x+M+1)+…+g(M+N-1) < Na/2
が成り立つが、N>0 は任意であったからこれは f の有界性に反する。したがって、a≧0.
-g や逆方向の極限についても同様であるから、lim_(x→±∞)g(x)=0.
補題2から g は定数であり、補題1から f も定数である。
以上から補題4にあたる部分が示されたが、
補題1〜4から主張を示す部分は、定理3.1と同様の議論が成立する。□

系として(予想1)も成り立つことがわかりますが、証明は略。
同様の結果が成り立つことを保証するのに条件(iv)'は本当に必要なのか?という疑問は当然わきますが、
任意の f∈V に対して定義されて f の x→±∞ での挙動「のみ」に依存する線形な作用素が、
そもそもそんな簡単には作れないような気がしてて、
ここは選択公理とか使ったりする必要がありそうと感じているため、
その辺の反例の構成とかは興味のある有志にお任せしたいと思います…
0269132人目の素数さん
垢版 |
2018/11/15(木) 17:29:24.42ID:0h25dXYt
>>268
誤:
M∈R, ε>0 s.t. C(r)<|a/6| for∀x∈[M,M+N] がとれるから、

正:
M∈R, ε>0 s.t. C_ε(r)<|a/6| for∀r∈[M,M+N] がとれるから、
0272132人目の素数さん
垢版 |
2018/11/15(木) 18:15:07.67ID:0YjszKFG
できるわけないやん。trは等しいけどdetは異なる行列なんか死ぬほどあるやろ?
0273132人目の素数さん
垢版 |
2018/11/15(木) 18:44:55.40ID:Kjq0ut8v
>>271
正解です。直ぐに出てくるなんて凄いな。
私は最近知ったんだけど、この手の内容について参考文献とかあったら教えてください。
0275132人目の素数さん
垢版 |
2018/11/15(木) 19:13:37.81ID:7EQ5hn9e
>>273
大学で使うような線形代数の教科書だったら何でも載ってる気がするけどなあ、わからんが
wikipedia の固有多項式の項目を見てみるだけでも結構色んな情報引き出せたりすると思うけど、
もし文献が欲しいならすまんが数学の本スレとか別のところで聞いてみてくれ

類題
3次正方行列Aについて、tr(A) を det(Aと単位行列Eの式) の式で表せ。(detの中身は複数種類でも可)
0276132人目の素数さん
垢版 |
2018/11/15(木) 23:25:29.70ID:v4atAZWV
池の鯉を網で56匹すくいました。
すくった56匹に目印をつけ、池にもどしました。
次の日に鯉45匹をすくったところ、36匹に目印がついていました。
池の鯉はおよそ何匹ですか。

これ、70匹でも69匹でも同確率になるよね?
0279132人目の素数さん
垢版 |
2018/11/16(金) 00:21:41.73ID:vFBRPbWk
56*45/36=70で求まる70匹のとき36匹の目印が見つかる確率

137149850039891/562949953421312
0.2436270741410791

69匹のときの36匹の目印が見つかる確率も
137149850039891/562949953421312
0.2436270741410791
となった。

import math
from fractions import Fraction
def choose(n, r):
return math.factorial(n) // (math.factorial(n - r) * math.factorial(r))
def dhyper(x,g,b,s):
return choose(g,x)*choose(b,s-x) / choose(g+b,s)
f69 = dhyper(36,56,69-56,45)
print (Fraction(f69))
print(f69)
f70 = dhyper(36,56,70-56,45)
print (Fraction(f70))
print (f70)
0280132人目の素数さん
垢版 |
2018/11/16(金) 00:25:09.70ID:LBn7birX
プログラムのご紹介、乙。
でも、ま、プログラムとしての新規性の証明、解説が必要かな?
0281132人目の素数さん
垢版 |
2018/11/16(金) 00:33:03.74ID:vFBRPbWk
>>280
Rの超幾何分布関数で算出したら
最頻値が2つ出てきたので分数表示できるpythonでやっても同じになって困惑してるのが現状。
0282132人目の素数さん
垢版 |
2018/11/16(金) 07:16:29.90ID:ZFCX7NBV
全ての成分が自然数で、対角成分が全て0の正方行列Aについて
tr(A^3)は6の倍数であることを証明せよ
0283132人目の素数さん
垢版 |
2018/11/16(金) 07:37:02.65ID:Av10eeCd
>>282
修正
正方行列→対称行列
です
0284132人目の素数さん
垢版 |
2018/11/16(金) 08:09:34.01ID:xmVvCZPI
>>282 >>283
B = A^3,
tr(B) = Σ[i=1,n] B(i,i)
= Σ[1≦i,j,k≦n] A(i,j) A(j,k) A(k,i)
= 6Σ[1≦i<j<k≦n] A(i,j) A(j,k) A(i,k)
∵ {i,j,k} のどれかが一致すれば 0
ぢゃね?
0285132人目の素数さん
垢版 |
2018/11/16(金) 08:47:46.29ID:U19cHKqd
単発質問スレより 引用

1問目は1から9を多くて1回づつ使って等式を完成させる
https://i.imgur.com/os8xCr9.jpg

□/□ * □/□ = □□/□

(a/b)*(c/d) = (10*e+f) /g

左辺の分数は互換なのでa>cとして

Prelude> let r = [[a,b,c,d,e,f,g]|a<-[1..9],b<-[2..9],c<-[1..9],d<-[2..9],e<-[1..9],f<-[1..9],g<-[2..9],(a/b)*(c/d)==(10*e+f)/g,
a/=b,a/=c,a/=d,a/=e,a/=f,a/=g,b/=c,b/=d,b/=e,b/=f,b/=g,c/=d,c/=e,c/=f,c/=g,d/=e,d/=f,d/=g,e/=f,e/=g,f/=g,a>c]
Prelude> let f x = map floor x
Prelude> map f r
[[7,2,3,6,1,4,8],[7,6,3,2,1,4,8],[8,2,3,6,1,4,7],[8,2,7,4,6,3,9],[8,2,7,6,1,4,3],[8,4,7,2,6,3,9],[8,4,7,6,2,1,9],[8,6,3,2,1,4,7],[9,2,7,4,6,3,8],
[9,2,8,4,6,3,7],[9,4,7,2,6,3,8],[9,4,7,6,2,1,8],[9,4,8,2,6,3,7],[9,4,8,6,2,1,7],[9,6,7,4,2,1,8],[9,6,8,4,2,1,7]]



2問目は2個答える
最大になるように-5から5を多くて1回づつ使う
最小になるように-5から5を多くて1回づつ使う
https://i.imgur.com/H7btl39.jpg

こっちが終わらない :(
0286イナ ◆/7jUdUKiSM
垢版 |
2018/11/16(金) 09:08:17.41ID:ksjAcdFq
>>277訂正。
16×5⇒14×5

ま、でも印をつけられる段階ですでに「ぜんぜん捕まらないすばっしっこい鯉」が10匹ぐらいいると思うんだよね。
すべての鯉をx匹、ぜんぜん捕まらないすばっしっこい鯉をy匹とおくと、
x-y=70(匹)
印をつけられる確率は4/5じゃない気がする。
印をつけるたびすでに印をつけられた鯉が生け簀に増えてくわけで、
すでに印をつけられた鯉を捕ることもあるはず。
y=10なら80匹だし、それに70匹ぐらいなら80匹もぎりぎりオッケーじゃないの。
0287132人目の素数さん
垢版 |
2018/11/16(金) 09:40:36.84ID:vFBRPbWk
>>285
2問目も計算、終わってた

y = [a/b*(c-d)-e*(f-g)|a<-[-5..5],b<- [-5..(-1)]++[1..5],c<-[-5..5],d<-[-5..5],e<-[-5..5],f<-[-5..5],g<-[-5..5],
a/=b,a/=c,a/=d,a/=e,a/=f,a/=g,b/=c,b/=d,b/=e,b/=f,b/=g,c/=d,c/=e,c/=f,c/=g,d/=e,d/=f,d/=g,e/=f,e/=g,f/=g]
f x = map floor x
yMax = maximum y
z=[[a,b,c,d,e,f,g]|a<-[-5..5],b<- [-5..(-1)]++[1..5],c<-[-5..5],d<-[-5..5],e<-[-5..5],f<-[-5..5],g<-[-5..5]
,a/=b,a/=c,a/=d,a/=e,a/=f,a/=g,b/=c,b/=d,b/=e,b/=f,b/=g,c/=d,c/=e,c/=f,c/=g,d/=e,d/=f,d/=g,e/=f,e/=g,f/=g,a/b*(c-d)-e*(f-g)==yMax]
map f z

Prelude> map f z
[[-5,-1,3,-4,5,-3,4],[-5,-1,3,-3,5,-4,4],[-5,-1,4,-4,5,-3,3],[-5,-1,4,-3,5,-4,3],[-5,1,-4,3,5,-3,4],[-5,1,-4,4,5,-3,3],[-5,1,-3,3,5,-4,4],[-5,1,-3,4,5,-4,3],
[5,-1,-4,3,-5,4,-3],[5,-1,-4,4,-5,3,-3],[5,-1,-3,3,-5,4,-4],[5,-1,-3,4,-5,3,-4],[5,1,3,-4,-5,4,-3],[5,1,3,-3,-5,4,-4],[5,1,4,-4,-5,3,-3],[5,1,4,-3,-5,3,-4]]
0288132人目の素数さん
垢版 |
2018/11/16(金) 09:55:52.98ID:U19cHKqd
>>286
70匹のときの確率
137149850039891/562949953421312  = 0.2436270741410791

80匹のときの確率 639173184839639/36028797018963968 = 0.017740619663298957

だから、80匹は可能性が低い
0289132人目の素数さん
垢版 |
2018/11/16(金) 10:03:56.75ID:U19cHKqd
>>286
池の鯉の可能性を56+(45−36)=65匹から上限を10000匹にして
その確率は一様分布に従う(つまり、65匹の確率も10000匹の確率も同じ)として計算したら

最頻値
$`mode`
[1] 69 70

中央値
$median
[1] 71

期待値
$mean
[1] 71.17647

95%信頼区間(highest density)
$CI.hdi
[1] 65 78

パーセンタイル(2.5.%-97.5%)
$CI.Qqtl
[1] 66 80

80匹はまあ、ギリギリセーフといえなくもない。
0290132人目の素数さん
垢版 |
2018/11/16(金) 10:33:03.57ID:3fj2avs/
>>36の出題者らしきレスなんにも出てこないけど、これ本当にとけるんかな?時々未解決問題貼るやついるからなぁ。
まぁもう諦めたからどっちでもいいけど。
0291132人目の素数さん
垢版 |
2018/11/16(金) 10:49:59.06ID:U19cHKqd
>>281
Wolframdでも同じだなぁ。69匹と70匹は同確率。
では、56*45/36=70の値は一体なんだろ

choose(56,36)*choose(13,9)/choose(69,45)
https://www.wolframalpha.com/input/?i=choose(56,36)*choose(13,9)%2Fchoose(69,45)
3591292705/14740942556
choose(56,36)*choose(14,9)/choose(70,45)
https://www.wolframalpha.com/input/?i=choose(56,36)*choose(14,9)%2Fchoose(70,45)
3591292705/14740942556

choose(13,9)/choose(69,45)
11/35471218518158136
(13!/(4!*9!)) /(69!/(24!*45!))

choose(14,9)/choose(70,45)
11/35471218518158136
(14!/(5!*9!))/(70!/(25!*45!))
0292132人目の素数さん
垢版 |
2018/11/16(金) 12:24:02.61ID:C4anFRr3
大きな数を扱う化学では

コップの中の真水56ccを濃度1%の食塩水56ccで置き換えました。
よく混ぜた後45cc取り出して濃度を測ると36/45%でした。
最初の真水の量は何ccだったのか計算せよ。

36/45=56/x
x=70
答え 70cc

とかしてるわけだけど、
確率分布はどんな様子になってるんだろう。
>>276の問題の各数値を10^23倍くらいにすればいいわけだよね
0293132人目の素数さん
垢版 |
2018/11/16(金) 13:11:22.71ID:cPC01E8S
小学校 「さくらんぼ計算」に戸惑う声
https://headlines.yahoo.co.jp/hl?a=20181115-00000006-jct-soci
https://amd.c.yimg.jp/im_sigg4tEc9zbWoY9xWDiiNhYroA---x900-y720-q90-exp3h-pril/amd/20181115-00000006-jct-000-1-view.jpg
さくらんぼ計算とは、「8+7」の足し算で、7を2と5に分け、8にこの2を足して10にする。
そして、10と残りの5を足して15と計算するやり方だ。7の下にぶら下がったさくらんぼの実を2つ描き、
2と5を実の中に書くことから、さくらんぼ計算と呼ばれている。

報告主は、「10+7」の10を3と7に分けるといったムダなことをする子供もいたとして、
こうした考え方を示した文科省に疑問をぶつけていた。このほかにも、さくらんぼ計算のせいで
娘が算数が大嫌いになり、中学3年になっても苦手から抜け出せずに数学を拒否している。
0294132人目の素数さん
垢版 |
2018/11/16(金) 13:55:38.45ID:WpNoJk+p
>>95
■P1stを求める

宝一つの時の自陣当たり数

(n(n+1)/2)-1 ……@

その中での宝二個の組み合わせ数

((n(n+1)/2)-1)(((n(n+1)/2)-1)-1)/2 ……A

最終マスと@との組み合わせ数  

(n(n+1)/2)-1 ……B

自陣の当たりと相手の当たりで自分が勝つ
組み合わせはAと差分の和

差分は1 3 7 13 22 34 50 70 95 125 161 203
252 308 372 444 525 615……

それを表す関数

(4n^3+6n^2-4n-3+3(-1)^n)/48

nが一つずれているのでn-1に補正

{4(n-1)^3+6(n-1)^2-4(n-1)-3+3(-1)^(n-1)}/48 ……C

計算知能でAx2+B+Cを入力すると

P1st ={12n^4+28n^3-42n^2-52n-3(-1)^n+51}/48 ……D

全n(n+1)マスで宝二個の組合わせ数

n(n+1){n(n+1)-1}/2 ……E

引き分け数は、n(n+1)-1と同着数の和

同着数は1 2 4 6 9 12 16 20 25……

これを表す関数は {2n^2-1+(-1)^(n)}/8 ……F

n(n+1)-1 ……G

計算知能でF+Gを入力すると

even =(10n^2+8n+(-1)^n-9)/8 ……H

計算知能でE-D-Hを入力すると

Q1st ={12n^4+20n^3-18n^2-20n-3(-1)^n+3}/48 
0295イナ ◆/7jUdUKiSM
垢版 |
2018/11/16(金) 14:41:56.79ID:ksjAcdFq
>>286
>>288-289>>291
期待値の問題か。
80匹もぎりぎりセーフってことで正解だね。
一万匹は無理だね。
金銭的にも興行的にも。
0296イナ ◆/7jUdUKiSM
垢版 |
2018/11/16(金) 15:20:42.09ID:ksjAcdFq
どこ〜かで〜かね〜がな〜ぁて〜♪ らし〜くな〜ぃこと〜ばか゚〜ぅか〜んで〜♪ さむ〜さか゚〜ここ〜ちよ〜くて〜♪ な〜んでこ〜ぃな〜んかして〜んだろ〜♪ 前>>295
~、、,, ~~゚~~~。~ ~~~ ~
(-.-))⌒〜っ゙~ ~ ~~~
υυ〜~~~ ~~ ~ ~゚ ~~
~~~~゚ ~ ~ ~~ ゚ ~~~ ~
0298132人目の素数さん
垢版 |
2018/11/16(金) 15:40:55.85ID:U19cHKqd
>>292
カードの照合の問題も、最初に選んだ10枚に印をつけて再捕獲したと考えればいいんだろうけど
200/3=66.6枚という最頻値がどれほどの信頼できるのか疑問。

固有の番号の書かれたカードが何枚あり、
その枚数は1000枚以下であることはわかっているが、その数を推定したい。
調査員が無作為に10枚選んで番号を記録して元に戻した。
別の調査員が無作為に20枚選んで番号を記録した。
二人の調査員の記録した番号を照合すると3枚の番号が一致していた。
この情報からカード枚数の期待値を求めよ。

事前分布としてある枚数である確率を一様分布にするのが現実離れといえるけど。
まあ、男女の生まれる確率分布を一様分布として計算するのに似ているかも。
0299132人目の素数さん
垢版 |
2018/11/16(金) 15:43:07.21ID:U19cHKqd
>さくらんぼ計算のせい

アベノセイダーズを彷彿とさせるような記述だなぁ。
0300132人目の素数さん
垢版 |
2018/11/16(金) 16:04:40.32ID:U19cHKqd
>>297
サイコロをふって1回1の目がでた。
サイコロを降った回数は1から100回のどれかである。
1回ふって1回出た確率、2回ふって1回出た確率、3回ふって1回出た確率、...、100回ふって1回出た確率
全部足したらいくらになる?
0302132人目の素数さん
垢版 |
2018/11/16(金) 16:37:21.65ID:U19cHKqd
問題の数値を変えてわずか3匹しか目印なしとすると

池の鯉を網で56匹すくいました。
すくった56匹に目印をつけ、池にもどしました。
次の日に鯉45匹をすくったところ、3匹に目印がついていました。
池の鯉はおよそ何匹ですか。

56*45/3=840匹になるのだけど、
この数値ってどれほどアテにしていいんだろうね?
0303132人目の素数さん
垢版 |
2018/11/16(金) 17:04:41.28ID:C4anFRr3
数値を10倍、100倍にしたときのグラフを描かせてみた
青が10倍で範囲は690匹から7100匹、
赤が100倍で範囲は6900〜7100匹

https://www.wolframalpha.com/input/?i=plot+choose(560,+360)*choose(x-560,+90)%2Fchoose(x,450),++choose(5600,+3600)*choose(x*10-5600,+900)%2Fchoose(x*10,4500),+from+x%3D690+to+710

https://i.imgur.com/wMoy71B.jpg

10倍程度だとあまり先鋭化しない
100倍でも思ったより先鋭化しない
化学で扱うような10^23あたりのサンプル数なら推定値の
±0.001%の範囲である確率が0.9999とかになるんだろうな
0304132人目の素数さん
垢版 |
2018/11/16(金) 17:10:17.60ID:iOODzE0M
そこが統計が数学科であんまり好まれないとこだろうねぇ。
確率の問題と深く関わってはいるけど厳密には確率の問題ではない。
母数の分布とは言うけどそんなもんわかりっこないから本来どうしようもないし。
けどわからんわからんいうてても何も始まらんので適当に××仮説とか立てて「こう考えるとするとこうなる」ぐらいの事しか言えない。
信頼区間にしてもwikiにも明示されてるけどあくまで"指標"でしかない。確率でもなんでもない。
確率だと思うには母数の分布になんかの仮定入れないと無理だけど、その仮定の入れ方ごとに答え変わるし、ましてや次はその仮定どれくらい信頼できるねんと言う話に戻ってしまう。
0305132人目の素数さん
垢版 |
2018/11/16(金) 17:20:57.75ID:U19cHKqd
>>303
全体の数より、再捕獲での陽性割合を変化させた方がグラフは大きく変化すると思う。
目印陽性がすくないと信頼区間が広くなって推定値が信頼できないが、目印陽性が多いと信頼区間が狭くなる。
0306132人目の素数さん
垢版 |
2018/11/16(金) 17:39:34.15ID:C4anFRr3
>>305
なるほど。

とはいえさっきはグラフの形だけ見て「あまり先鋭化しない」なんて書いたけど、
10倍では690〜710以外の範囲でも無視できない程の確率があるのに
100倍だと6900〜7100以外での確率はほぼゼロだから大分違うか
0308132人目の素数さん
垢版 |
2018/11/16(金) 17:59:04.35ID:3fj2avs/
そもそも信頼区間の定義とは違う意味で信頼区間という用語使ってるレス多いな。
0309132人目の素数さん
垢版 |
2018/11/16(金) 18:10:26.73ID:vFBRPbWk
信頼区間の計算式って沢山あるよな。
1000打数100安打の95%信頼区間
> binom::binom.confint(100,1000)
method x n mean lower upper
1 agresti-coull 100 1000 0.1000000 0.08284688 0.1202145
2 asymptotic 100 1000 0.1000000 0.08140615 0.1185939
3 bayes 100 1000 0.1003996 0.08206073 0.1191877
4 cloglog 100 1000 0.1000000 0.08239444 0.1195577
5 exact 100 1000 0.1000000 0.08210533 0.1202879
6 logit 100 1000 0.1000000 0.08288164 0.1201906
7 probit 100 1000 0.1000000 0.08264461 0.1198768
8 profile 100 1000 0.1000000 0.08243331 0.1196133
9 lrt 100 1000 0.1000000 0.08243172 0.1196130
10 prop.test 100 1000 0.1000000 0.08245237 0.1206909
11 wilson 100 1000 0.1000000 0.08290944 0.1201520
0310132人目の素数さん
垢版 |
2018/11/16(金) 19:58:26.87ID:U19cHKqd
>>305
自分でも興味があったので弄ってみた。

魚の総数を上限1万匹とし、その確率分布は一様分布を仮定。

再捕獲した45匹中何匹に目印がついているかで推測される95%信頼区間(Highest Density Interval)をグラフにしてみた。

http://i.imgur.com/NKcQ61u.png

Rのコードはここに置いた(通知を変えて実行できる)

http://tpcg.io/TBD7MO
0311132人目の素数さん
垢版 |
2018/11/16(金) 21:16:06.61ID:xmVvCZPI
>>271
 tr(AA) や tr(A^3) も使えばできますね。

Aの固有多項式は
 f(x) = det(A-xI) = det(A) - {(tr A)^2 - tr(AA)}/2・x + tr(A) xx - x^3,
ケーリー・ハミルトンにより
 f(A) = det(A)I - {(tr A)^2 - tr(AA)}/2・A + tr(A)AA - A^3 = O,
 det(A) = (1/3)tr[ {(tr A)^2 - tr(AA)}/2・A - tr(A)AA + A^3 ] = …
0312132人目の素数さん
垢版 |
2018/11/16(金) 22:24:21.87ID:3fj2avs/
tr(A^i)全部使っていいなら固有多項式の係数全部表せるのはまんまニュートンの漸化式やん。
0313132人目の素数さん
垢版 |
2018/11/16(金) 23:11:07.23ID:1P5sF5+y
>>305>>310
単に>>303風のグラフを描くなら対数で計算すればコップの中の水分子とかでも計算できますね
あとでやってみよう
0314132人目の素数さん
垢版 |
2018/11/17(土) 08:36:24.31ID:pOy6FHDl
>>311
A = {a_ij} が3次行列のとき
tr(A) = a11 + a22 + a33,
tr(AA) = (a11)^2 + (a22)^2 + (a33)^2 + 2 a12 a21 + 2 a23 a32 + 2 a31 a13,

∴Aの固有多項式は
det(A-xI) = det{[a11-x, a12, a13], [a21, a22-x, a23], [a31, a32, a33-x]}
= det(A) - (a11 a22 + a22 a33 + a33 a11 - a12 a21 - a23 a32 - a31 a13)x + (a11+a22+a33)xx -x^3
= det(A) - {(tr A)^2 -tr(AA)}/2 x + tr(A) xx - x^3,

>>312
n次多項式がn個の根λ1,λ2,・・・,λn をもつ(ガウス)が既知ならば
 tr(A^i) = (λ1)^i + (λ2)^i + … + (λn)^i,
これは {λ1,λ2,…,λn} のi次の対称多項式だから、1〜i次の基本対称式で表わせる。
逆に、i次の基本対称式は tr(A) 〜 tr(A^i) で表わせる。
0315132人目の素数さん
垢版 |
2018/11/17(土) 08:41:34.54ID:XBWc+pi8
池の鯉の総数と調査します。
五郎君が名前に因んで56匹を捕まえて目印をつけ、池にもどしました。
次の日に三郎君が自分の名前に因んで36匹の目印のついた鯉を捕まえることにしました。
鯉45匹めで予定の36匹が捕まりました。
池の鯉はおよそ何匹ですか。
0316132人目の素数さん
垢版 |
2018/11/17(土) 09:12:10.72ID:js5kwOKA
>>314
トレースって跡ともいうけど、ずっと「あと」って読んでいた。
線形代数の本の索引で、ア行になくサ行にあったので気づいたわ。
0318132人目の素数さん
垢版 |
2018/11/17(土) 12:09:53.28ID:pOy6FHDl
>>311
A = {a_ij} がn次行列のときも
tr(A) = a_11 + a_22 + ・・・ + a_nn,
tr(AA) = (a_11)^2 + … + (a_nn)^2 + 2 Σ[i<j] a_ij a_ji,
(trA)^2 - tr(AA) = 2Σ[i<j] (a_ii a_jj - a_ij a_ji),

∴ Aの固有多項式は
det(A-xI) = det{[a_11-x, a_12, …, a_1n], [a_21, a_22-x, …, a_2n], ……, [a_n1, a_n2, …, a_nn-x]}
= det(A) - …… + Σ[i<j] (a_ii a_jj - a_ij a_jj) (-x)^{n-2} + (a_11+…+a_nn)(-x)^{n-1} + (-x)^n
= det(A) - …… + {(tr A)^2 - tr(AA)}/2 (-x)^{n-2} + tr(A)(-x)^{n-1} + (-x)^n,
0319132人目の素数さん
垢版 |
2018/11/17(土) 12:30:54.72ID:NWNFiSSF
昨日出した問題だけど
数2Bまでの知識で解ける

a,b,c,dは実数とする。
a+c=-4/3, b+4ac+d=-2, ad+bc=4, bd=1のとき、(a^2-b)(c^2-d)<0を示せ。ただし、計算機は使ってはならない。
0320132人目の素数さん
垢版 |
2018/11/17(土) 15:28:43.18ID:A1Nd7rYy
あれ?
a^2d=a(4-bc), bc^2=c(4-ad)より
a^2d+bc^2=4(a+c)-(ad+bc)=-16/3-4
になって
(a^2-b)(c^2-d)
=(ac)^2-(a^2d+bc^2)+bd
=(ac)^2-(-16/3-4)+1
になって負になりっこない希ガス。
0321132人目の素数さん
垢版 |
2018/11/17(土) 15:32:26.94ID:UVEx0ybP
>>319
f(x) = (x^2+2ax+b)(x^2+2cx+d) = x^4-(8/3)x^3-2x^2+8x+1
をxで微分したら 4(x-2)(x-1)(x+1) となるから、
x=-1 で極小 f(-1)<0, x=1 で極大 f(1)>0, x=2 で極小 f(2)>0 となってそれ以外の区間で単調であるから、
f は ちょうど2つの単根を持つ。
したがって、(x+a)^2 - (a^2-b) と (x+c)^2 - (c^2-d) はどちらも重根を持たず、どちらか一方のみが2つの実根を持つことから、題意が示される。
0323132人目の素数さん
垢版 |
2018/11/17(土) 15:37:21.74ID:KDdXvc30
>>321
すげぇ……判別式の形露骨に出ないように姑息に2a,2cみたいな係数にしてたのによく見破ったな……

>>322
ある
ウルフラム先生が言ってるんだから間違いない多分
0326132人目の素数さん
垢版 |
2018/11/17(土) 15:56:05.11ID:KDdXvc30
>>325
これa,b,c,dの値固定されてるからacの値によらず(ac)^2-(-16/3-4)+1が常に負になる必要はないんじゃね(特定のa,b,c,dで負になるなら良い)
0327132人目の素数さん
垢版 |
2018/11/17(土) 16:02:33.51ID:A1Nd7rYy
>>326
いや、変形に計算間違いがないならacが実数である限り何であっても正になってしまう。
いまパソコンが壊れててて計算しか出来ないから検算しようがない。
自分で再計算しても同じとこ間違うのが関の山だし。
まぁ問題に一言も実数とは断ってないから実数でないのかもしれないけど。
0329132人目の素数さん
垢版 |
2018/11/17(土) 16:09:42.72ID:A1Nd7rYy
>>328
aについてしか解いてくれてないけど。
虚数解混じってるし。
全部実数になる解あるの?
0330132人目の素数さん
垢版 |
2018/11/17(土) 16:11:44.94ID:KDdXvc30
>>329
More rootってあるだろ、これ以上はスレチになるからこことは違うところで調べてくれ
0331132人目の素数さん
垢版 |
2018/11/17(土) 16:11:55.46ID:A1Nd7rYy
あ、いや、ごめんなさい。
実数解ひとつあれば全部実数になる解は自動的にあるな。
あれ?>>320どこ間違ってるんだろう。
パソコンないとこうも手詰まりになるもんだな。
0332132人目の素数さん
垢版 |
2018/11/17(土) 16:15:18.14ID:knKRx/Qn
>>331
> a^2d=a(4-bc), bc^2=c(4-ad)より
> a^2d+bc^2=4(a+c)-(ad+bc)=-16/3-4
a^2d+bc^2=4(a+c)-ac(b+d)
計算見直すだけだろ
0334132人目の素数さん
垢版 |
2018/11/17(土) 16:22:25.07ID:UVEx0ybP
Wolfram 先生に
factorize x^4-(8/3)x^3-2x^2+8x+1
ってお願いして irreducible factorization 出してもらえば普通に a,b,c,d 構成できるんじゃないの

あと >>320 だけど
a^2d+bc^2=4(a+c)-(ad+bc)=-16/3-4
は計算間違いで本当は
a^2d+bc^2=4(a+c)-(abc+acd)
になって特に何が言える訳でもないと思うよ
0335132人目の素数さん
垢版 |
2018/11/17(土) 16:25:04.30ID:js5kwOKA
まだやってたのか、NG入れてあぼーんしてたから気づかなんだわ。
数学板の常連は意外と面倒見がいいんだな。
0336132人目の素数さん
垢版 |
2018/11/17(土) 16:30:49.85ID:UVEx0ybP
解決してた すまん

未解決だけど投稿します
各項の係数の絶対値が1以下であるような整数係数多項式 f(x)≠0 であって、x^4+x^3+3x^2+x+1 で割り切れるようなものは存在するか。
0338132人目の素数さん
垢版 |
2018/11/17(土) 16:41:11.53ID:pHGjVcYj
m種類の文字をm^n個並べた円順列で、連続するn文字の並びがすべて異なるものは常に存在するか?
例:文字0,1の円順列[00011101]中の3文字の並びは 000,001,011,111,110,101,010,100 で、すべて異なる。
0339132人目の素数さん
垢版 |
2018/11/17(土) 18:50:57.91ID:A1Nd7rYy
じゃトレースがらみで

Aをn次正方行列とするとき、行列Bを

B[i j] =tr(A^(i+j))

で定められるn次正方行列とする。
この時

Aの固有値が全て異なることと det(B)≠0 である事は必要十分である事を示せ。
0341132人目の素数さん
垢版 |
2018/11/17(土) 19:39:06.00ID:pHGjVcYj
>>339,340
tr(A^i)はAの固有値のi乗の総和なので、行列BはAの固有値のヴァンデルモンド行列とその転置との積。
したがってBの行列式は差積の2乗つまり判別式。
0344132人目の素数さん
垢版 |
2018/11/18(日) 01:04:20.36ID:ZJOO9Eig
>>338
m=3、n=2の場合で考える。
別のケースでもいけるけど記述がうるさくなるので。
0〜m^n-1の整数を頂点とし[x/m] ≡ y (mod m^n/m)の時xからyに→を描いて向き付きのグラフGを作る。
このグラフが全ての点をちょうど一回づつ通る向き付きのループを持つ事を示せば良い。
隣接行列をfromが横方向、toが縦方向に来るようにとる。
また、見やすいようにfromは最下位桁以外が一致するものを固めてならべ、toは最上位桁以外が固まるように並べる。
今の場合なら例えば以下のようになる。
. 00 01 02 10 11 12 20 21 22
00 1 1 1
10 1 1 1
20 1 1 1
01 1 1 1
11 1 1 1
21 1 1 1
02 1 1 1
12 1 1 1
22 1 1 1
このなかの1から1をm^n個選びその表す部分グラフが連結成分数が1の向き付きループになるものを見つければ良い。
各行、各列からちょうど一個づつ選べば向き付きループの有限和にはなる。
例えば対角線上の1を全て選べば良い。しかしそれだと連結成分が複数出てくるので繋げていく。
まず左上のm×mを必要なだけ挿げ替えてそのブロックにあるループが成分一個になるようにする。
上の例なら
. 00 01 02 10 11 12 20 21 22
00 ○
10 ○
20 ○
01 ○
11 *
21 △
02 ○
12 △
22 ※
となる。
この時点で00→20→02→10→01→00、11→11、12→21→12、22→22の4成分。
ここで右上のブロックを占有するループの中の→のtoは最下位桁以外を無視して0〜m^n/m-1までの全ての数がでるから各ブロックを少なくとも1回づつ通過する。
よって先の様な挿げ替えを再び行って成分数を1にできる。
0345132人目の素数さん
垢版 |
2018/11/18(日) 01:42:47.40ID:ZJOO9Eig
訂正
×まず左上のm×mを必要なだけ挿げ替えて
○ まず左上の(m^n/m)×(m^n/m)を必要なだけ挿げ替えて

訂正ついでにも少し補足。
挿げ替えとはA→B、C→DをA→D、C→Bと→を選び直す事。
この作業を何回か繰り返せば同じブロックに属する→は全て同一成分に出来る。
今の例なら
A→B→‥→A、C→D→‥→Cという2成分が
A→D→‥→C→B→‥→Aとつながって1成分減る。
0346132人目の素数さん
垢版 |
2018/11/18(日) 02:05:20.86ID:ZJOO9Eig
あ、いらん訂正した。>>344であってます。
>>345は無かったことに。
同一ブロックに入るのはたとえばm=3, n=5ならたとえば
fromが02430,02431,02432,02433,02434,
toが. 00243,10243,20243,30243,40243
であるm×m個。
このfromがm個、toがm個のm×m通りは全て→で結ばれていて自由に挿げ替えられる。
0347132人目の素数さん
垢版 |
2018/11/18(日) 02:36:13.22ID:ZJOO9Eig
>>346
スレ汚しすまん。まだ間違ってる。
挿げ替えのアルゴリズムは>>344では不十分。
以外に訂正。

とりあえず、全ブロックの→を挿げ替えで同一成分にする。
これで十分。
もしこれで2成分以上あったとする。
しかし元のグラフは連結なので2成分X YとXの通過する点とYの通過する点を結ぶ→がみつかる。
実際異なる連結成分を結ぶパスで長さ最小のパスを取ればそれは→である。
たとえばm=5, n=5で
X: …→02432→30243→…
Y: …→02431→10243→…
の02432と10243の様な組みである。
そしてこの部分は同一ブロックに属する。
しかし同一ブロックの→は全て同一連結成分になる様に取っているのでそれらが異なる連結成分に属するのは矛盾。
0348132人目の素数さん
垢版 |
2018/11/18(日) 03:56:43.71ID:ENzLbcND
>>343

[1] (20点)
aを2以上の整数とし、有理数bを b = 1 + 1/a により定める。自然数nに対して、
 S_n = Σ[k=1,n] k^(1/a),
とおく。ただし、k^(1/a) とはa乗するとkになる正の実数のことである。
以下の設問に答えよ。
 (1) lim[n→∞] S_n / n^b = 1/b を示せ。
 (2) lim[n→∞] (S_n - (n^b)/b)= ∞ を示せ。

[4] (20点)
nを自然数とする。整数kに関する次の条件(C),(D)を考える。
 (C) 0≦k<n.
 (D) k/n ≦ 1/m < (k+1)/n を満たす自然数mが存在する。
条件(C),(D)を満たす整数kの個数を T_n とする。以下の設問に答えよ。
(1) T_50 を求めよ。
(2) 次の極限値を求めよ。
  lim[n→∞]log(T_n)/log(n)

http://suseum.jp/gq/question/2951
0349132人目の素数さん
垢版 |
2018/11/18(日) 16:54:39.76ID:ENzLbcND
>>321

x - 2/3 = X,
とおくと
f(x) = x^4 -(8/3)x^3 -2x^2 +8x +1
= X^4 -(14/3)X^2 +(80/27)X +(131/27)
= (X^2 +f)^2 -(14/3 +2f)(X-g)^2,
となり、2次因子に分解する。

2(14/3 +2f)g = (80/27),
ff -(14/3 +2f)gg = (131/27),
より
f^3 +(7/3)f^2 -(131/27)f -(9053/729) = 0,
g^3 -(2/5)g^2 +(7/3)g -(10/27) = 0,

f = (-7 +6(13+2√11)^{1/3} +6(13-2√11)^{1/3})/9 = 2.256314207884
g = (2 +3(4+25√11)^{1/3} -3(-4+25√11)^{1/3})/15 = 0.161393818172
0350132人目の素数さん
垢版 |
2018/11/19(月) 00:15:59.14ID:eL1RQpps
>>349 (続き)
 f(x) = (x^2+2ax+b) (x^2+2cx+d)
とする。

x^2 +2ax +b = X^2 +2√(7/6 +f/2)・(X-g) +f,
判別式 a^2 -b = (7/6 +f/2) -f +2g√(7/6 +f/2)
 = (7/6 -f/2) +20/[27√(7/6 +f/2)]
 = 0.5274900867207

x^2 +2cx +d = X^2 -2√(7/6 +f/2)・(X-g) +f,
判別式 c^2 -d = (7/6 +f/2) -f -2g√(7/6 +f/2)
 = (7/6 -f/2) -20/[27√(7/6 +f/2)]
 = -0.4504709612713

(a^2 -b)(c^2 -d) = (7/6 -f/2)^2 -(80/27)^2/(7/6 +f/2)
 = -(4/3)[10 - (13+2√11)^{2/3} - (13-2√11)^{2/3}]
 = -0.237618966426144
 < 0,
0351132人目の素数さん
垢版 |
2018/11/19(月) 07:45:15.59ID:5ARxnQrn
問題 : 4 リットルと 3 リットルの容器を使って 2 リットルの水を測るにはどうすればいい?
これが最短手順でいい?
[(0,0),(4,0),(1,3),(1,0),(0,1),(4,1),(2,3),(2,0)]
0353132人目の素数さん
垢版 |
2018/11/19(月) 11:25:42.32ID:eL1RQpps
>>350 訂正

(a^2 -b)(c^2 -d) = (7/6 -f/2)^2 - (20/27)^2/(7/6 +f/2)
 = (1/3)[10 - (13+2√11)^{2/3} - (13-2√11)^{2/3}]
 = -0.237618966426144
 < 0,

分かスレ448-819,827
0356イナ ◆/7jUdUKiSM
垢版 |
2018/11/19(月) 18:19:06.42ID:/GTUzlHS
>>355満タン(10)(7)(3)の三つの容器をこの並びで置き、
初め(10)(0)(0)入っているのを移していく
→(3)(7)(0)
→(3)(4)(3)
→(6)(4)(0)
→(6)(1)(3)
→(9)(0)(1)
→(2)(7)(1)
→(2)(5)(3)
→(5)(5)(0)
8回で二分できた。
0357132人目の素数さん
垢版 |
2018/11/19(月) 18:46:32.29ID:TVJAPYv0
>>344-347
だいたい考えてた解答と一緒です。
自分のは、1234→2341→3412→4123→1234のようなサイクルをつないでいくやり方でした。

探索なしで生成する方法はないんでしょうかね?
0358イナ ◆/7jUdUKiSM
垢版 |
2018/11/19(月) 18:46:44.45ID:/GTUzlHS
>>356両手ありで空にしたとこにすぐ入れるのをノーカンにするなら7回だけど、
(10)(0)(0)
→(3)(7)(0)
→(3)(4)(3)
→(6)(4)(0)省略可
→(6)(1)(3)
→(9)(1)(0)省略可
→(9)(0)(1)
→(2)(7)(1)
→(2)(5)(3)
→(5)(5)(0)
片手だと9回か。
0359132人目の素数さん
垢版 |
2018/11/19(月) 23:43:46.25ID:eL1RQpps
>>350 >>353

(a^2 -b)(c^2 -d) = (7/6 -f/2)^2 - (20/27)^2/(7/6 +f/2)
 = (1/3) [(13+2√11)^{1/3} - (13-2√11)^{1/3}]^2
 = -0.237618966426144
 < 0,
0360132人目の素数さん
垢版 |
2018/11/19(月) 23:55:14.32ID:yvqX1603
>>359
■式を展開してゆくと

(a^2-b)(c^2-d)<0
a^2c^2-a^2d-c^2b+bd<0
a^2c^2-(a^2d+c^2b)+bd<0
a^2c^2-(a+c)(ad+bc)+ac(b+d)+bd<0
ac(ac+b+d)-(a+c)(ad+bc)+bd<0
ac(b+4ac+d)-3a^2c^2-(a+c)(ad+bc)+bd<0

a+c=-4/3, b+4ac+d=-2, ad+bc=4, bd=1
は作ることができたが

最終的に9a^2c^2+6ac-19>0となる
0361132人目の素数さん
垢版 |
2018/11/19(月) 23:55:39.30ID:vaYg27wd
>>357
え?自作問題なんですか?
よくこんなの思いつきましたね!すっげ!
とりあえず>>344-347みたいな解答できた後、明示的な解が作れないか考えたけど出来ずorz。
普段ならパソコンに探索させてやってみるんですが、今パソコン壊れててそれもできず。
少なくともm=2とかに限定すれば出来て不思議なさそうなんですけどねぇ?
0362132人目の素数さん
垢版 |
2018/11/20(火) 01:41:08.34ID:bRya54dl
>>355
こうなると人間技では無理だな。

100Lの容器いっぱいに油が入っています。51Lの容器と49Lの容器を使って、この油を50Lずつに分けます。どのような分け方がありますか。
0365132人目の素数さん
垢版 |
2018/11/20(火) 20:29:08.33ID:2b66cHGf
無限にも一般化できそうねこれ 既に解答あるかどうか知ってる訳じゃないけど

n を正の整数とする時、関数 f:Z→Z であって、F:Z→Z^n;F(x)=(f(x+1),f(x+2),…,f(x+n)) が全単射になるものは常に存在するか?
0366132人目の素数さん
垢版 |
2018/11/20(火) 20:41:37.58ID:2b66cHGf
De Bruijn torus みたいに多次元への一般化も考えられてるみたいだし、更にこんな一般化もできる ごっちゃごちゃやけど

m, n_1, n_2,…, n_m を正の整数とし、N=n_1*n_2*…*n_m とおく。この時、関数 f:Z^m→Z であって
F:Z^m→Z^N; F(x) = ( f(x+(1,1,…,1)), f(x+(1,1,…,2)), …, f(x+(n_1,n_2,…,n_m)) )
が全単射になるものは常に存在するか?
0367132人目の素数さん
垢版 |
2018/11/21(水) 01:28:51.54ID:e25FOlfs
とりあえず>>364 のページには ”こうすりゃできる” というのが載ってはいるが、なぜそれで出来るサッパリわがんね。
試しにm=n=3で手計算でやってみると

aaabaacabbabcacbaccbbbcbccc

……できてる……すっげ!
0372132人目の素数さん
垢版 |
2018/11/21(水) 18:57:16.48ID:vRlJnRg/
>>365
文字が整数ではなく自然数だったら、つまりf:N→N, F:N→N^nだったら、
n=2の場合、0 01 1 0212 2 031323 3 0414…
n=3の場合、0 001011 1 002012022102112122 2 003013…
のようにすればよさそうだけど、n≧4の場合はどうすればいいんだろう?
0375132人目の素数さん
垢版 |
2018/11/22(木) 01:47:18.64ID:x/Au2Ugh
>>374
左上から
 横7cm → 縦5cm → 横4cm → 横8cm → 縦9cm → 横5cm,

下3段の縦の比
右端から 4:4:8 → 4:4:9 → 8:9 → 8:4:5 →→ 7:7:5:4:5
横の比 20:35:21:(5cm)
0377132人目の素数さん
垢版 |
2018/11/23(金) 01:34:48.76ID:4a5jgypk
>>363のwikiに載ってる方法
0 ≦ x ≦ m^n-1をみたす整数を(必要なら上位を0で埋めて)m進数n桁表示で表示したとき、その最上位以外をひとつずつ上位に移し、最高位を最下位に移す写像を f とする。
このとき f は0 ≦ x ≦ m^n-1をみたす整数の全体 S の置換を与える。
この置換を互いに可換な循環置換の積 f = g[1]g[2]…g[t] で表す。
ただしg[i] = (n[i1]…n[ij]) n[i1]が最小限と表示するときワードw[i] = n[i1]…n[ij]は辞書式順序で昇順であるとする。
n[11]n[12]…をすべてつなげたワードをwとする。
そのワードの各文字をm^(n-1)で割った商で置き換えたワードがde Bruijn sequenceとなる。

--例--
m=3、n=2のとき。
f = (0) (1 3) (2 6) (4) (5 7) (8)
であるから
w = 0 1 3 2 6 4 5 7 8
であり各文字を3^1で割った商に置き換えて
001021222
はde Bruijn sequenceとなる。

……やっと証明わかった。
こんな構成法絶対思いつかん。
0378132人目の素数さん
垢版 |
2018/11/23(金) 18:46:55.67ID:imnUH0Gw
実数 x に対して [x] を x の整数部分と定める。この時、次の値を求めよ:
lim_(n→∞) (1/√n)Σ_(k=1,[√n])(n/k-[n/k])
0379132人目の素数さん
垢版 |
2018/11/23(金) 21:38:04.43ID:v31SUiKG
>>378
収束するん?
全然収束してる感ないけど?

*Main> let f n = (/(sqrt n)) $ sum [n/k - (fromInteger $ truncate $ n/k) |k<-[1..(sqrt n)]]
*Main> mapM_ print [f x | x<-[10000..10010]]
0.39775176396203077
0.42960405947242447
0.4414551710047714
0.4733020995001393
0.46515384320018427
0.4370114006470376
0.4688537799785083
0.5106894801475358
0.4425649747743292
0.4843978105580605
0.37630141203414064
0380132人目の素数さん
垢版 |
2018/11/23(金) 22:23:27.31ID:afoQoOE+
じゃあ問題変えよか
次を示せ:
Σ_(k=1,[√n]) n/k-[n/k] = (1/2)√n + O(n^(10/21))
0381132人目の素数さん
垢版 |
2018/11/23(金) 23:06:09.28ID:K/FSQatd
>>380

>>378より主張が強くなってるけどホントに成立するん?
相当収束遅いんかな?
100000000000でもまだ0.01ぐらいの誤差あるけど。
この辺までくると計算誤差かもしれないけど。
もってる R(n)/n^(10/21) の上からの評価値と矛盾してない?
収束してても不思議ない感くらいはあるんだけど。

*Main> mapM_ print [f x | x<-[100000000000..100000000010]]
0.49838288499722044
0.4983868107954477
0.4983780874456663
0.4984167982310302
0.49843969769390706
0.4984309743244439
0.4984601983048952
0.4984957468592659
0.4984870234930951
0.49847830012017674
0.4984695768000378
0382132人目の素数さん
垢版 |
2018/11/23(金) 23:47:54.50ID:afoQoOE+
>>381
誤差項の係数まで出してるわけじゃないから計算一つ一つ追ってくのはしんどくてすぐできそうにないけど、>>380 の右辺を
(√n)・(1/2 + O(n^(-1/42)))
と変形できて、n にそのまま 10^11 入れて n^(-1/42) 計算しても 0.547 くらいになるから、
むしろ >>381 の数値の 1/2 との近さは"良すぎる"くらいだね 言い換えれば >>380 の評価がガバガバすぎるってことなんだけど
0384132人目の素数さん
垢版 |
2018/11/24(土) 00:57:22.81ID:R0eGczxp
>>275
> 3次正方行列Aについて、tr(A) を det(Aと単位行列Eの式) の式で表せ。(detの中身は複数種類でも可)

これどうやるんですか?
0385132人目の素数さん
垢版 |
2018/11/24(土) 01:23:19.32ID:lLG6RWzh
>>384
A の固有値を a,b,c とおくと
det(A+E) + det(A-E) - 2det(A)
= (a+1)(b+1)(c+1) + (a-1)(b-1)(c-1) - 2abc
= 2(a+b+c) = 2tr(A)
でいける
0388132人目の素数さん
垢版 |
2018/11/24(土) 10:55:38.86ID:9xCWeV16
>>387
想定してる解答ではそういう複素解析的な技術は使ってないよ
使えるのかどうかはわからないけど、n>n' として n' の時の和が n の時の和の部分和になってる訳じゃないから
今までと同じような方法で臨むのは難しそうな気がする
0389132人目の素数さん
垢版 |
2018/11/24(土) 12:56:55.99ID:464U4GYy
>>388
違うのか…
Σ_(k=1,[√n]) (n/k-[n/k] - 1/2) = O(n^(10/21))
を示せば十分で
x - [x] - 1/2 = -2/πΣ[i] sin(2πix)/i
を使って
Σ_(k=1,[√n]) (n/k-[n/k] - 1/2) = -2/πΣ[k] Σ[i] sin (2πi n/k) / i

Σ[i] sin (2πi n/k) / i = Σ [ξ] <sin (2πi n/k), ξ(i)> L(ξ, 1)
でL(ξ,1)を評価するのかなと。
でもこのL(ξ,1)の評価ネットで探してもありそでなさそで……
これむずい。気長にやります。
0390132人目の素数さん
垢版 |
2018/11/24(土) 13:09:45.85ID:HscCLQgZ
{x}=x−[x]と定義して、

Σ(1≦k≦x^{1/2}){x/k}=(1/2)x^{1/2}+O(x^{1/3})

が示せた(計算ミスがなければ)。方針は>>389なんだけど、
L関数ではなく、普通にフェイェール核を使う。
あとはオイラー・マクローリン。
0393132人目の素数さん
垢版 |
2018/11/24(土) 13:30:52.39ID:464U4GYy
私も>>389訂正。L(ξ, 1)じゃなくてL(1,ξ)ね。
あとwolframで確かめてみたら
x - [x] - 1/2 = -1/πΣ[i] sin(2πix)/iですね。
https://www.wolframalpha.com/input/?i=plot+-1%2Fpi*(sin(1+x)%2F1%2Bsin(2+x)%2F2%2B+sin(3+x)%2F3%2Bsin(4+x)%2F4%2Bsin(5+x)%2F5%2Bsin(6+x)%2F6%2Bsin(7+x)%2F7%2Bsin(8+x)%2F8%2Bsin(9+x)%2F9%2Bsin(10+x)%2F10)
これ気合で i:1〜10 足したけどもっと賢く入力できないのかな?
ここからオイラーマクローリンで行こうとおもったんだけど
∫[1,∞] a cos (ax)x (x - [x] - 1/2) dx (ただし a = 2πn/k)
の項が出てきて評価ができなかった。
Dirichlet の不連続因子っての使うのかなとも思ったんだけど眠くなってやめた。
積分苦手 orz。
0394132人目の素数さん
垢版 |
2018/11/24(土) 14:01:12.00ID:HscCLQgZ
計算が面倒くさすぎるので、使う道具だけ書いておきます。間違ってたらスマン。

m≧0に対して、D_m:R→R を D_m(x)=(sin mπx)/sin πx (x∈R−Z), m (x∈Z) と定義する。
また、f_m=D_{2m+1} (m≧0) と定義する。

基本的な性質の一覧。

D_0(x)=0, D_1(x)=1, D_m(1±x)=D_m(x)=D_m(−x) (x∈R, m≧1)

f_0(x)=1, f_m(1±x)=f_m(x)=f_m(−x) (x∈R, m≧1)

m≧1に対して∫(0,1/2)f_m(t)dt=∫(1/2,1)f_m(t)dt=1/2

m≧1, x∈R に対して Σ(n=1〜m) cos2πnx = −1/2+f_m(x)/2

m≧1, x∈R に対して {x}−1/2=−Σ(n=1〜m) (sin 2πnx)/(πn)+∫(1/2,{x})f_m(t)dt
0395132人目の素数さん
垢版 |
2018/11/24(土) 14:12:41.48ID:HscCLQgZ
定義 命題Pに対して、[[P]] ∈ {0,1} を次のように定義する。

[[P]]=0 (Pが偽のとき), 1 (Pが真のとき).

定理1 x∈R に対して lim(m→∞)∫(1/2,{x})D_m(t)dt = −[[x∈Z]]/2.
特に、x∈R に対して lim(m→∞)∫(1/2,{x})f_m(t)dt = −[[x∈Z]]/2.
特に、x∈R に対して Σ(n=1〜∞) (sin 2πnx)/(πn) = 1/2−{x}−[[x∈Z]]/2.
0396132人目の素数さん
垢版 |
2018/11/24(土) 14:18:51.97ID:HscCLQgZ
定理2 g:[0,1/2]→Rは[0,1/2]上でルベーグ積分可能であり、g(+0)が存在するとする。このとき、

Clim(m→∞)∫(0,1/2)f_m(t)g(t)dt=g(+0)/2.

ただし、実数列 {a_m}_m に対して Clim(m→∞)a_m=lim(m→∞)(a_1+a_2+…+a_m)/m と定義する。

定理3 a<bとする。g:[a,b]→Rは[a,b]上でルベーグ積分可能であり、n∈[a,b]∩Zのとき、
g(x)はx=nにおいて片側極限が必ず存在するとする。このとき、

Clim(m→∞)∫(a,b)f_m(t)g(t)dt=Σ(n∈[a,b]∩Z) (g(n−0)+g(n+0))/2.

ただし、a∈Z のときは、(g(a−0)+g(a+0))/2 の部分を g(a+0)/2 で置き換える。
また、b∈Z のときは、(g(b−0)+g(b+0))/2 の部分を g(b−0)/2 で置き換える。
0397132人目の素数さん
垢版 |
2018/11/24(土) 14:21:47.97ID:HscCLQgZ
定理4(オイラー・マクローリンの公式) a∈Z, b∈R, a<b とする。
f:[a,b]→C はC^1級とする。このとき、任意の実数 x∈[a,b] に対して

Σ(a≦k≦x)f(k)=∫(a,x)f(t)dt+(f(a)+(1−2{x})f(x))/2+∫(a,x)f'(t)({t}−1/2)dt.

定理5(オイラー・マクローリンの公式) a, b∈R, a<b とする。
f:[a,b]→C はC^1級とする。このとき、任意の実数 x∈[a,b] に対して

Σ(a<k≦x)f(k)=∫(a,x)f(t)dt+((1−2{x})f(x)−(1−2{a})f(a))/2+∫(a,x)f'(t)({t}−1/2)dt.
0398132人目の素数さん
垢版 |
2018/11/24(土) 14:32:22.84ID:HscCLQgZ
Σ(1≦k≦x^{1/2}){x/k}=(1/2)x^{1/2}+O(x^{1/3}) の証明を大雑把に。

x>1として、Σ(1≦k≦x^{1/2}){x/k}=Σ(1≦k≦x^{1/3}){x/k}+Σ(x^{1/3}<k≦x^{1/2}){x/k}
と分解して、まず Σ(1≦k≦x^{1/3}){x/k}=O(x^{1/3}). 次に、m≧1を任意に取って

Σ(x^{1/3}<k≦x^{1/2}){x/k}
=Σ(x^{1/3}<k≦x^{1/2})(1/2−Σ(n=1〜m) (sin 2πnx/k)/(πn)+∫(1/2,{x/k})f_m(t)dt)
=([x^{1/2}]−[x^{1/3}])/2−Σ(n=1〜m)(1/(πn))Σ(x^{1/3}<k≦x^{1/2}) sin 2πnx/k
 +Σ(x^{1/3}<k≦x^{1/2})∫(1/2,{x/k})f_m(t)dt (1)

と分解する。
0400132人目の素数さん
垢版 |
2018/11/24(土) 14:42:15.81ID:UbXGWPhk
>>398
kの小さいとこ分けるのはやったんだけどなぁ。
まぁやってみます。
ところで出題者さんに質問。
この誤差項のx^(10/21)の肩の数字はいくらでも小さく採れるんですか?
0403132人目の素数さん
垢版 |
2018/11/24(土) 15:35:04.80ID:qzNDTVhe
>>400
や、今解答として用意してる計算のしかたであれば、最善のパラメーターのとり方でこの結果。
もっと和のとり方を工夫したら改善できるかもだけども
0405132人目の素数さん
垢版 |
2018/11/24(土) 16:48:54.13ID:qzNDTVhe
普通に改善できた…でもパラメータ変えて誤差項の指数小さくできるかはっきりしないし問題は変えないことにする
(もし小さくできたとしてもややこしくなるだけだし変えないつもりだけど)

>>404
うーん、フーリエ展開から攻めるのはちょっとオススメしにくい
小数部分をとる関数 {x} に不連続点があるせいで級数が絶対収束しないから、
各フーリエ係数ごとに和をとってから全体を評価しようとするとどうしてもばかでかくなってしまう気がする
フーリエ級数いじるの得意じゃないし本当にできないのかとかについては何とも言えないけど
0406132人目の素数さん
垢版 |
2018/11/24(土) 16:55:34.93ID:qzNDTVhe
停滞するのもあれだし類題出しときます

lim_(n→∞) (1/logn)Σ_(k=1,[logn]) (n/k-[n/k])
は存在するか。
0407132人目の素数さん
垢版 |
2018/11/24(土) 17:17:24.06ID:HscCLQgZ
まだ暗算レベルだけど、Σ(k≦x^{1/2})({x/k}^2−{x/k}) だったら、
>>394-399で失敗した計算が救済できて

Σ(k≦x^{1/2})({x/n}^2−{x/n})=(-1/6)x^{1/2}+O(x^{1/3})

あたりのオーダーが言えそうな気がする。
また間違うかもしれないので書かないけどw
0408132人目の素数さん
垢版 |
2018/11/24(土) 19:35:26.74ID:HscCLQgZ
>>407もダメだった。

B(t)={t}−1/2と置くと、>>407の場合、
∫(x^{1/3},x^{1/2})B(t)B(x/t)dt みたいなのが出てきて、
これのxに関するオーダーが計算できないw

どうやら、オイラー・マクローリンで計算すると、{x/n}の難しいところが
B(t)に移転してしまい、B(t)の積分計算ができなくなって失敗するっぽい。
0409132人目の素数さん
垢版 |
2018/11/25(日) 02:09:19.35ID:AuW29Ma5
>>275 >>384 >>385

・4次のとき
 det(A+xE) - det(A-xE)
 = (a+x)(b+x)(c+x)(d+x) - (a-x)(b-x)(c-x)(d-x)
 = 2 S_3 x + 2 tr(A) x^3,
より
 tr(A) = {det(A+2E) -2det(A+E) +2det(A-E) -det(A-2E)}/(2・3!)

・5次のとき
 det(A+xE) -2det(A) + det(A-xE)
 = (a+x)(b+x)(c+x)(d+x)(e+x) -2abcde + (a-x)(b-x)(c-x)(d-x)(e-x)
 = 2 S_3 x^2 + 2 tr(A) x^4,
より
 tr(A) = {det(A+2E) -4det(A+E) +6det(A) -4det(A-E) +det(A-2E)}/(4!)

・6次のとき
 det(A+xE) - det(A-xE)
 = (a+x)(b+x)(c+x)(d+x)(e+x)(f+x) - (a-x)(b-x)(c-x)(d-x)(e-x)(f-x)
 = 2 S_5 x + 2 S_3 x^3 + 2 tr(A) x^5,
より
 tr(A) = {det(A+3E) -4det(A+2E) +5det(A+E) -5det(A-E) +4det(A-2E) -det(A-3E)}/(2・5!)

・7次のとき
 det(A+xE) -2det(A) + det(A-xE)
 = (a+x)(b+x)(c+x)(d+x)(e+x)(f+x)(g+x) -2abcdefg + (a-x)(b-x)(c-x)(d-x)(e-x)(f-x)(g-x)
 = 2 S_5 x^2 + 2 S_3 x^4 + 2 tr(A) x^6,
より
 tr(A) = {det(A+3E) -6det(A+2E) +15det(A+E) -20det(A) +15det(A-E) -6det(A-2E) +det(A-3E)}/(2・6!)

S_k はk次の基本対称式。 S_1 = tr(A), S_n = det(A),
0410132人目の素数さん
垢版 |
2018/11/25(日) 02:25:52.63ID:AuW29Ma5
>>275 >>384 >>385
A, E はn次の正方行列とする。

・nが奇数のとき
 tr(A) = {1/(n-1)!}Σ[k=0,n-1] (-1)^k C(n-1,k) det{A + (k-(n-1)/2)E},

・nが偶数のとき
 tr(A) = {1/2(n-1)!}Σ[k=0,n] (-1)^k {C(n-1,k)-C(n-1,k-1)} det{A + (k-n/2)E},
 ただし、C(n-1,n) = C(n-1,-1) = 0
0411132人目の素数さん
垢版 |
2018/11/25(日) 02:42:24.60ID:XzUMGiUk
和のとり方を改善してパラメータをとり直した結果、
>>380 の誤差項を O(n^(5/12)・logn) まで改善できたので一応報告
けど予定通り、問題の主張を変えるつもりはなし
0413132人目の素数さん
垢版 |
2018/11/25(日) 14:43:15.35ID:4bOKPxfm
>>412
手助けになればいいけど、下の定理2.1.6を使えば Σ_(k=√n) sin(n/k) = O(n^(1/3)) まで落とせそう
https://spectrum.library.concordia.ca/978881/

Theorem2.1.6 (2nd derivative test)
fは区間[a,b]上で二階連続的微分可能であり、c>1 であって
0<λ≦f''(t)≦cλ (for∀t∈[a,b])
が成り立っているとする。この時、
Σ_(a<k≦b) e^(2πif(k)) = O_c( (b-a)λ^(1/2) + λ^(-1/2) ).□

具体的には m を正の整数(この場合およそ (√n)/(2π) )として、j=0,1,… に対して a_j = m・2^(-j-1), b_j = a・2^(-j) と定めて、
f(x)=N/x として各区間 [a_j,b_j] に対して定理を適用する訳なんだけど、
この場合 j の値にかかわらず c=8 で一様にとれるから、j=0,1,…,r で足し合わせても同じ implied constant で抑えられる。
あとは r の値をうまく調整すればOK
0414132人目の素数さん
垢版 |
2018/11/25(日) 18:39:02.79ID:9eLajdTN
>>413
sin 2πn/k はうまくいきました。
でも本丸は∫ a*cos(2πax)/x dxなんですよね〜。
こいつがO(1/a)とかならうまくいくし、数値実験てきには成立してそうなんだけどむずい。
やっぱりこりゃノーヒントじゃ無理だ。
>>378さんヒントお願いします。
0415132人目の素数さん
垢版 |
2018/11/25(日) 21:03:53.65ID:0+5Uplew
1時間に2本の列車があり、8:00 8:45 9:00 9:45 10:00 10:45 ...という風にダイヤが組まれている。
ランダムに駅に行ったときの待ち時間の期待値はいくらか?
0416132人目の素数さん
垢版 |
2018/11/25(日) 21:07:15.08ID:MRMVN18u
>>414
ほい

(補題)h/q (q>0) は既約分数であり、k=1,2,…,q の時実数 r(k) の絶対値は E 以下であるとする。
この時、任意の実数 a について次が成り立つ:
|Σ_(k=0,q-1) ({hk/q + a + r(k)} - 1/2)| ≦ 3qE + 3.
(ただし特にことわらなければ {x} は x の小数部分を表すものとする)
(∵)σ(k) := [q{hk/q + a}] は集合 {0,1,…,q-1} 上の全単射であるから、
ε(k) := {hk/q + a} - σ(k)/q ∈ [0,1/q) とおけば
Σ_(k=0,q-1) {hk/q + a + r(k)}
= Σ_(k=0,q-1) {σ(k)/q + ε(k) + r(k)}
= Σ_(k=0,q-1) {(k + 1/2)/q + r'(k)}. (σ(k) を k で置き換え。ただし |r'(k)| ≦ E + 1/(2q) )
ここで、r'(k)=0 (for k=0,1,…,q-1) の時この和が q/2 に等しくなることから、
| Σ_(k=0,q-1) {(k+1/2)/q + r'(k)} - {(k+1/2)/q} | ≦ 3qE+3
を示せばよい。そのためには、区間 [k/q - E, (k+1)/q + E] が関数{x}の不連続点(つまり整数)を通過する k とそうでない k に Σ を分けて、
それぞれを評価すればよい。詳細は略(突然の飽き)□
0417132人目の素数さん
垢版 |
2018/11/25(日) 22:00:17.82ID:UuPttQIq
>>294
■@^2+CでもP1stは求められる

((n(n+1)/2)-1)^2+{4(n-1)^3+6(n-1)^2-4(n-1)-3+3(-1)^(n-1)}/48 

計算知能で@^2+Cを入力すると

P1st ={12n^4+28n^3-42n^2-52n-3(-1)^n+51}/48
0423132人目の素数さん
垢版 |
2018/11/26(月) 00:47:04.32ID:BZQrI/i+
>>416
ありがとう。
また考えてみます。
Fourier展開みたいな技使ってもダメっぽいね。
コツコツやるしかないのね……
0425132人目の素数さん
垢版 |
2018/11/26(月) 01:22:34.01ID:zUO0KBha
東京駅からのぞみ号で朝8時から9時に出発する(9時発も可)。

無作為に選んだ8時台の時間に出発ホームに到着したとすると平均の待ち時間は何分か?

以下が東京8時台発のぞみ号の時刻表である。

8:00 8:10 8:13 8:20 8:23 8:30 8:40 8:47 8:50 9:00
0426イナ ◆/7jUdUKiSM
垢版 |
2018/11/26(月) 03:00:27.06ID:s06YIHN9
(0+9+8+7+6+5+4+3+2+1+0+2+1+0+6+5+4+3+2+1+0+2+1+0+6+5+4+3+2+1+0+9+8+7+6+5+4+3+2+1+0+6+5+4+3+2+1+0+2+1+0+9+8+7+6+5+4+3+2+1+0)÷61
=207÷61
=3.39344262
≒3.4(分)
3分24秒
>>425
0428132人目の素数さん
垢版 |
2018/11/26(月) 07:29:11.92ID:zUO0KBha
>>425
Prelude> let tt = [0,10,13,20,23,30,40,47,50,60]
Prelude> let diff = zipWith (-) (tail tt) (init tt)
Prelude> sum (map (\x -> x^2/2) diff) / sum(diff)
3.95
0429132人目の素数さん
垢版 |
2018/11/26(月) 08:20:42.11ID:zUO0KBha
>>426
レスありがとうございます。
筆算、乙。
0〜60分を1分ごと、0.1分ごと0.01分、0.01分ごとにして計算すると

> mean(ct2wt(seq(0,60,by=1)))
[1] 3.393443
> mean(ct2wt(seq(0,60,by=0.1)))
[1] 3.893511
> mean(ct2wt(seq(0,60,by=0.01)))
[1] 3.944343
> mean(ct2wt(seq(0,60,by=0.001)))
[1] 3.949434

面積計算の値3.95分に収束するようです。

Rのソース
tt=c(0,10,13,20,23,30,40,47,50,60) # time table
ct2wt <- function(x){ # clock time to waiting time
n=length(tt)
if(x<=tt[1]){w8=tt[1]-x
}else{
for(i in 1:(n-1)){
if(tt[i]<=x & x<=tt[i+1]){
w8=(tt[i+1]-x)
break
}
}}
return(w8)
}
ct2wt=Vectorize(ct2wt)
0430132人目の素数さん
垢版 |
2018/11/26(月) 15:12:05.01ID:kjzRc5OI
分単位でちょうどの時間に着いた時は
待ち時間なしの確率が1/2になるというわけだな
0431132人目の素数さん
垢版 |
2018/11/26(月) 15:40:21.97ID:cCOlyEA2
ある駅のホームの1番線には1時間ごと、2番線には(1/2)時間ごと、3番線には(1/3)時間ごとに電車が来るようにダイヤを組みたい。
ランダムな時間に駅に着いたときの平均待ち時間を最小にするには、どのようにダイヤを組めば良いか。
ただし、何番線の電車に乗っても向かう方向や停車駅に違いは無いとする。
0432132人目の素数さん
垢版 |
2018/11/26(月) 16:20:05.15ID:kjzRc5OI
単純に考えて、以下ぐらいしか思いつかない

3番線 00 20 40
2番線 20/3 110/3
1番線 50 
0433132人目の素数さん
垢版 |
2018/11/26(月) 16:27:19.74ID:kjzRc5OI
>>432 訂正
最大15分はどうしても開くのかな?

3番線 00 20 40
2番線 05 35
1番線 50 
0435132人目の素数さん
垢版 |
2018/11/26(月) 17:05:53.16ID:kjzRc5OI
最小5分50秒 00 10 15 30 45 50 00
最大8分20秒 00 20 30 40 00
0436132人目の素数さん
垢版 |
2018/11/26(月) 17:53:39.06ID:yzjtacP4
>>433-434
正解
3番線を00,20,40で固定すれば、対称性から2番線は x,30+x (0≦x≦10) のみを考えればよくて、
x をどうとっても1番線を 50 とするのが最適解(のうちの1つ)であることがわかる。
(より長い空白のど真ん中に入れた方がより平均待ち時間を削減できるから)
0437132人目の素数さん
垢版 |
2018/11/26(月) 18:17:18.85ID:mGDYWVbl
いつもの顰蹙のプログラミング解

w8 <- function(xy,Print=FALSE){
x=xy[1];y=xy[2]
if(x<0|x>20|y<0|y>30)return(Inf)
tt=c(0,x,x+20,x+40,y,y+30,60)
tt=sort(tt)
d=diff(tt)
w=sum(d^2/2)/sum(d)
if(Print){
print(tt)
cat(sum(d^2/2),'/',sum(d))
}
return(w)
}
optim(par=c(0,0),w8,method='Nelder-Mead') # 最小値
> optim(par=c(0,0),w8,method='Nelder-Mead')
$`par`
[1] 9.999003 14.999925

$value
[1] 5.833333

w8(c(10,15),P=T)
> w8(c(10,15),P=T)
[1] 0 10 15 30 45 50 60
350 / 60[1] 5.833333

optim(par=c(0,0),w8,method='Nelder-Mead',control=list(fnscale=-1)) # 最大値
> optim(par=c(0,0),w8,method='Nelder-Mead',control=list(fnscale=-1))
$`par`
[1] 0 0

$value
[1] 8.333333

w8(c(0,0),P=T)
> w8(c(0,0),P=T)
[1] 0 0 0 20 30 40 60
500 / 60[1] 8.333333
0438132人目の素数さん
垢版 |
2018/11/26(月) 18:31:44.90ID:mGDYWVbl
細かいことを言えば
1番線 03
2番線 18 48
3番線 13 33 53
でもいいはず。
0440132人目の素数さん
垢版 |
2018/11/26(月) 20:08:04.25ID:mGDYWVbl
>>431
4番線は1時間に4本電車が来るとしてプログラムに解かせると

[[1]]
[1] 0

[[2]]
[1] 15 45

[[3]]
[1] 10 30 50

[[4]]
[1] 7.5 22.5 37.5 52.5

待ち時間の期待値は3.333333
0441132人目の素数さん
垢版 |
2018/11/26(月) 20:47:10.08ID:yzjtacP4
>>440
おつ n(≧2)番線に 60*(2k-1)/2n (k=1,2,…,n) 分に来させれば良いと予想したくなるなw ファレイ数列みたいだ
しかしそれだと30分に到着する番線が大量発生して非効率そうだ
0442132人目の素数さん
垢版 |
2018/11/26(月) 21:42:27.22ID:mGDYWVbl
>>441
そんなに単純じゃないみたい。

> densha(c(15,10,7.5,6),P=T)
[[1]]
[1] 0

[[2]]
[1] 15 45

[[3]]
[1] 10 30 50

[[4]]
[1] 7.5 22.5 37.5 52.5

[[5]]
[1] 6 18 30 42 54

155 / 60
[1] 2.583333

よりも、こっちの方が平均待ち時間が少ない。

> densha(c(14,8,6,6),P=T)
[[1]]
[1] 0

[[2]]
[1] 14 44

[[3]]
[1] 8 28 48

[[4]]
[1] 6 21 36 51

[[5]]
[1] 6 18 30 42 54

150 / 60
[1] 2.5
0443132人目の素数さん
垢版 |
2018/11/26(月) 22:33:06.60ID:mGDYWVbl
>>442
こっちの方がもっと少なかった。
> densha(c(16,12,8,6),P=T)
[[1]]
[1] 0

[[2]]
[1] 16 46

[[3]]
[1] 12 32 52

[[4]]
[1] 8 23 38 53

[[5]]
[1] 6 18 30 42 54

148 / 60
[1] 2.466667

プログラムでの最適解はこんな値を返してきた。
> optim(par=30/2:5,densha)
$`par`
[1] 15.967750 11.935738 8.225689 5.999897
0444132人目の素数さん
垢版 |
2018/11/27(火) 11:33:32.61ID:b5HUeFcY
至急お願いします

異なる業種間で賃金格差が存在しているかどうかを調査するために、一部上場企業の金融業と製造業から総合職(入社5年目)の社員をそれぞれ100名ずつ無作為抽出したところ、平均賃金は金融業が600万円、製造業が570万円でした。

また、標本標準偏差はどちらも70万円でした。業種間で賃金格差が存在しているどうかを有意水準5%で検定するとき、その結果として正しいものを以下から選びなさい。

A. 金融業の方が賃金が高い

B. 製造業の方が賃金が高い

C. 金融業と製造業に賃金格差があるとは言えない

D. いずれでもない
0445132人目の素数さん
垢版 |
2018/11/27(火) 11:35:03.74ID:b5HUeFcY
某テレビ局のプロデューサーX氏は製作番組の視聴率の目標を20%と想定していました。500世帯を対象に調査をしたところ、平均視聴率は15%でした。X氏の製作番組は目標視聴率20%を達成したかどうかを有意水準5%で検定します。

真の視聴率をpとするとき、帰無仮説と対立仮説、および検定結果として正しい組合せを以下から選びなさい。

A. 帰無仮説p=0.20、対立仮説p<0.20、帰無仮説を採択

B. 帰無仮説p=0.15、対立仮説p<0.15、帰無仮説を採択

C. 帰無仮説p=0.20、対立仮説p<0.20、帰無仮説を棄却

D. 帰無仮説p=0.15、対立仮説p<0.15、帰無仮説を棄却

E. いずれでもない

サイコロを300回投げたところ、そのうち60回で1が出ました。このサイコロに歪みがないかを有意水準5%で検定します。

サイコロを1回投げたとき1が出る確率をpとするとき、帰無仮説と対立仮説、および検定結果として正しい組合せを以下から選びなさい。

A. 帰無仮説p=1/6、対立仮説p≠1/6、帰無仮説を棄却

B. 帰無仮説p>1/6、対立仮説p=1/6、帰無仮説を棄却

C. 帰無仮説p<1/6、対立仮説p>1/6、帰無仮説を採択

D. 帰無仮説p=1/6、対立仮説p≠1/6、帰無仮説を採択

E. いずれでもない
0447132人目の素数さん
垢版 |
2018/11/27(火) 14:19:17.27ID:Lp6axtvL
>>444
> T.test=function(n1,n2,m1,m2,sd1,sd2){
+ SE12=sqrt((1/n1+1/n2)*((n1-1)*sd1^2+(n2-1)*sd2^2)/((n1-1)+(n2-1)))
+ T=(m1-m2)/SE12
+ 2*pt(abs(T),n1-1+n2-1,lower.tail = FALSE)
+
+ }
> T.test(100,100,600,570,70,70)
[1] 0.002767864
0448132人目の素数さん
垢版 |
2018/11/27(火) 14:22:23.16ID:Lp6axtvL
>>444
> kinyu=scale(rnorm(100))*70+600
> seizo=scale(rnorm(100))*70+570
> t.test(kinyu,seizo,var.equal = TRUE)

Two Sample t-test

data: kinyu and seizo
t = 3.0305, df = 198, p-value = 0.002768
alternative hypothesis: true difference in means is not equal to 0
95 percent confidence interval:
10.47802 49.52198
sample estimates:
mean of x mean of y
600 570

> t.test(kinyu,seizo,var.equal = TRUE,alt='greater')

Two Sample t-test

data: kinyu and seizo
t = 3.0305, df = 198, p-value = 0.001384
alternative hypothesis: true difference in means is greater than 0
95 percent confidence interval:
13.64024 Inf
sample estimates:
mean of x mean of y
600 570

答は A
0449132人目の素数さん
垢版 |
2018/11/27(火) 14:26:13.78ID:Lp6axtvL
>>445
前半
> binom.test(0.15*500,500,0.20,alternative = 'less')

Exact binomial test

data: 0.15 * 500 and 500
number of successes = 75, number of trials = 500, p-value = 0.002383
alternative hypothesis: true probability of success is less than 0.2
95 percent confidence interval:
0.0000000 0.1788032
sample estimates:
probability of success
0.15
答はC
0450132人目の素数さん
垢版 |
2018/11/27(火) 14:29:13.62ID:Lp6axtvL
>>445
後半
> binom.test(60,300,1/6)

Exact binomial test

data: 60 and 300
number of successes = 60, number of trials = 300, p-value = 0.1216
alternative hypothesis: true probability of success is not equal to 0.1666667
95 percent confidence interval:
0.1562313 0.2498044
sample estimates:
probability of success
0.2
帰無仮説は棄却できない。

帰無仮説を不採択だから、答はE
 
0455132人目の素数さん
垢版 |
2018/11/27(火) 23:44:32.45ID:vRyR0fY+
y=f(x) (0≦x≦a, f(x)≦0, f(0) = f(a) = 0)であらわされる滑り台上をボールを転がす運動を考える。
ただし、摩擦、空気抵抗などは無視してボールは重力とすべり台の面からの垂直抗力のみをうけて運動するものとする。
(0,0)にボールを静かにおいて滑り台を転がせ、(a,0)まで運動させるとき、その所要時間が最小となる曲線はなにか?
また東京ー大阪間の距離を400km、重力加速度を9.8m/s^2としたときの所要時間はおよそいくらか?

とある数学読み物で見つけた記事より。
当方答えのみ知っております。
導出の方法など知らないのでそれは自己採点でおながいしますw
0456132人目の素数さん
垢版 |
2018/11/28(水) 11:39:19.45ID:g3KJxOzn
有名問題だが、地球を球体とした場合はサイクロイドにはなんないと思うぞい
0457132人目の素数さん
垢版 |
2018/11/28(水) 12:33:05.86ID:Ra/jVdM5
>>456
そこは気になったんですが元記事通りの文章で。
今は400kmは地球が球状であることは無視でおながいします。
0458132人目の素数さん
垢版 |
2018/11/28(水) 19:18:07.40ID:aJc4zmU1
>>380
最後の手段でズルしちゃうけど、剰余項は少なくとも
O(n^(131/416+ε)) (ε>0は任意)まで改善できるはず。
ただし dirichlet divisor problem を経由するのでとてもズルイw

まず、nの約数の総和をd(n)とするとき、
Dirichlet hyperbola method により、1以上の実数xに対して

Σ(1≦n≦x)d(n)=2Σ(1≦n≦x^{1/2})[x/n]−[x^{1/2}]^2

が成り立つ。

Σ(1≦n≦x^{1/2})[x/n]=Σ(1≦n≦x^{1/2})(x/n)−Σ(1≦n≦x^{1/2}){x/n}

なので、お目当ての Σ(1≦n≦x^{1/2}){x/n} が出てきて

2Σ(1≦n≦x^{1/2}){x/n}=2xΣ(1≦n≦x^{1/2})(1/n)−[x^{1/2}]^2−Σ(1≦n≦x)d(n)

と表せる。
0459132人目の素数さん
垢版 |
2018/11/28(水) 19:22:38.31ID:aJc4zmU1
次に、Σ(1≦n≦x^{1/2})(1/n) に対してオイラー・マクローリンの公式

Σ(a≦n≦x)f(n)
=∫(a,x)f(t)dt+(f(a)+(1−2{x})f(x))/2+B_2({x})f'(x)−B_2(0)f'(a)−∫(a,x)B_2({t})f''(t)dt
(ただしaは整数でxは実数でB_2(t)=(t^2/2)−(t/2)+(1/12))

を適用して、γ=lim(x→∞)(Σ(a≦n≦x)(1/n)−log(x)) と合わせて計算すれば、

2Σ(1≦n≦x^{1/2}){x/n}=x^{1/2}+xlog(x)+(2γ−1)x+O(1)−Σ(1≦n≦x)d(n)

となるはず。一方で、dirichlet divisor problem の最新の結果では

Σ(1≦n≦x)d(n)=xlog(x)+(2γ−1)x+O(x^θ), infθ≦131/416

となっているので、

2Σ(1≦n≦x^{1/2}){x/n}=x^{1/2}+O(1)−O(x^θ)=x^{1/2}+O(x^θ), infθ≦131/416

となって、目標の剰余項 O(n^(131/416+ε)) (ε>0は任意)になる。
0460132人目の素数さん
垢版 |
2018/11/28(水) 19:26:03.82ID:aJc4zmU1
ちなみに、逆の計算もできて、

Σ(1≦n≦x^{1/2}){x/n}=(1/2)x^{1/2}+O(x^α)

が成り立つなら

Σ(1≦n≦x)d(n)=xlog(x)+(2γ−1)x+O(x^α)

も成り立つ。よって、>>380の剰余項はdirichlet divisor problemの剰余項と
本質的に同じものになる。そして、dirichlet divisor problemの剰余項では
infθ≧1/4 が成り立つらしいので、>>380の剰余項もそこまでが限界になる。
ぴったり infθ=1/4 が成り立つかは未解決問題らしい。
0461132人目の素数さん
垢版 |
2018/11/28(水) 19:29:43.00ID:aJc4zmU1
dirichlet divisor problemにおけるO(x^θ)の変遷はwikipediaに年表が載っていて、
最初にO(x^{1/2})があって、次の更新は1904年で、このときはO(x^{1/3}log x)が得られている。
一応、この時点で>>411よりも精度が高い。該当する論文は、よく分からないけどたぶんこれ。

sur une fonction transcendante et ses applications a la sommation de quelques series
sur une fonction transcendante et ses applications a la sommation de quelques series(suite)

どちらも非常に長い上に、ゼータ関数を経由していて何をやっているのか全然分からん。
最初の更新がこんな高度なレベルってのが信じられんw

別の見方をすると、>>380>>411ならO(x^{1/3}log x)には及ばないものの非自明な更新が
初等的に得られるということでもある。
0462132人目の素数さん
垢版 |
2018/11/28(水) 20:32:26.13ID:r8zTzMor
問題(第1種情報処理技術者試験・平成元年度春期午前問17を改題)

ある医院では、患者が平均10分間隔でポアソン到着にしたがって訪ねてくることがわかった。
医者は1人であり、1人の患者の診断及び処方にかかる時間は平均8分の指数分布であった。

設問1 患者が待ち始めてから、診断を受け始めるまでの「平均待ち時間」を求めなさい。
0467132人目の素数さん
垢版 |
2018/11/29(木) 00:12:29.89ID:KKV0PTEX
となると今度は>>380の出題者さんの難しい論文を引用しないでも済む方法が聞きたい。
おながいします。
0468132人目の素数さん
垢版 |
2018/11/29(木) 05:26:25.15ID:j6Cen0SX
>>458-461
ふぁっ!? そんなもんがあったのか…勉強になるわ、後でじっくり読もう
一応 >>380 の解答載せるけど、だいぶゴリッゴリの力ずくでそれ程有意義とも思えんけどまあ許してちょ…
あとリクエストにおこたえしてガバガバながら一番最後以外の細かい係数も出してみた

実数 δ, δ', β, c を 0<δ/2≦β<δ<δ'<1, 3δ'≦1+δ, 0<c≦δ-β を満たすものとして任意に固定する。
(パラメータを最適化したら δ'=1/3, δ=1/6, β=c=1/12 となるから先に代入して読んでいっても可)
n を正の整数とする。各正の整数 r≦n^(δ'-δ) に対して集合 J_r を
J_r := {mは0以上の整数| r ≦ 1 + m/(n^δ) < 1+r }
と定める。また、各 m∈J_r に対して集合 I_m を
I_m := {kは2以上の整数| 1+m/(n^δ) ≦ n/(k(k-1)) < 1+(m+1)/(n^δ)}
と定める。以降、r と m は上に示された範囲のみを動くものとする。

まずは I_m の元の個数 |I_m| を評価したい。I_m の条件式を整理すると
(1/2)*(1+√(1+4n*(1+(m+1)/(n^δ))^(-1))) < k ≦ (1/2)*(1+√(1+4n*(1+m/(n^δ))^(-1)))
となるから、この範囲の k の個数を考えると
|I_m| ≦ 1 + √(n*(1+m/(n^δ))^(-1)) - √(n*(1+(m+1)/(n^δ))^(-1))
≦ 1 + (√n)*(1/(n^δ))/(2*(1+m/(n^δ))^(3/2))
≦ 1 + (1/2) * n^(1/2 - δ) * r^(-3/2)
≦ (3/2)*n^(1/2-δ)*r^(-3/2) (∵ 1/2-δ-(3/2)(δ'-δ)≧0 )
=: L_r.
0469132人目の素数さん
垢版 |
2018/11/29(木) 05:28:21.92ID:j6Cen0SX
ディリクレのディオファントス近似定理から、各 m について
|m/(n^δ) - p/q| ≦ 2/(qn^β) ((p,q)=1, 1≦q≦n^β) …(☆)
を満たす整数 p,q が存在する。
また、逆に q<n^β を固定したときに(☆)を満たす p が存在するような m∈J_r の個数は、
≦ φ(q)*(2+2(n^δ)/(qn^β)) (φはオイラーのトーシェント関数)
≦ 2(q+n^(δ-β)).
また、そのような m について
n/k = n/k_0 + (1+m/(n^δ))(k-k_0) + ψ~(k) (for∀k∈I_m. ただし k_0:=min(I_m))
とおくと |ψ~(k+1)-ψ~(k)|≦n^(-δ) が成り立つが、これより
n/k = n/k_0 + (1+(p/q))(k-k_0) + ψ(k) (for∀k∈I_m)
とおくと |ψ(k+1)-ψ(k)|≦n^(-δ)+2/(qn^β) が成り立つ。ゆえに、>>416 の補題から
Σ_(k∈I_m) ({n/k}-1/2)
≦ [|I_m|/q]( 3q*(q/(n^δ) + 2/(n^β)) + 3 ) + q (∵q個の連続する整数ごとに和をとる。あぶれる項はq以下)
≦ 3*L_r*(q/(n^δ) + 3/q) + q.

ここで、J_r を2つの集合
J'_r := {m∈J_r| q≦n^c ととれる },
J''_r := {m∈J_r| q≦n^c ととれない }
に分割すると、
|Σ_(m∈J'_r) Σ_(k∈I_m) ({n/k}-1/2)| ({x}はxの小数部分)
≦ Σ_(q=1,[n^c]) 2(q+n^(δ-β))*(3*L_r*(q/(n^δ)+3/q)+q)
≦ Σ_(q=1,[n^c]) 4(n^(δ-β))*(12(L_r)/q + q) (∵c≦δ-β)
≦ 48*L_r*n^(δ-β)*(1+clogn) + 4n^(δ-β+2c).
また、
|Σ_(m∈J''_r) Σ_(k∈I_m) ({n/k}-1/2)|
≦ Σ_(m∈J''_r) (12(L_r)/(n^c) + n^β)
≦ 12*L_r*n^(δ-c)+n^(δ+β).
以上から、
|Σ_(m∈J_r) Σ_(k∈I_m) ({n/k}-1/2)|
≦ (上二つのJ'_r,J''_rについての評価式の和).
0470132人目の素数さん
垢版 |
2018/11/29(木) 05:33:09.64ID:j6Cen0SX
この評価式を r=1,2,…,[n^(δ'-δ)] について足し合わせれば、
Σ_(r=1,[n^(δ'-δ)]) Σ_(m∈J_r) Σ_(k∈I_m) ({n/k}-1/2)
≦ 72ζ(3/2)*n^(1/2-β)*(1+clogn) + 4n^(δ'-β+2c) + 18ζ(3/2)*n^(1/2-c) + n^(δ'+β).
また、k=1,…,[√n] のうち、このΣで k の項が足されていないようなものの個数は
≦ 1+n^((1/2)(1+δ-δ'))
であることがわかる。ゆえに、
|Σ_(k=1,[√n]) {k/n}-(1/2)| ≦ (上二つの評価式の和)+1. (最後の +1 は整数 k>[√n] の項で足された分)
δ'=1/3, δ=1/6, β=c=1/12 と定めることにより |Σ_(k=1,[√n]) ({n/k}-1/2)|=O(n^(5/12)*logn) がわかる。□
0471132人目の素数さん
垢版 |
2018/11/29(木) 06:22:50.32ID:HF9YHqBY
>>468-470
乙です。

こっちは、剰余項を考えないバージョンの

lim(x→∞)x^{−1/2}Σ(k≦x^{1/2}){x/k}=1/2

が一般的な形で証明できたかもしれない。使うのは>>413で、m∈Z−{0}のときは

lim(x→∞)x^{−1/2}Σ(k≦x^{1/2})e^{2πimx/k}=lim(x→∞)x^{−1/2}O(x^{1/3})=0

が成り立ち、m=0 のときは

lim(x→∞)x^{−1/2}Σ(k≦x^{1/2})e^{2πimx/k}=1

が成り立つので、あとは Weyl's criterion の手法を機械的に真似すればいい。
こうすると、より一般的に、リーマン可積分な任意の f:[0,1]→R に対して

lim(x→∞)x^{−1/2}Σ(k≦x^{1/2})f({x/k})=∫(0,1)f(t)dt

が示せるはず。例のごとく間違ってる可能性もあるので、詳しくは書かないけどw
0472132人目の素数さん
垢版 |
2018/11/29(木) 17:20:21.53ID:eeR3aEhF
有名なのできっと既出だとは思いますが、すごくお気に入りなのでぜひ紹介させてください。

ドラゴンの目の色は、赤か緑のどちらかである。
ドラゴンは不死身だが、もし自分の目の色が緑だと知ってしまった場合、そのドラゴンは日付が変わると同時にトカゲに変身してしまう。
だからドラゴンたちは決して鏡を見ないし、お互いの目の色の話もしないのだ。

ところで、ある島にはドラゴンが100頭住んでいて、その目はすべて緑色である。
ここを訪れたある旅人は去り際に、見送りに来た100頭のドラゴン全員に聞こえるようにこう言った。
「この島には少なくとも一頭、緑の目のドラゴンがいる」
このあと、この島で何日後に、何が起こるだろうか。

ただしドラゴンたちは十分に聡明で、もしあるドラゴンがトカゲに変身したとき次の日にはそのことを皆が知ることになるものとし、
100頭のドラゴンは互いに顔見知りであり自身以外の99頭が緑の目をもつことを知っているものとし、
この島にドラゴンは出入りしないものとする。
0473132人目の素数さん
垢版 |
2018/11/29(木) 18:58:12.47ID:+9fHdpLo
不思議だなこれ

寝る。
次の日n頭中の誰もトカゲになってないなら、
どうn-1頭をとってもその中に少なくとも1頭緑目がいることになる。
適当にn-1頭が集まって「この中に少なくとも1頭緑目がいる」と宣言する。しなくても皆気付いてるけど。
寝る。

次の日集まったn-1頭中の誰もトカゲになってないなら、
その中からどうn-2頭をとってもうち少なくとも1頭は緑目がいることになる。
適当にn-2頭が集まって「この中に少なくとも1頭緑目がいる」と宣言する。しなくても皆気付いてるけど。

以下略

一方で「島には少なくとも1頭緑目がいる」という旅人の言葉は
全てのドラゴンにとって既知の事実だし、
それが全てのドラゴンにとって既知の事実だということも皆が知っているし、
それが全てのドラゴンにとって既知の事実だということも皆が知っているということも皆が知っている。
旅人の言葉は何の情報を与えてドラゴンの推論エンジンを始動したのか?
0474132人目の素数さん
垢版 |
2018/11/29(木) 21:03:17.45ID:HF9YHqBY
>>471の方針のもとで剰余項ありのケースを考えたら、
C_0^∞関数で近似して普通に剰余項が出たっぽい。

f:[0,1]→Rは2階微分可能で、f '' ∈L^2 が成り立つとする。
このとき、α=1/3に対して

Σ(k≦x^{1/2})f({x/k})=x^{1/2}∫(0,1)f(t)dt+O(x^{(2/5)α+(3/10)})

が成り立つ。

残念ながら、O(x^{5/12}log(x))よりは精度が悪いw
0475132人目の素数さん
垢版 |
2018/11/30(金) 00:36:11.15ID:rQpMmAJV
>>472
類題をどっかで見たようなと思ったけど、これと同じかな?
こっちのほうがイメージしやすそう

100組の夫婦が暮らす村がある。
100人の夫は全員他の家の妻と不倫をしている。
この村のルールでは、妻が夫の不倫を知ったらその夜に夫を殺さなければならない。
妻は他の家の夫が不倫をしていることを必ず見抜くことができるが、
自分の夫の不倫には絶対に気がつかない。
妻たちは全員論理的な思考ができて、かつ他の妻に夫が不倫をしていることを決して言わない。
村を訪れた旅人が「少なくとも一人の夫が不倫をしている」と言った。
その後どうなるか。
0476132人目の素数さん
垢版 |
2018/11/30(金) 03:38:09.32ID:6JPMu1Zv
見直したけど大丈夫っぽいので、>>471の詳細を書いてみます。
剰余項つきの>>474はまた今度。

f:[0,1]→Rに対して

S_u(f)=limsup(x→∞)x^{−1/2}Σ(k≦x^{1/2})f({x/k})
S_d(f)=liminf(x→∞)x^{−1/2}Σ(k≦x^{1/2})f({x/k})

と置く。lim が実数として存在するときには

S(f)=lim(x→∞)x^{−1/2}Σ(k≦x^{1/2})f({x/k})

とも置く。S(f)が存在することと、
S_u(f)=S_d(f)∈R が成り立つことは同値であり、
そのときS(f)=S_u(f)=S_d(f)∈Rが成り立つ。
0477132人目の素数さん
垢版 |
2018/11/30(金) 03:41:32.79ID:6JPMu1Zv
fがリーマン可積分なら、S(f)が存在してS(f)=∫(0,1)f(t)dtが成り立つことを示したい。
Weyl's criterion の真似をする(f(x)が特殊な形から出発して一般化していく)。

f(x)=sin(2πkx) (k∈Z) のときは、もしk≠0なら、
>>413により S(f)=0=∫(0,1) f(t)dt である。
また、k=0のときは S(f)=0=∫(0,1) f(t)dt である。よって、この場合は成立。

f(x)=cos(2πkx) (k∈Z) のときは、もしk≠0なら、>>413と同じように計算して、
S(f)=0=∫(0,1) f(t)dt である。また、k=0のときは
S(f)=1=∫(0,1) f(t)dt である。よって、この場合も成立。

S(f)の線形性により、有限個の sin(2πkx), cos(2πkx) (k∈Z) の
R係数の線型結合で表されるg(x)に対しても、S(g)が存在してS(g)=∫(0,1)g(t)dtとなる。
0478132人目の素数さん
垢版 |
2018/11/30(金) 03:44:54.09ID:6JPMu1Zv
次に、f:[0,1]→R が連続で f(0)=f(1) を満たすときを考える。
この場合、フーリエ展開の理論から、任意のε>0に対して、有限個の
sin(2πkx), cos(2πkx) (k∈Z) のR係数の線型結合で表されるg(x)であって

sup(x∈[0,1])|f(x)−g(x)|<ε

を満たすものが取れる。S(g)=∫(0,1)g(t)dt により、
S_u(g)=S_d(g)=∫(0,1)g(t)dt なので、g−ε≦f≦g+ε (各点)
と合わせて

S_u(f)≦S_u(g+ε)=S_u(g)+ε=∫(0,1) g(t)dt+ε

S_d(f)≧S_d(g−ε)=S_d(g)−ε=∫(0,1) g(t)dt−ε

S_u(f)≧S_d(f)

となる。また、|∫(0,1)g(t)dt−∫(0,1)f(t)dt|≦εである。よって、
∫(0,1)f(t)dt−2ε≦S_d(f)≦S_u(f)≦∫(0,1)f(t)dt+2ε となる。
ε>0は任意だったから、この場合も成立。
0479132人目の素数さん
垢版 |
2018/11/30(金) 03:48:44.97ID:6JPMu1Zv
次に、A⊂[0,1] に対して、1_A:[0,1]→R を 1_A(x)=1 (x∈A), 0 (x∈[0,1]−A) と定義する。
ここでは、I⊂[0,1] は開区間または閉区間または半開区間とする。f=1_I のときを考える。
任意のε>0に対して、折れ線で構成される連続関数 f_1,f_2:[0,1]→R であって

f_k(0)=f_k(1) (k=1,2),
f_1≦f≦f_2 (各点の意味),
∫(0,1)(f_2(t)−f_1(t))dt<ε

を満たすものが簡単に構成できる。S_u(f_k)=S_d(f_k)=∫(0,1)f_k(t)dt なので、

S_u(f)≦S_u(f_2)=∫(0,1)f_2(t)dt≦∫(0,1)f_1(t)dt+ε≦∫(0,1)f(t)dt+ε

S_d(f)≧S_d(f_1)=∫(0,1)f_1(t)dt≧−ε+∫(0,1)f_2(t)dt≧−ε+∫(0,1)f(t)dt

S_u(f)≧S_d(f)

により、−ε+∫(0,1)f(t)dt≦S_d(f)≦S_u(f)≦∫(0,1)f(t)dt+ε となる。
ε>0は任意だったから、この場合も成立。
0480132人目の素数さん
垢版 |
2018/11/30(金) 03:53:36.23ID:6JPMu1Zv
よって、有限個の 1_I (I⊂[0,1]は開区間または閉区間または半開区間)の
R係数の線型結合で表されるg(x)に対しても、S(g)が存在してS(g)=∫(0,1)g(t)dtとなる。
すなわち、任意の階段関数 g:[0,1]→R に対して、S(g)が存在してS(g)=∫(0,1)g(t)dtとなる。

最後に、f:[0,1]→R はリーマン可積分とする。リーマン積分の性質により、
任意のε>0に対して、ある階段関数 f_1,f_2:[0,1]→R が存在して、

f_1≦f≦f_2 (各点の意味),
∫(0,1)(f_2(t)−f_1(t))dt<ε

が成り立つ。よって、>>479と同じ計算で S(f)=∫(0,1)f(t)dt となる。

特にf(x)=xとして、lim(x→∞)x^{−1/2}Σ(k≦x^{1/2}){x/k}=1/2 が成り立つ。□
0481132人目の素数さん
垢版 |
2018/11/30(金) 15:38:06.60ID:rQpMmAJV
>>473
解説を読んでもすぐにはピンとこないよね

>99日後に分かる
>「少なくとも99頭のドラゴンが緑の目をしていること」はみんな知っているよ
>これが「旅人がドラゴンに与えた情報」です
0482132人目の素数さん
垢版 |
2018/11/30(金) 15:56:26.16ID:FBGG17Gf
>>475
今までに
他の家で誰も殺されていないことを全ての妻が知っている、のが前提だよね?
0483132人目の素数さん
垢版 |
2018/11/30(金) 17:09:43.19ID:DJTATwgI
>>480
なるほど、求める性質が成り立ってくれるとわかる関数のクラスをどんどん"扱いやすいもの"にしていくという方法か

そうすると、2以上のαに対して S'(f)=lim_(x→∞) x^(1/α) Σ_(k=1,[x^(1/α)]) f(k) と定義した時に
同じ議論が成り立つかどうかも、最初の sinkx, coskx についての箇所さえチェックすればいいことになるんかな
まあ、これを考えるとなると、一般の多項式についてガウス和チックなこと考えなきゃいけなくて
かなり面倒になることが予想される…w ここも興味ある有志にお任せするとしようか
0484132人目の素数さん
垢版 |
2018/11/30(金) 17:16:10.09ID:REFDuxRq
緑目ドラゴン3頭が島で暮らしている。
旅人が「少なくとも1頭は緑目」と言う。

ドラゴンAは考える
「我以下の2頭は緑目」
「論理的に考えて3頭中どの2頭をとっても少なくとも1頭緑目がいる」
「我とドラゴンBのうち少なくとも1頭は緑目」
「見てわかるようにドラゴンBは緑目」

「だがドラゴンBから見ると『論理的に考えて3頭中どの2頭をとっても少なくとも1頭緑目がいる』は成り立たない」
「我が赤目の場合、ドラゴンBからは緑目は1頭しか見えていないのだからな」

「とすれば論理的に考えて」
「皆が緑目であるか、」
「自分だけが赤目であるか、」
「のいずれかであるな」

おしまい。
0485132人目の素数さん
垢版 |
2018/11/30(金) 17:28:59.35ID:REFDuxRq
>>484
>「我以下の2頭は緑目」

我以外、の書き間違えでした。
自分の目の色はわからないので自分と他人とでは情報に対称性がないわけです。
よって「自分から見るとAが帰結されるから他人もAと帰結するはず」とは言えない。

「少なくとも1頭緑目がいる」という既知の情報からは
「自分だけが赤目か皆が緑目のいずれかである」
という既知の事実以上の推論は不可能でした。
0486132人目の素数さん
垢版 |
2018/11/30(金) 19:07:58.32ID:rQpMmAJV
@ 既知の情報だから何も起こらない説
A 外部発言がなくても全員が変身する説

信じるか信じないかは、あなた次第
0487132人目の素数さん
垢版 |
2018/11/30(金) 19:31:32.14ID:REFDuxRq
>>473
>適当にn-1頭が集まって「この中に少なくとも1頭緑目がいる」と宣言する。しなくても皆気付いてるけど。

この「皆気付いてるけど」が少なくとも n=3 のときは成り立たないんだよね
0488132人目の素数さん
垢版 |
2018/11/30(金) 23:57:51.84ID:FDb9lbGr
3匹の場合でこうかな

1日目のA:
1)我(A)が赤目だと仮定しよう。そのとき、Bは赤目(A)と緑目(C)が見えている。
Bは「a)我(B)が赤目だと仮定しよう。そのとき、Cは赤目(A,B)だけが見えている。
ならば、Cは『緑目が少なくとも一頭いるのだから、我(C)が緑目だ』と考える。
b)我(B)が緑目だと仮定しよう。そのとき、Cは赤目(A)と緑目(B)が見えている。
ならば、Cは『緑目が少なくとも一頭いることからは、我(C)が緑目だとはいえない』と考える。
a)b)から、明日になってCがトカゲになっていたら我(B)は赤目、ならなかったら我(B)は緑目だ」と考える。
2)我(A)が緑目だと仮定しよう。そのとき、Bは緑目(A,C)が見えている。
Bは「緑目が少なくとも一頭いることからは、我(B)が緑目だとはいえない」と考える。

2日目のA:
Cはトカゲにならなかった。
1)我(A)が赤目だと仮定しよう。Bは「我(B)は緑目だ」と考える。
2)我(A)が緑目だと仮定しよう。Bは「我(B)は緑目だとはいえない」と考える。
1)2)から、明日になってBがトカゲになっていたら我(A)は赤目、ならなかったら我(A)は緑目だ。

3日目のA:
Bはトカゲにならなかった。
我(A)は緑目だ。
0490132人目の素数さん
垢版 |
2018/12/01(土) 00:25:39.26ID:subMGBQf
>>488
そうするとこの場合はどうなるんだろう。

島に3頭の緑目ドラゴンがいた。
ドラゴンAはある日ふと思った。
「この島には少なくとも1頭の緑目がいる」
0491132人目の素数さん
垢版 |
2018/12/01(土) 00:26:50.87ID:subMGBQf
もちろんBCも(目で見て)それ(少なくとも1頭緑目がいることを)を知っている、とAは考えている。
0492132人目の素数さん
垢版 |
2018/12/01(土) 00:37:15.89ID:UzK/K8Af
>>490
その場合はAの想像の中のBの想像の中のCは「緑目が少なくとも一頭いる」ことは知らない。
ABCが「緑目が少なくとも一頭いる」という知識を共有してないといけない。
0493132人目の素数さん
垢版 |
2018/12/01(土) 00:50:34.34ID:subMGBQf
>>492
1行目はわかるんだけれども、2行目、
旅人なしでも
少なくとも1頭はいるという知識は共有しているし
共有しているという知識も共有していますよね。
0494132人目の素数さん
垢版 |
2018/12/01(土) 04:25:09.77ID:CULPj2qo
「あなた方の少なくとも1匹は緑目」
=「あなた以外のドラゴンがみな緑目でないなら、あなたは緑目」

「俺達の少なくとも1匹は緑目」からは後者の推論が成り立たない
0495132人目の素数さん
垢版 |
2018/12/01(土) 05:59:03.59ID:subMGBQf
「俺たちの少なくとも1頭は緑目」
=「俺以外の2頭が赤目なら俺は緑目」

形式的には全く問題なく推論が成り立つ気がするが
0497イナ ◆/7jUdUKiSM
垢版 |
2018/12/01(土) 09:36:39.36ID:Lf06XF8t
>>496
すぐに走りだす奴がいないということは三人の帽子は順不同の白白赤か白白白。

つまり白赤が見えてるか白白が見えてる。白赤が見えてるとき人は一般に自分が赤である確率が1/3だから走るはずだ。が、あとの二人が走らないということは白白が見えてるってこった!――そう思ってみんな走った。
0500132人目の素数さん
垢版 |
2018/12/01(土) 12:39:41.36ID:CULPj2qo
>>495
「俺たちの〜」は、(自分を除いて)少なくとも1頭が緑目、という限定主観情報
「あなた方の〜」は、(自分も含めて)少なくとも1頭が緑目、という客観情報

限定主観情報からは自分が緑目の推論は不可能
0501132人目の素数さん
垢版 |
2018/12/01(土) 16:52:23.81ID:CULPj2qo
「共有知識」で検索してたら関連パズルを発見

A,B,Cの3人が次のようなゲームをしました.
まずゲームマスターが3つの帽子に30, 2018, 2048と書き込み,この順にA,B,Cにかぶせます.
ゲームマスターは全員の前でこう告げます:
「帽子には全て正の整数が書かれています.それから,帽子に書かれた数のうち最大のものは,残りの2つの数の和になっています.」
3人は自分以外の人の帽子は見えますが,自分の帽子は見えません.
次にゲームマスターはA,B,Cの順に「自分の帽子に書かれた数がわかりますか?」と尋ねます.
全員が続けて「わからない」と答えたら,再びA,B,Cの順に同じことを尋ねます.
最初に自分の帽子に書かれた数字がわかるのは3人のうち誰で,それはその人への何回目の質問のときでしょうか.
ただし参加者全員は十分に明晰で,嘘をつくこともないとし,発言は全員に聞こえるものとします.
0502 【だん吉】
垢版 |
2018/12/01(土) 18:51:55.48ID:Lf06XF8t
師走、師走、師走!
師が走ると書いて師走!!
笑わば笑え、師走の気層のひかりの底を唾しはぎしりゆききする、俺はひとりの修羅なのだ! 前>>497

開運!!

参考文献
『春と修羅』宮沢賢治
0503 【小吉】
垢版 |
2018/12/01(土) 19:23:22.88ID:Lf06XF8t
ヾ∩∩〃
((`e`)やぁ
(っц)~きつい戦い
~ιγ) でした。
_υ`υ_ 二人とも走んなかったんで、こいつら白白見てる、俺白だ! 前>>502ピンときました。どのみち同時に走ったところで勝つ確率1/3なんで、それならスタート切って2/3失格じゃないなら一抜けできるなって。
0504132人目の素数さん
垢版 |
2018/12/01(土) 19:47:10.44ID:uyFzn6KH
>>501
とりあえず73ターン目のCは正解出来るみたいね。
これ以前に誰かが正解出来る可能性が否定出来てないのでダメだけど。

まず1ターン目誰もわからないので(a,a)の形が見えてる人がいないことが分かる。
次に2ターン目誰もわからないので(a,2a)の形が見えてる人がいないことが分かる。
次に3ターン目誰もわからないので(a,3a), (2a, 3a)の形が見えてる人がいないことが分かる。
……
と続けていくと遅くとも73ターン目のBの終了時点で(15a, 994a)の形が見えてる人がいないとわかり、Cはこの時点で(30,2018,1988) でないとわかるのでこのターンで正解出来る。
これが最初かどうかは不明。
0506132人目の素数さん
垢版 |
2018/12/01(土) 21:12:27.84ID:uyFzn6KH
もし(a,2a)と見えてる人がいたらその人にとって可能性は
(a,a,2a)か(a,2a,3a)しかないけど1ターン目で前者が否定されてるから(a,2a,3a)に確定するはず。
にも関わらず2ターン目で全員解答不能ならそのように見えてる人はいないとわかる。
0507132人目の素数さん
垢版 |
2018/12/01(土) 22:26:37.88ID:oLhduph7
ヒカキンの年収が10億超え!?明石家さんま・坂上忍も驚愕の総資産とは??
https://logtube.jp/variety/28439
【衝撃】ヒカキンの年収・月収を暴露!広告収入が15億円超え!?
https://nicotubers.com/yutuber/hikakin-nensyu-gessyu/
HIKAKIN(ヒカキン)の年収が14億円!?トップYouTuberになるまでの道のりは?
https://youtuberhyouron.com/hikakinnensyu/
ヒカキンの月収は1億円!読唇術でダウンタウンなうの坂上忍を検証!
https://mitarashi-highland.com/blog/fun/hikakin
なぜか観てしまう!!サバイバル系youtuberまとめ
http://tokyohitori.hatenablog.com/entry/2016/10/01/102830
あのPewDiePieがついに、初心YouTuber向けに「視聴回数」「チャンネル登録者数」を増やすコツを公開!
http://naototube.com/2017/08/14/for-new-youtubers/
27歳で年収8億円 女性ユーチューバー「リリー・シン」の生き方
https://headlines.yahoo.co.jp/article?a=20170802-00017174-forbes-bus_all
1年で何十億円も稼ぐ高収入ユーチューバー世界ランキングトップ10
https://gigazine.net/news/20151016-highest-paid-youtuber-2015/
おもちゃのレビューで年間12億円! 今、話題のYouTuberは6歳の男の子
https://www.businessinsider.jp/post-108355
彼女はいかにして750万人のファンがいるYouTubeスターとなったのか?
https://www.businessinsider.jp/post-242
1億円稼ぐ9歳のYouTuberがすごすぎる……アメリカで話題のEvanTubeHD
https://weekly.ascii.jp/elem/000/000/305/305548/
世界で最も稼ぐユーチューバー、2連覇の首位は年収17億円
https://forbesjapan.com/articles/detail/14474
0508132人目の素数さん
垢版 |
2018/12/01(土) 22:47:57.42ID:CULPj2qo
>>506
なるほど
ということは4ターン目終了時は以下が否定か
(a,4a)、(2a,5a)、(3a,4a)、(3a,5a)

まだ (15a,994a) の理屈は分かってないけど
1988=2*2*7*71 は関係あるかもね
0509132人目の素数さん
垢版 |
2018/12/01(土) 23:13:44.17ID:ix7k9ti5
昔家庭教師だったけど、数年前に学生に出された問題がわからない。
曰く数学オリンピックに入賞した奴が考えた問題だそうで、それの回答をせがんできたが自分には無理だった
今でも悔しいので、誰か答えがわかったら

多角形の対角線(以下A)をすべて結び、対角線同士の交点をXとする。またXから、他のXもしくは多角形の頂点へと線を結び(以下B)、B同士、またはBとAが交わった点をYとする。
またYから、他のY、もしくはX・多角形の頂点へと線を結び、それらの線同士、またはそれらの線とB、Aが交わった点をZとする。
以下この操作を無限に繰り返すことができない多角形をMとする。

@Mに該当する六角形を1つ図示せよ
AMは七角形以上の多角形には存在しないことを証明せよ

@は分かった
Aは無理だった
0511132人目の素数さん
垢版 |
2018/12/01(土) 23:38:40.23ID:ix7k9ti5
>>510
ヒントでいいなら、まず無限に上の操作を繰り返せない五角形を考えてみたらいいと思う
六角形はその要領で考えたら自然と出来るよ
0512132人目の素数さん
垢版 |
2018/12/02(日) 00:10:17.97ID:qrkUsr2r
>>511
いや、問題文の意味がわからない。
>>509の文章からは今まで獲得した点集合から4点ABCDを線分AB、CDを選んでその交点を新たに追加していいように思える。
でもそのルールだとどんな凸5角形からスタートしても無限に交点を生成できる。
たとえばABCDEを凸5角形でこの順にならんでるとしてAC、BDの交点をX[1]、X[1]E、ADの交点をY[2]、
以下AC、BY[i-1]の交点をX[i]、AD、X[i]Eの交点をY[i]とえらんでいけば、X[i]はAC上のすべて異なる点になってしまう。
なんかこの作業が禁止される禁止則がないとだめだけど>>509の文章ではそれがなにか伝わってこない。
なので6角形でMに属する図形の例あげてもらえれば伝わってくるのではないかと。
0513132人目の素数さん
垢版 |
2018/12/02(日) 01:06:23.43ID:fdTrITVn
>>512
五角形は長方形の対角線の交点に頂点が来るようにしたらいけるよ
それ以上結びようがない
0515132人目の素数さん
垢版 |
2018/12/02(日) 01:15:55.59ID:DwQ9ijI8
あ、ちがうな。

>五角形は長方形の対角線の交点に頂点が来るようにしたらいけるよ

これで意味わかった。
0516132人目の素数さん
垢版 |
2018/12/02(日) 01:18:27.34ID:B+C2Ix9O
>>513
もう一度確認。
凸でなくてもいい(180°超えるのは可)
だけどピッタリ180°は不可で桶?
でないと△ABCのBC上に4点とればMになっちゃうもんね?
0517132人目の素数さん
垢版 |
2018/12/02(日) 01:30:04.51ID:/eJ4uY04
>>513
だめだ。まだ不明点あるや。
凸でなくていいならたとえば六角形ABCDEFで∠Aが180°超える場合、対角線BFは6角形の外にでるけど、たとえばこのBFと直線ACの交点を追加するのはありなん?
それともあくまで “辺”、”対角線” はその延長を含まず追加される点は常にもとのPolygonの中に限定されるの?
0519132人目の素数さん
垢版 |
2018/12/02(日) 01:49:02.50ID:Dx2sd3hu
>>518
(1)の6角形の6点は全てことならないとだめなん?
たとえば長方形ABCDの交点をXとして
ABXCDXを6活計とみなすのはあり?
0521132人目の素数さん
垢版 |
2018/12/02(日) 01:56:45.89ID:36l5kxLk
>>520
これはなしなのか…
だと(1)の方がさっぱりわからん。
なんか可能性みおとしてるのかな。
凸包はやっぱり4角形?
0522132人目の素数さん
垢版 |
2018/12/02(日) 02:09:34.62ID:B+C2Ix9O
>>520
以下の解釈であってる?

Pを平面上の凸とは限らない多角形とする。
ただし内角は180°だけは禁止し、その周は単純閉曲線(S^1と同相)とする。
帰納的にF[i]を定める。

F[1] = Pの頂点集合。
F[i+1] = { l と mの交点 | l、m は F[1]∪…∪F[i] から任意に選んだ2点を結ぶ直線}

(1) #P = 6、#∪F[i] < ∞ をみたすPが存在することを示せ。
(2) #P ≧ 7、#∪F[i] < ∞ となるPは存在しないことを示せ。
0523132人目の素数さん
垢版 |
2018/12/02(日) 02:44:44.87ID:D2g2qj2O
>>508
省略表記の否定比率リスト(合ってる自信はない)

@ 112
A 123
B 134 235
C 145 347 257 358
D 156 459 37.10 47.11 279 57.12 38.11 58.13
0525132人目の素数さん
垢版 |
2018/12/02(日) 03:32:20.82ID:KWCNNez9
>>524
問題から出題の経緯までそっくりwww
もしかしてサイトの本人?
でも設定微妙にちがうよね?
>>509は対角線の延長の交点も追加するのありといってるけどそれだとサイトの6点配置が2つ載ってるけど両方アウトになる。
0526132人目の素数さん
垢版 |
2018/12/02(日) 04:53:25.97ID:D2g2qj2O
>>504
やっと否定比率の途中過程が分かった

1  1:1:2
2  1:2:3
3  1:3:4
4  3:4:7
5  4:7:11
6  4:11:15
7  4:15:19
8  15:19:34

73  15:994:1009
0527132人目の素数さん
垢版 |
2018/12/02(日) 08:34:17.53ID:Gu3V0Rky
正三角形ABCがある。正三角形ABCの内部に、点Aから5cm、点Bから12cm、点Cから13cm離れた場所で点Pがある。
この時、正三角形ABCの面積を中学数学のみで求めよ。
0529132人目の素数さん
垢版 |
2018/12/02(日) 11:26:56.11ID:br7NSo8H
>>527
maxima 先生に聞いたら結構な値になるな。

vol(ab,ac,ad,bc,bd,cd):=sqrt(determinant(matrix([0,1,1,1,1],[1,0,ab^2,ac^2,ad^2],[1,ab^2,0,bc^2,bd^2],[1,ac^2,bc^2,0,cd^2],[1,ad^2,bd^2,cd^2,0]))/8)/6;
solve(vol(5,12,13,x,x,x)=0);
[x=−sqrt(20*3^(3/2)+169),x=sqrt(20*3^(3/2)+169),x=−sqrt(169−20*3^(3/2)),x=sqrt(169−20*3^(3/2)),x=0]
0530132人目の素数さん
垢版 |
2018/12/02(日) 11:51:44.05ID:s5Fyrcu3
>>525
人違いだけど、こうまで一致してるとこの類いの問題は少なくともある界隈では結構有名ってことなのかねえ
まあ、問題設定違って答えも違うということは、おそらく生徒か>>509の問題の聞き間違えってことなのかな
0531132人目の素数さん
垢版 |
2018/12/02(日) 20:33:32.98ID:UHgyXCtV
# A,B,C,...,G,J,K 11人のうち、何人かが嘘つきで残りは正直者で
# 嘘つきは必ず嘘をつく。全員だれが嘘つきか知っている。
# 11人に「うそつきは何人いますか」と聞くと、下のように答えました。
# A:1人,B:2人,C:3人,...,G:9人,J:10人,K:12人
# 誰が嘘つきでしょうか
0535132人目の素数さん
垢版 |
2018/12/02(日) 21:51:41.65ID:8De20Fh0
>>532
この手の問題って全員が同等に他人の思考を推論できるというのが前提になってるね。あってんじゃね。
0537132人目の素数さん
垢版 |
2018/12/02(日) 23:08:54.72ID:EzxIt8h1
>>531

> # A,B,C,...,G,J,K 11人のうち、何人かが嘘つきで残りは正直者で

H,Iはいないの?
書き忘れ?

> # A:1人,B:2人,C:3人,...,G:9人,J:10人,K:12人

これは
I:9人,J:10人,K:12人?
0540132人目の素数さん
垢版 |
2018/12/02(日) 23:14:43.65ID:EzxIt8h1
>>538
まぁ全員言ってる事違うので正直者が二人以上いることはない。

問題文では全員嘘つきが禁止されてないので

K:12人

だと全員嘘つきの解もありえる。

K:11人

ならその可能性が消えるのでJだけが正直者が唯一の解。
0542132人目の素数さん
垢版 |
2018/12/03(月) 03:28:39.04ID:IEuLNtWu
1枚だけページが破れた本がある。
破れていないページ番号を合計すると15000になる。
破れたページは何ページ目だろうか?
0545イナ ◆/7jUdUKiSM
垢版 |
2018/12/03(月) 10:34:23.13ID:QNCy2EJ5
>>533
ADとBFの交点をGとし、FからADに垂線Hを引く。
△BEF=□ABCD-△ABG-△BCE+△FGH
=18×18-6×18-18×9÷2+3×9÷2
=324-54-81+13.5
=202.5(cu)
>>532
0548132人目の素数さん
垢版 |
2018/12/03(月) 12:01:48.76ID:se7yN58/
Prelude Data.List> head [x | x<-[1..], (div (x*(x+1)) 2) - x - x-1 > 15000]
175
Prelude Data.List> [(x,y)|x<-[1..174],y<-[1..x],div (x*(x+1)) 2 - y- y-1 == 15000]
[(173,25),(174,112)]

より最終ページが奇数が許されるなら 25, 26 ページ。
最終ページが必ず偶数なら解無し。
0549132人目の素数さん
垢版 |
2018/12/03(月) 12:46:13.13ID:E4TDocZk
臭いページを破ったんじゃないの?
全178ページで合計は 178*(178+1)/2 = 15931
破られたページは 465,466 ページ
0551132人目の素数さん
垢版 |
2018/12/03(月) 13:50:56.60ID:QN9IvGKv
1ページ破るから、表裏がなくなんのね。
ようやく意味がわかった。
0552132人目の素数さん
垢版 |
2018/12/03(月) 14:35:51.51ID:QN9IvGKv
>>549
178頁しかないのにどうやって465,466 ページを破るんだ?

日本の諺:無い袖は振れない
0553132人目の素数さん
垢版 |
2018/12/03(月) 14:56:23.08ID:QN9IvGKv
>>551
いつもの顰蹙、プログラム解
rip <- function(n){
page=(n*(n+1)/2-15000)%/%2
torn=ifelse(page%%2,page,0)
ifelse(0<torn & torn<n,torn,0)
}
rip=Vectorize(rip)
n*(n+1)/2-3=15000
1/2*(5*sqrt(4801)-1) # 172.7231
n*(n+1)/2-(2n-1)=15000
1/2*(3+7*sqrt(2449)) # 174.7058

> rip(173)
[1] 25
> rip(174)
[1] 0
0556132人目の素数さん
垢版 |
2018/12/03(月) 16:40:22.42ID:CykXOOQX
嘘つきがファジーとしてみました。

A,B,C,...,G,J,K 11人のうち、何人かが嘘つきで残りは正直者で
全員だれが嘘つきか知っている。
正直者は嘘をつかないが
嘘つきは嘘をつくこともつかないこともある。
11人に「うそつきは何人いますか」と聞くと、下のように答えました。
A:1人,B:2人,,...,J:10人,K:11人
誰が嘘つきでしょうか
0558132人目の素数さん
垢版 |
2018/12/03(月) 19:10:55.96ID:QN9IvGKv
>>556
11人が1, 2, 3, 4, 5, 5, 5, 5, 9, 9, 9 答えたとすると
嘘つきが必ず嘘をつくか、嘘もホントも答えるかで変わってくるね。
0559132人目の素数さん
垢版 |
2018/12/03(月) 19:21:43.31ID:nkVZQTbD
分かりやすい説明がありましたわ

「誰も」とか「誰か」の内容は文脈によって決まるので
ある場合にそれは、太郎と花子と次郎という想定が可能である

太郎が花子をねたみ
花子が次郎をねたみ
次郎が太郎をねたんでいる

そういった場合には、「誰もが誰かをねたんでいる」けれど
誰もからねたまれている「誰か」は存在しない

と、そういうことらしいわ
0560132人目の素数さん
垢版 |
2018/12/03(月) 21:46:47.16ID:CykXOOQX
>>558
嘘つきがファジーだとすると
この例だと
全員嘘つき以外に9と答えた3人のうち2人が正直者で他が嘘つき
という組合せも可能。
総当たりのプログラムではそれだけになったがプログラムに余り自信がない。
コイントスや待ち時間と違ってシミュレーションができない。
0564132人目の素数さん
垢版 |
2018/12/04(火) 01:08:47.08ID:zMVauxBb
>>558のFuzzy versionのsolver
testimonies = [
(==1).length.(filter (==False)),
(==2).length.(filter (==False)),
(==3).length.(filter (==False)),
(==4).length.(filter (==False)),
(==5).length.(filter (==False)),
(==5).length.(filter (==False)),
(==5).length.(filter (==False)),
(==5).length.(filter (==False)),
(==9).length.(filter (==False)),
(==9).length.(filter (==False)),
(==9).length.(filter (==False))
]
isCompatible ts theCase = all (==True) $ zipWith (||) (map not theCase) (map (¥x -> x theCase) ts)
cases = (!! (length testimonies)) $ iterate (¥x-> [a:b|a<-[True,False],b<-x]) [[]]
main = mapM_ print [theCase | theCase<-cases, isCompatible testimonies theCase]

*Main> main
[False,False,False,False,False,False,False,False,True,True,False]
[False,False,False,False,False,False,False,False,True,False,True]
[False,False,False,False,False,False,False,False,False,True,True]
[False,False,False,False,False,False,False,False,False,False,False]
0565132人目の素数さん
垢版 |
2018/12/04(火) 08:34:38.33ID:QKKYvADK
>>564
いつもHaskellコードありがとうございます。
Rの結果(嘘つきを1で表示)と一致したので安心しました。

> liars <- function(Answer,Strict=TRUE){ # duplicate answer and/or case of all liars permitted
+ N=length(Answer)
+ arg=list()
+ for(i in 1:N) arg[[i]]=0:1
+ dat=do.call(expand.grid,arg) # expand.grid(0:1,0:1,0:1,...,0:1)
+ colnames(dat)=LETTERS[1:N]
+ check <- function(y,answer=Answer){
+ if(all(y==1)) {!all(1:N %in% answer)}
+ else{ # Strict: all honest answer compatible & not included in liar's anwer
+ if(Strict){all(answer[y==0]==sum(y)) & !(sum(y) %in% answer[y==1])}
+ else {all(answer[y==0]==sum(y))}
+ }
+ }
+ res=as.matrix(dat[apply(dat,1,check),])
+ rownames(res)=NULL
+ return(res)
+ }
>
> liars(c(1, 2, 3, 4, 5, 5, 5, 5, 9, 9, 9),Strict=FALSE)
A B C D E F G H I J K
[1,] 1 1 1 1 1 1 1 1 1 0 0
[2,] 1 1 1 1 1 1 1 1 0 1 0
[3,] 1 1 1 1 1 1 1 1 0 0 1
[4,] 1 1 1 1 1 1 1 1 1 1 1
0566132人目の素数さん
垢版 |
2018/12/04(火) 12:43:32.06ID:5n977z/C
>>558 の Fuzzy Version 位なら理詰めでもそんなに苦労はしないけど。
でも今回みたいな問題なら理詰めで解く事に拘ってもしょうがない気もする。

一致してる証言の数が最大4なので最低でも7人の嘘つきがいる。
よってその数が5人以下と証言しているA〜Iは嘘つきであることが確定する。
よって正直者がいるとすればI,J,Kのうちの何人かに限られるが、その数も証言により2と確定する。
よって正直者の集合は{J,K},{I,K},{I,J}のいずれかであることが必要。
逆にこのとき条件は満たされる。
全員嘘つきも条件を満たすので以上4つが解である。
0567132人目の素数さん
垢版 |
2018/12/04(火) 15:13:07.58ID:7f8uMrnq
答が7,7,7,7,8,8,8,9,9,10,11 だとこんな結果(1が嘘つき、0が正直者)

> liars(c(7,7,7,7,8,8,8,9,9,10,11),Strict=F)
A B C D E F G H I J K
1 1 1 1 1 1 1 1 1 1 0 1
2 1 1 1 1 1 1 1 0 0 1 1
3 1 1 1 1 0 0 0 1 1 1 1
4 0 0 0 0 1 1 1 1 1 1 1
5 1 1 1 1 1 1 1 1 1 1 1
Answer 7 7 7 7 8 8 8 9 9 10 11
0568132人目の素数さん
垢版 |
2018/12/04(火) 21:10:26.55ID:ekJSUZdS
A,Bの二人が同じ出発地点から同じ目的地点に向けて同じ道を同時に出発する。
最初はAが歩き、Bが自転車で出発する。
Bがある距離進んだところで自転車を残して歩き出す。
Aが自転車にたどり着いたら自転車に乗ってBを追い越し、ある距離進んだところで自転車を残して歩き出す。
全行程をこのようにして進む。

どの時点でも二人のどちらかは歩いているが、片方が自転車にのっているときもあるので、自転車が無いときより早く着くという意見がある。
どの時点でもどちらかが必ず歩いていたのだから、自転車があっても早くならないという意見もある。

どちらの意見が正しいのか?
0569132人目の素数さん
垢版 |
2018/12/04(火) 23:09:14.80ID:9sqWfWC3
>>568
イメージしにくかったら具体例で考えてみるというのはどうか。
徒歩は時速3キロ、自転車は時速20キロ、目的地は60キロ先。徒歩のみで行けば20時間かかる。
Bが自転車を30キロ地点で降りたとすると、そこまで1.5時間かかっているので、残りの行程は10時間、到着まで11.5時間となる。
Aは10時間かけて自転車のある30キロ地点に着き、それから残りの行程を10時間かけて進む。到着までは同じく11.5時間となる。
徒歩のみよりはどちらも早い。
0570132人目の素数さん
垢版 |
2018/12/04(火) 23:10:24.61ID:9sqWfWC3
>>569訂正
>Aは10時間かけて自転車のある30キロ地点に着き、それから残りの行程を1.5時間かけて進む。到着までは同じく11.5時間となる。
0572132人目の素数さん
垢版 |
2018/12/05(水) 06:58:44.87ID:kDWs7GYM
歩行速度a, 自転車速度bで平均速度は1/((1/a+1/b)/2) =2ab/(a+b)
上の例だとの2*3*30/(3+20)=120/23=60/11.5=5.217
自転車での走行距離には無関係なんだな。
0573132人目の素数さん
垢版 |
2018/12/05(水) 07:04:26.84ID:kDWs7GYM
0 < a < b のときa < 2ab/(a+b)と言えるか、という問題に帰結。
あとは任せた。
0578132人目の素数さん
垢版 |
2018/12/05(水) 09:44:41.52ID:O2YJX1G3
>>569
上の例でL=60,x=30
0<a<b, 0<x<Lの時
a < (a b L)/(a x + b (L - x))を示せに帰着。

c=x/Lとおいて0<c<1
右辺=ab/((a-b)c+b)
左辺 - 右辺=a-ab/((a-b)c+b)=(ac (a - b))/(a c + b (1 - c))
分子<0,分母>0で不等式成立。
0579132人目の素数さん
垢版 |
2018/12/05(水) 13:04:57.15ID:NGe6N71c
>568
どちらかが常に歩いてはいるが、Bが自転車を置いた場所にAが辿り着く頃には、Bはもっと先の場所を歩いているわけだから、そのぶん時間が短縮されるのは自明。
0581イナ ◆/7jUdUKiSM
垢版 |
2018/12/05(水) 20:09:17.35ID:duwFGlVv
>>563
△FEDは考えなくていい。求められてない。求められてるのは△BEFの面積。
FもEもDもそこにあるだけ。どこへも行かない。
>>545
GH=3、FH=HD=DE=9を作図する。
HDとFEの交点Mを中心に、
△DMEと△HMFは点対称だから、
△DMEは□ABCDから引かなくていい。
△DMEを引いてもどっちみち△BEFを求めるとき、
△HMFを足すことになるから。
0582132人目の素数さん
垢版 |
2018/12/07(金) 12:31:49.36ID:a4KQBFJb
S を集合とする。T は S の部分集合からなるある集合であり、包含関係に関して全順序をなす。すなわち、
A,B∈T ならば (A⊂B または B⊂A)
が成り立つ。
(1) S が可算集合の時、T の濃度はどのくらい大きくなれるか。
(2:未解決) S が非可算の場合はどうか。
0583132人目の素数さん
垢版 |
2018/12/07(金) 13:04:38.50ID:+DIQ+jRM
>>582
(1)はQの部分集合Aで
x∈A、y≦x⇒y∈A を満たすもの全体が連続体濃度になるので連続体濃度が答えかな?
0584132人目の素数さん
垢版 |
2018/12/07(金) 15:03:46.83ID:ppqSvBiP
>>583
正解
(2)は一般連続体仮説を採用すれば似たような方法で T=|2^S| となる T の存在が言えるけど、
果たして ZFC だけから出せるだろうか?というのが疑問になってできた問題
0585132人目の素数さん
垢版 |
2018/12/10(月) 01:33:32.73ID:MNYdlnCZ
kを自然数とする。
漸化式
x[1] = x[2] = 1
x[n+2] = x[n+1](x[n+1]+k)/x[n]
で定められる数列の全ての項は整数であることを示せ。
0586132人目の素数さん
垢版 |
2018/12/10(月) 08:38:45.57ID:nBAqzpCd
n人でじゃんけんを1回だけする。
1回のじゃんけんでの勝者の数をxとする。
xの期待値が最大となるnはいくつか?
0587132人目の素数さん
垢版 |
2018/12/10(月) 10:41:18.69ID:fcMs8kve
>>586
期待値は
Σ[k=1,n-1] C[n,k] (1/3)^(n-1)
= n ((2/3)^(n-1) - (1/3)^(n-1))
1≦n≦4においてはn=4のとき最大値 28/27。
n ≧ 2 のとき
(n+1) (2/3)^n
≦ n (2/3)^(n^1) (2n+2)/(3n)
より n≧5のとき
n(2/3)^(n-1) ≦ 5 (2/3)^4 = 80/81
∴n≧5のとき期待値<28/27。
0590132人目の素数さん
垢版 |
2018/12/11(火) 00:11:13.28ID:QM6MoWQK
>>585
g_0(t)=1, g_1(t)=1, g_2(t)=1, g_3(t)=t+1,
g_n(t)=(2t+2)g_{n−2}(t)−g_{n−4}(t) (n≧4)

としてg_n(t)を定義すると、n≧0に対して、g_n(t)は0でない整数係数多項式である。
また、数学的帰納法で

g_n(t)g_{n−3}(t)−g_{n−1}(t)g_{n−2}(t)=t (n≧3)

が示せる。次に、f_n(t)=g_n(t)g_{n−1}(t) (n≧1) と置くと、
n≧1に対して、f_n(t)は0でない整数係数多項式である。また、

f_1(t)=1, f_2(t)=1, f_n(t)=f_{n−1}(t)(f_{n−1}(t)+t)/f_{n−2}(t) (n≧3)

が成り立つことが示せる。特に、>>585のx[n]に対して x[n]=f_n(k) (n≧1) である。
f_n(t)は整数係数多項式だから、f_n(k)は整数であり、よってx[n]は整数である。
0591132人目の素数さん
垢版 |
2018/12/11(火) 01:08:41.28ID:h7NydyA2
>>590
おお、すばらしい。正解です。
解答も想定解答の一つと同じ。
ちなみにこれk=1の場合が同志社大学の過去問らしいんですが、どなたか元ネタご存知ありません?
0592132人目の素数さん
垢版 |
2018/12/11(火) 04:18:22.38ID:Fzml7DDz
単位円の面積を4等分する曲線(分岐あり)の長さの最小値を求めよ
0593132人目の素数さん
垢版 |
2018/12/11(火) 05:01:03.99ID:AqNnxt5M
そういやなんかこの手の問題で面積極小になるときの曲線とか面とかのなす角は××°になるって話をなんか”××の原理”って呼ぶって話してた人いたけどなんの原理でしたっけ?
なんかギリシャ時代だかなんだかの学者さんの名前だった記憶があるんですけど。
0594132人目の素数さん
垢版 |
2018/12/11(火) 05:04:37.43ID:Fzml7DDz
プラトーの法則だね
プラトー自体は19世紀ベルギーの物理学者
2次元の場合はジャンクションが120°になって境界とのなす角は直角になる
0596132人目の素数さん
垢版 |
2018/12/11(火) 07:58:39.78ID:gGtYeNxO
>>585
 k>0 だから特性多項式 tt -2(k+1)t +1 は2実根α、βをもつ。
 α = k+1 - √(k(k+2)),
 β = k+1 + √(k(k+2)),
 β-α = 2√(k(k+2)),
題意より
x[n+2] - x[n+1] = k(β^n - α^n)/(β-α),
であるが、上記により
α+β = 2(k+1)、αβ = 1。
またα、βの整係数対称式はこれらの整多項式だから整数。


なお、
x[n+1] = 1 + k[(β^n -1)/(β-1) - (α^n -1)/(α-1)] /(β-α),
0597132人目の素数さん
垢版 |
2018/12/11(火) 12:13:23.54ID:812OVAmO
>>585
ちょっと理屈わかんないけど数値実験では合わないですね。
Prelude> let alpha k = k + 1 - (sqrt $ k*(k+2))
Prelude> let beta k = k + 1 + (sqrt $ k*(k+2))
Prelude> let x k n = 1 + k*(((beta k)^(n-1) -1)/((beta k)-1) - ((alpha k)^(n-1) -1)/((beta k)-1)) /((beta k)-(alpha k))
Prelude> let k = 1 in mapM_ print [x k n | n<-[1..10]]
1.0
1.3660254037844388
2.464101615137755
6.49038105676658
21.497422611928567
77.49930939094767
286.4998149518621
1066.4999504165007
3977.4999867141414
14841.499996440063

Prelude> let y k = map head $ iterate (¥[u,v]->[v,(v)*(v+1)/(u)]) [1,1]
Prelude> let k = 1 in mapM_ print $ take 10 $ y k
1.0
1.0
2.0
6.0
21.0
77.0
286.0
1066.0
3977.0
14841.0
0598132人目の素数さん
垢版 |
2018/12/11(火) 12:52:45.22ID:+jUDREv9
>>596-597
失礼。あってますね。orz
Prelude> let theAlpha k = k+1 - (sqrt $ k*(k+2))
Prelude> let theBeta k = k+1 + (sqrt $ k*(k+2))
Prelude> let x k n = let {alpha = theAlpha k; beta = theBeta k} in 1 + k*((beta^n -1)/(beta-1) - (alpha^n -1)/(alpha-1)) /(beta-alpha)
Prelude> let k = 1 in mapM_ print [x k n | n<-[0..9]]
1.0
1.0
2.0
6.0
21.0
77.0
286.0
1066.0
3977.0
14840.999999999996
Prelude> let y k = map head $ iterate (¥[u,v]->[v,(v)*(v+1)/(u)]) [1,1]
Prelude> let k = 1 in mapM_ print $ take 10 $ y k
1.0
1.0
2.0
6.0
21.0
77.0
286.0
1066.0
3977.0
14841.0
0599132人目の素数さん
垢版 |
2018/12/11(火) 16:56:36.87ID:gGtYeNxO
>>585

題意から
x[1] = 1, x[2] = 1, x[3] = k+1,

y[n] = {x[n+1] + k + x[n-1]} / x[n] とおくと、

y[2] = {x[3] + k + x[1]} / x[2] = 2(k+1),

y[n+1] - y[n] = {x[n+2]/x[n+1] - (x[n+1]+k)/x[n]} + {(x[n]+k)/x[n+1] - x[n-1]/x[n]} = 0,

∴ y[n] = 2(k+1),

これより、線形漸化式
x[n+1] - 2(k+1)x[n] + x[n-1] + k = 0,
を得る。
∴ x[n] のすべての項は整数である。

k>0 だから特性多項式 tt -2(k+1)t +1 は2実根をもつ。
>>596 に続く。
0600132人目の素数さん
垢版 |
2018/12/11(火) 17:03:48.34ID:NJc7rTqU
>>591,585
x[n]=a[n]a[n-1] とおくと、a[n]=(a[n-1]a[n-2] + k)/a[n-3] となって、Somos Sequence に似てる。
(k=0がSomos Sequence) http://mathworld.wolfram.com/SomosSequence.html
a[0]=a[1]=1, a[n]=(a[n-1]^2 + k)/a[n-2] でも整数性があるみたいだけど、
a[0]=a[1]=a[2]=a[3]=1, a[n]=(a[n-1]a[n-3] + a[n-2]^2 + k)/a[n-4] では整数性がない。

Somos Sequenceはクラスター代数と深い関係があるようなので、元ネタはその関係じゃないのかな?
0601132人目の素数さん
垢版 |
2018/12/11(火) 17:11:21.29ID:G3kmmu1J
>>600
そうなんですか?
用意した解法のもう一方がsomosの整数性を証明したローラン現象という事がこの数列でも起こってることを確認する方法なんですが、一般にこの現象を起こす何か十分条件的なものがあるんですかねぇ?
そのクラスター代数との関わりってのはどんなんですか?
0602132人目の素数さん
垢版 |
2018/12/11(火) 17:17:19.20ID:NJc7rTqU
>>601
Somos Sequenceはクラスター代数と関係があるってことを聞いたことがあるだけなので、
すまないけど詳しいことは全く知らないです。
0603132人目の素数さん
垢版 |
2018/12/11(火) 18:56:26.41ID:Er+Nf/KD
>>602
ですか。
残念。
この同志社の問題どうやって作ったのか謎で謎で。
多分なんかこの手の問題出てきた背景的なものあると思うんだけどなぁ。
0604132人目の素数さん
垢版 |
2018/12/12(水) 01:33:43.21ID:neKV7OZz
>>599

線形漸化式
x[n+1] - 2(k+1)x[n] + x[n-1] + k = 0,
より
x[n] = (1/2) + T_{2n-3}(coshθ)/(2coshθ)
 = {coshθ + cosh((2n-3)θ)} /(2coshθ)
 = cosh((n-1)θ)cosh((n-2)θ) /(coshθ),
ここにθは cosh(2θ) = k+1,
T_n はn次の第一種チェビシェフ多項式。
0605132人目の素数さん
垢版 |
2018/12/12(水) 01:35:23.62ID:neKV7OZz
>>600

線形漸化式
a[n+2] - 2(k+1)a[n] + a[n-2] = 0,
より
a[n] = cosh((n-1)θ)   (n:奇数)
   = cosh((n-1)θ)/coshθ  (n:偶数)
ここにθは cosh(2θ) = a[3] = k+1,

a[2m+1] = T_m(1+k),
a[2m] = T_{2m-1}(coshθ) / coshθ
   = {a[2m+1] + a[2m-1]} / (2+k),
T_n はn次の第一種チェビシェフ多項式。

nが小さい方は
a[1] = 1,
a[2] = 1,
a[3] = 1+k,
a[4] = 1+2k,
a[5] = 1+4k+2kk,
a[6] = 1+6k+4kk,
a[7] = 1+9k+12kk+4k^3,
0607 【月】
垢版 |
2018/12/12(水) 02:14:17.55ID:Gi2B8yq5
>>592境界線の長いほうをx、短いほうが円の中心を通り2yとなるようにとると、
xは頂角120°、底辺√2の二等辺三角形の等しい二辺だから、
x・(√3/2)=√2/2
x=√2/√3=(√6)/3
y=√2/2-x/2=√2/2-√6/6=(3√2-√6)/3
最小値は、
4x+2y=(4√6)/3+(3√2-√6)/3
=√6+√2
≒3.86370331
>>545
0608 【上級国民】
垢版 |
2018/12/12(水) 02:21:16.65ID:Gi2B8yq5
>>607訂正。
>>592境界線の長いほうをx、短いほうが円の中心を通り2yとなるようにとると、
xは頂角120°、底辺√2の二等辺三角形の等しい二辺だから、
x・(√3/2)=√2/2
x=√2/√3
=(√6)/3
y=√2/2-x/2
=√2/2-√6/6
=(3√2-√6)/6
最小値は、
4x+2y=(4√6)/3+(3√2-√6)/3
=√6+√2
≒3.86370331
>>545
0609132人目の素数さん
垢版 |
2018/12/12(水) 02:57:52.09ID:neKV7OZz
>>600

a[0] = 1, a[1] = 1, a[2] = k+1,
a[n]a[n-2] - a[n-1]^2 = k,
の場合は
b[n] = (a[n+1] + a[n-1]) /a[n] とおくと

b[1] = (a[2] + a[0])/a[1] = k+2,

b[n] - b[n-1] = {(a[n+1] +a[n-1])a[n-1] - (a[n]+a[n-2])a[n]} / (a[n]a[n-1])
 = {(a[n+1]a[n-1] - a[n]^2) - (a[n]a[n-2] - a[n-1]^2)} / (a[n]a[n-1])
 = (k-k) / (a[n]a[n-1])
 = 0,
∴ b[n] = k+2,

線形漸化式
a[n+1] - (k+2)a[n] + a[n-1] = 0,
より
a[n] = T_{2n-3}(coshθ) /(coshθ)
  = cosh((2n-3)θ) /(coshθ),
ここにθは cosh(2θ) = (k+2)/2,
T_n はn次の第一種チェビシェフ多項式。
0610132人目の素数さん
垢版 |
2018/12/12(水) 04:21:42.53ID:vVlAH203
>>608
不正解です
それだと面積4等分にならない
0612 【鶴】
垢版 |
2018/12/12(水) 11:20:38.81ID:Gi2B8yq5
>>608訂正。
前々>>607前々の前>>545>>581
>>592境界線の長いほうをx、短いほうが円の中心を通り2yとなるようにとると、
頂点が4つあるほうの領域を円の中心を通る半径で二分した領域の面積は、弧をLとして、
π/8=L/2+(xy√3)/2
斜辺1の直角三角形について、三平方の定理より、
(x/2+y)^2+{(x√3)/2}^2=1
x^2/4+xy+y^2+3x/4=1――@
最小値4x+2y=mとおくと、
y=m/2-2x
@に代入すると、
x^2/4+x(m/2-2x)+(m/2-2x)^2+3x/4=1
x^2+(x/2)m-2x^2+m^2/4-2xm+4x^2+3x/4-1=0
x^2+2xm-8x^2+m^2-8xm+16x^2+3x-4=0
9x^2-6xm+m^2+3x-4=0
9x^2+(3-6m)x+m^2-4=0
(判別式)=(3-6m)^2-4・9(m^2-4)=0
9-36m+36m^2-36m^2+144=0
36m=153
m=17/4(=4.25)
0613 【小吉】
垢版 |
2018/12/12(水) 11:49:20.18ID:Gi2B8yq5
>>612訂正。面積要らなかった。
>>592境界線の長いほうをx、短いほうが円の中心を通り2yとなるようにとると、
内角120°で分岐する長さxの直線2つと弧で囲まれた領域にある、
半径1を斜辺とし、その弧に対する弦の半分とその弦の中点から円の中心までを二辺とする直角三角形について、三平方の定理より、
(x/2+y)^2+{(x√3)/2}^2=1
x^2/4+xy+y^2+3x/4=1――@
最小値4x+2y=mとおくと、
y=m/2-2x
@に代入すると、
x^2/4+x(m/2-2x)+(m/2-2x)^2+3x/4=1
x^2+(x/2)m-2x^2+m^2/4-2xm+4x^2+3x/4-1=0
x^2+2xm-8x^2+m^2-8xm+16x^2+3x-4=0
9x^2-6xm+m^2+3x-4=0
9x^2+(3-6m)x+m^2-4=0
(判別式)=(3-6m)^2-4・9(m^2-4)=0
9-36m+36m^2-36m^2+144=0
36m=153
m=17/4(=4.25)
0614132人目の素数さん
垢版 |
2018/12/12(水) 12:40:53.92ID:LhhrZ/FI
>>611
これを解いて最小値は

> len = function(θ) 4*sqrt((cos(θ)-((4*θ)-pi)/sin(θ)/4)^2 + sin(θ)^2) + 2*((4*θ)-pi)/sin(θ)/4
> optimize(len,c(0,pi/2))$o
[1] 3.95125826
0618132人目の素数さん
垢版 |
2018/12/12(水) 13:46:29.04ID:CF6JIWOg
>>611
>>614

残念ながら不正解
直線のみならそれで大丈夫だけど解は曲線です
0620132人目の素数さん
垢版 |
2018/12/12(水) 15:34:55.02ID:hGPRw/Of
この場合もプラトーの法則成立するんでない?
・曲線は円、または直線。
・分岐は3枝でなす角は120°。
・外円との接続部は90°。
0621132人目の素数さん
垢版 |
2018/12/12(水) 16:53:47.97ID:VPYuy2Z7
>>620
minimize 2x+4(1-x*cosθ)(π/3-θ)/sin(π/3-θ) where θ-x*sinθ+((1-x*cosθ)((π/3-θ)/sin(π/3-θ)-1))^2=π/4
という式をたててみたがwolframalphaさんは答えを出してくれなかった

手で計算したところ
x=0.18059
θ=0.93017(単位円との交点間の弧の長い方の角度=106.59度)
のとき、分割線長の総和=3.9376
0622132人目の素数さん
垢版 |
2018/12/12(水) 17:04:46.38ID:VPYuy2Z7
>>621で仮定した条件
分割線が(-x,0)-(x,0)の線分、および>>620の条件を満たす(±x,0)-(±cosθ,sinθ),(±x,0)-(±cosθ,-sinθ)の円弧(複号同順)
0624132人目の素数さん
垢版 |
2018/12/12(水) 17:25:08.78ID:VPYuy2Z7
>>623
そうなります
ぜんぶ直線だとプラトーの法則が成立しないので

中央の線分を円弧に変えたらもっと良くなるのかどうかは検証していません
0625132人目の素数さん
垢版 |
2018/12/12(水) 17:51:53.99ID:neKV7OZz
2つの分岐点を (0, ±b) とする。(0<b<1)

境界円の中心 {±(1-bb)/(2b√3),±(1+bb)/(2b)}
境界円の半径 R = (1-bb)/(b√3),

境界円と外円の交点 {±((√3)/2)(1-bb)/(1+b+bb),±(1/2)(1+4b+bb)/(1+b+bb)}

境界円の円周角 θ = (π/3) - arctan((√3)(1-bb)/(1+b+bb)),

境界線の合計長さ L = 2b + 4Rθ,
0629132人目の素数さん
垢版 |
2018/12/12(水) 19:03:07.39ID:VPYuy2Z7
>>628
分岐のところの角度をそれぞれ120°にすると条件が揃います
0630132人目の素数さん
垢版 |
2018/12/12(水) 20:24:12.24ID:LhhrZ/FI
>>625
bbってb*b=b^2ですよね?

Lの最小値を求めようとグラフにしたらこんなになったんだけど。

https://i.imgur.com/NAxEnvp.png

"
2つの分岐点を (0, ±b) とする。(0<b<1)
境界円の中心 {±(1-bb)/(2b√3),±(1+bb)/(2b)}
境界円の半径 R = (1-bb)/(b√3),
境界円と外円の交点 {±((√3)/2)(1-bb)/(1+b+bb),±(1/2)(1+4b+bb)/(1+b+bb)}
境界円の中心角 θ = (π/3) - arctan((√3)(1-bb)/(1+b+bb))
境界線の合計長さ L = 2b + 4Rθ
"

L = function(b){
R=(1-b*b)/b/sqrt(3)
theta=pi/3-atan(sqrt(3)*(1-b*b)/(1+b+b*b))
2*b + 4*R*theta
}
curve(L(x))
0631132人目の素数さん
垢版 |
2018/12/12(水) 20:35:24.99ID:DiP58j3Z
結局
円内にある4つの円弧のうち右上のやつの半径をrとして(全部同じだけど)
(1) x^2 + y^2 ≦ 1 (y ≧ 0)、(x-rsinθ-cosθ)^2+(y - sinθ+rcosθ)^2 ≦ r^2
.  の表す領域の面積がπ/8
(2) rcosθ - sinθ = r/2
をとくのか。
(1)がめんどい。
0632132人目の素数さん
垢版 |
2018/12/12(水) 20:39:23.53ID:LhhrZ/FI
>>621
その値で作図してみました。
野球のボールというよりメガネ小僧って感じになりました。
0638132人目の素数さん
垢版 |
2018/12/12(水) 23:31:03.32ID:LJWyFz3p
∫[a-r,a-rcosθ]√(1-x^2)dx + ∫[a-rcosθ,1]√(r^2-(x-a)^2)dx = π/8 のときの 2(a-r) + 4rθの最小値を求める問題に帰着したけど
数式が複雑過ぎて断念。
0639132人目の素数さん
垢版 |
2018/12/13(木) 01:11:40.41ID:earCgaZv
こんなのキレイな解はでないんじゃね?
方程式たてて近似解求めるしかないよ。
多分だけど。
0640132人目の素数さん
垢版 |
2018/12/13(木) 01:21:07.87ID:PfacXwcf
計算機の出番だな
0641132人目の素数さん
垢版 |
2018/12/13(木) 01:41:27.16ID:WL83an/s
>>626 訂正

境界円の中心角 θ = (π/3) - arctan((√3)(1-bb)/(1+4b+bb)),
0642132人目の素数さん
垢版 |
2018/12/13(木) 03:37:05.83ID:WL83an/s
>>592

長さ 2b の境界線を共有する2個について、それぞれの面積は
σ(b) = {中心角 2(θ+π/6) の扇形} + 2・(底辺bの) - 2・(三日月形D)
 = (θ+π/6) + b・sin(π/3-θ) - RR(θ-sinθ),

ここに  >>625 >>626 から
 R(b) = (1-bb)/(b√3),
 θ(b) = (π/3) - arctan((√3)(1-bb)/(1+4b+bb)),

等分条件(σ=π/4)から b を求めると、

b = 0.122010156676718
θ = 0.198522403464296
R = 4.66154271422004
L = 2b + 4Rθ = 3.94570296726719

* 三日月形:弧と弦に囲まれた部分。
0643132人目の素数さん
垢版 |
2018/12/13(木) 04:29:03.52ID:WL83an/s
>>630
 >>625 >>626 のθの式が違ってました。スミマセン。(正しくは >>641)

 L(x) は L(0)=4 から L(1)=2 に単調に減少すると思います。L '(0)=0
 L(x) 〜 4 - 2x^(3/2) のような感じです
0644132人目の素数さん
垢版 |
2018/12/13(木) 05:01:42.49ID:051qyB7z
maxima先生の解答

r(s):=sin(s)/(cos(s)-1/2);
a(s,t):=r(s)*cos(t)+r(s)*sin(s)+cos(s);
b(s,t):=r(s)*sin(t)+sin(s)-r(s)*cos(s);
define(c(s,t),diff(a(s,t),t));
define(f(s),ratsimp(integrate(b(s,t)*c(s,t),t,5/6*%pi,%pi/2+s)+integrate(-(sin(t))^2,t,s,0)));
g(x):=f(x) - %pi/8;
h(x):=if g((x[1]+x[2])/2)>0 then [x[1],(x[1]+x[2])/2] else [(x[1]+x[2])/2,x[2]];
k(n,x):=if n = 1 then x else k(n-1,h(x));
s0:k(50,[0,1])[1],numer;
r(s0);
2 *a(s0,5/6*%pi)+4*r(s0)*(5/6*%pi - (%pi/2+s0)),numer;

(%o9) .8486751477323029
(%o10) 4.661542714220053
(%o11) 3.945702967267186
0647132人目の素数さん
垢版 |
2018/12/13(木) 07:12:18.51ID:aBlYWIwN
>>646
いや

直線ではなくまぎれもなく円弧で描いているんだけど
曲率が小さいのでほとんど直線にしか見えないのはご容赦
0648132人目の素数さん
垢版 |
2018/12/13(木) 07:12:55.22ID:xqqYuQS5
>>639
二つの円の半径と中心間の長さが与えられたときの変数の関係って
簡単な式にならないよね?
0650132人目の素数さん
垢版 |
2018/12/13(木) 12:42:43.67ID:vWEXT14C
>>644
s,t,rが何かわからんから、さっぱりわからん。
最後はニュートンラフソンぽいな。
0651132人目の素数さん
垢版 |
2018/12/13(木) 13:22:54.83ID:7IASv7Ug
B>0を定数とせよ.
N(B)=#{(a_1,...,a_n):a_1,...,a_nは最大公約数が1の整数, |a_i| ≦B}
則ちN(B)とは, 絶対値がB以下の2つの整数の組(a,b)で, a,bが互いに素なもの達全体の個数である.
此の時極限値
lim_{B→∞} N(B)/B^2
の値を求めよ.
0652132人目の素数さん
垢版 |
2018/12/13(木) 13:24:58.10ID:7IASv7Ug
>>36
出題しておきながら解き方を忘れたのだが, 局所大域原理が使えない為, K3曲面のBrauer群等を使わなければならない筈.
0653132人目の素数さん
垢版 |
2018/12/13(木) 13:57:23.72ID:GL4/bHh2
>>650
敷居の円弧のうち右上のものの右上端を(cos s, sin s), 半径をr(s)としてる。
するとその中心Aは
(cos s,sin s) + r(cos(s - π/2), sin(s-π/2))
= (cos s + r(s) sin s, sin s + r(s) cos s)
この円上の点Pの偏角を t としてPの座標(a(s,t),b(s,t))は
a(s,t):=r(s)*cos(t)+r(s)*sin(s)+cos(s);
b(s,t):=r(s)*sin(t)+sin(s)-r(s)*cos(s);
円弧の右上端が t = s +π/2、左下端が t = 5π/6。
面積f(s)がGreenの公式より
f(s) = ∫[5/6π,s+π/2](b(s,t)a’(s,t)dt +∫[s,0] sin(t) (cos t)’ dt + ∫ [略] 0 dx。
でこのf(s)がπ/8になる s が s0。
本来Maximaにはnewtonってニュートンラフソン使うパッケージがあるんだけどうまく動いてくれなかったので力技で求めてる。
(g(a[n])<0<g(b[n]) において g(a[n]+b[n]) >0 なら a[n+1] = a[n]、b[n+1] = (a[n]+b[n])/2…の第50項)
s0が決まればあとは代入するだけ。

maximaはもうオワコンなので極力つかいたくないんだけど、本問一番メンドイのが面積をパラメータ s で表す部分。
言語としては古臭くてなんだかなぁってとこ多いんだけどその辺のパッケージは充実してるから中々切れない。
sagemathにそろそろ移行したいんだけど。
0656132人目の素数さん
垢版 |
2018/12/13(木) 14:49:19.50ID:vWEXT14C
図がないとどうも理解がはかどらないので図を書いてみた。

https://i.imgur.com/L6VeY57.png

ロケット形PQRの面積がπ/8であるときの

4×弧PQ + 2×線分OQの最小値を求める のが課題。

θ,αは中心角, rが境界円の半径, bはOQの長さ

minimize 2*b+4rα where (θ-sinθ)/2+r^2(α-sinα)/2+(1-b)sinθ/2=pi/8 ,0<b<1,0<θ<pi/2

はWolframではタイムアウトした :(
0657132人目の素数さん
垢版 |
2018/12/13(木) 15:29:53.81ID:PfacXwcf
>>656
∠RQA=π/6という条件を入れてみるとどうか
0658イナ ◆/7jUdUKiSM
垢版 |
2018/12/13(木) 18:45:15.28ID:j/usDCql
>>613
境界線を4つの円弧と4つの直線xで描き、単位円に90°入射、分岐点を120°にして、2つの分岐点の距離をyとする。
分岐点の距離を長くして直線を短くして円弧を長くしたほうが最小値になるのかな?
直線4分割と図を重ねて描くと面積の過不足が一致すればいいはず。
0660132人目の素数さん
垢版 |
2018/12/13(木) 22:08:39.71ID:vWEXT14C
>>656
内包表示できる Haskellでやってみたけど、計算が終わらない。

-- b+4rα where (θ-sinθ)/2+r^2(α-sinα)/2+(1-b)sinθ/2=pi/8 ,0<b<1,0<θ<pi/2

rangeB = map (/1000) [1..1000]
rangeTheta = map (\x -> x * pi/2/1000) [1..1000]
rangeAlpha = map (\x -> x * pi/1000) [1..1000]
rangeR = map (\x -> x * 1/1000) [1000..10000]
re = [2*b+4*r*α| b<-rangeB,θ<-rangeTheta, α<-rangeAlpha,r<-rangeR,(θ-sin(θ))/2+r^2*(α-sin(α))/2+(1-b)*sin(θ)/2==pi/8]
minimum re
0663132人目の素数さん
垢版 |
2018/12/14(金) 00:52:30.86ID:7J/61zOG
>>662
その図でPA=r(s)、∠POR = s、
円弧PQ上の点Xで半直線AXの偏角がtであるものの座標が(a(s,t),b(s,t))。
Pは t = s + π/2、Qは t = 5π/6。
Qのy座標 b(s,5π/6) が 0 であることとb(s,t):=r(s)*sin(t)+sin(s)-r(s)*cos(s)であることからr(s):=sin(s)/(cos(s)-1/2)が出ます。
ちなみにPAはx^2+y^2=1に接してます。
0664132人目の素数さん
垢版 |
2018/12/14(金) 03:48:13.90ID:DbCBFUHo
>>611 >>614

境界線はすべて線分。
分岐点 (±x, 0)
境界線と外円の交点 (±cosθ,±sinθ)

θ = 0.869336877
のとき
π/3 - θ = 0.1778606742
x = 0.1098816404
y = 0.9328737443
L = 2x + 4y = 3.95125825794117475

(交角の条件 90゚,120゚ を満たさない。)
0665132人目の素数さん
垢版 |
2018/12/14(金) 06:00:27.62ID:DbCBFUHo
>>650
(>>644 ぢゃないが)

境界線と単位円の交点を(cos(s), sin(s)) とする。(s=s0 は定数)
交点で接線をひく。半径に垂直なので、x軸から(反時計回りに)π/2 +s の方を向く。
境界円の半径をrとする。(rは定数)
境界円の中心:(r sin(s)+cos(s), sin(s)-r cos(s))
境界点は、中心からrだけ離れている。
(a(r,s), b(r,s)) = (r cos(t)+r sin(s)+cos(s), r sin(t)+sin(s)-r cos(s))
tは、中心から境界点を見た方位。(x軸から反時計回りに)
分岐点の交角は120゚だから、π/2 +s ≦ t ≦ 5π/6,
分岐点がx軸上にある条件は
r sin(5π/6) + sin(s) - r cos(s) = 0,
∴ r(s) := sin(s)/[cos(s)-1/2];
c(s,t) := ∂a(s,t)/∂t;
f(s) := ∫[5π/6, π/2+s] b(s,t)c(s,t) dt + ∫[s,0] -(sin(t))^2 dt
 = ∫[a(s,5π/6), a(s,π/2+s)] b(s,t) da(s,t) + ∫[0,s] (sin(t))^2 dt;
はパーツの面積の半分。
g(s) := f(s) - π/8;
とおくと等面積条件は g(s) = 0,
h(x[1],x[2]) := [x[1], (x[1]+x[2])/2]  g((x[1]+x[2])/2) > 0 のとき
       := [(x[1]+x[2])/2, x[2]]  その他
k(n,x) := {g(s)=0 の根を含む、幅(1/2)^{n-1} の区間}
  (2分法? ニュートン法ぢゃなかったみたい…)
9行目 s0 := k(50,[0,1]);
10行目 r(s0);
11行目 2a(s0, 5π/6) + 4r(s0)(5π/6 - (π/2 + s0));
かなぁ
0670132人目の素数さん
垢版 |
2018/12/14(金) 10:30:15.76ID:X7/HRig3
>>667
https://i.imgur.com/DJUEwSb.png

∠OPAが90°∠OQXが120°という条件をいれないとこの問題は解けないんだろうか?

五角形の分岐線のときは結ぶ箇所が有限だったから、コンピュータがカブトガニ解をすぐに出してくれたんだが、
こっちは難しいなぁ。
0672イナ ◆/7jUdUKiSM
垢版 |
2018/12/14(金) 15:34:52.41ID:DQANJwVv
>>661
第T象限から第W象限まで形は同じだから、第T象限で考えると、
@直線y=x
A円弧(x-a-√2)^2+y^2=(a+√2)^2-1
B単位円x^2+y^2=1
の3つが互いに交差しあってxの区間が4つあって、
前2つと後2つが(直線y=xの上下の領域が)等しくなるから、
あとは積分だと思う。これが正攻法だと思う。aが決まると思う。
0673132人目の素数さん
垢版 |
2018/12/14(金) 19:10:43.44ID:BiTuD+9L
>>672
それだと自分で書いている「分岐点が120°」という条件すら満たしていないわけだが
0674イナ ◆/7jUdUKiSM
垢版 |
2018/12/14(金) 19:58:14.45ID:DQANJwVv
>>672
>>673分岐点の角度は、
すべて120°になる。
つまり単位円内の円弧は第1象限では点(a,0)においてx軸と60°の角をなすように始まり、緩やかに右にカーブしy=xをかすめるように突っ切って単位円と90°の角をなすように終わる。
このy=xとの交点を境に前後の面積が等しい。
@0≦x≦a
Aa≦x≦
B ≦x≦1/a+2
C1/a+2≦x≦1/√2
@+A=B+Cより、aが決まりそう。
A、Bの空欄は、
円弧と直線y=xの交点のx座標で、
円弧の中心はx軸上にはない。たぶん第2象限。
今ここを考え中。
円弧の内角は30°〜45°の範囲にあるが、カーブの向きから考えて37.5°より大きい。仮に40°なら、
境界線の最小値=2a+2π√2×4×40°/360°
=2a+(8/9)π√2
0678イナ ◆/7jUdUKiSM
垢版 |
2018/12/14(金) 20:51:23.17ID:DQANJwVv
>>676
>>677訂正したとおり、円弧の中心はx軸上にはなく、第3象限にあり、
直線y=-(1/√3)(x-a)上にあると考えています。
0680イナ ◆/7jUdUKiSM
垢版 |
2018/12/14(金) 21:42:45.71ID:DQANJwVv
>>678-679
円弧の中心を(b,(a-b)/√3)とすると、
円弧の半径は2角が30°と60°の直角三角形の辺の比より、
2(b-a)/√3
円弧の式は、
(x-b)^2+{y+(b-a)/√3}^2=4/3)(b-a)^2
単位円と円弧の交点は保留。
おそらく中心(b,(a-b)/√3)までの距離が2(b-a)/√3だから、bはaで表される。
点(a,0)を通り傾き√3の直線y=√3(x-a)とy=xの交点は、
(a√3/(√3-1),a√3/(√3-1))
有理化して、
((3+√3)a/2,(3+√3)a/2)
0682イナ ◆/7jUdUKiSM
垢版 |
2018/12/14(金) 22:18:15.23ID:DQANJwVv
>>680ちがうちがう。
y=√3(x-a)とy=xの交点じゃなく、円弧とy=xの交点じゃないと意味ない。

円弧の中心を(b,(a-b)/√3)とすると、
円弧の半径は2角が30°と60°の直角三角形の辺の比より、
2(b-a)/√3
円弧の式は、
(x-b)^2+{y+(b-a)/√3}^2=(4/3)(b-a)^2
y=xを代入すると、
x^2-2bx+b^2+x^2+2(b-a)x/√3+{(b-a)^2}/3=(4/3)(b-a)^2
2x^2+2(b-a-b√3)x/√3-(b-a)^2=0
x=(1/2)(a-b+b√3)-√{(b-a-b√3)^2+(b-a)}
=
計算中
0683 【大吉】
垢版 |
2018/12/15(土) 00:44:52.53ID:xEJOkZH0
>>682
x=(1/2){a+(√3-1)b}-(1/2)√[{(√3-1)b+a}^2+(b-a)]
=(1/2){a+(√3-1)b}-(1/2)√[{(4-2√3)b^2+2(√3-1)ab+a^2}+(b-a)]
0≦x≦aの範囲のy=xとx軸で囲まれた面積は、
a^2/2――@
a≦x≦(1/2){a+(√3-1)b}-(1/2)√[{(4-2√3)b^2+2(√3-1)ab+a^2}+(b-a)]の範囲のy=xと円弧で囲まれた面積は、
∫x=a〜(1/2){a+(√3-1)b}-(1/2)√[{(4-2√3)b^2+2(√3-1)ab+a^2}+(b-a)])
{x-(円弧の式をyについて解いたもの)}dx――A
円弧の式にx^2=1-y^2を代入し、yについて解くと、
やがてB+Cがわかり、
@+A=B+C
0684イナ ◆/7jUdUKiSM
垢版 |
2018/12/15(土) 22:31:12.14ID:xEJOkZH0
>>683
野球のボール→ の縫い目にΗのような境界線を描くとし、
単位円の中心から分岐点までの距離をaとする。
xy平面における点(a,0)である。
点(a,0)を通り第1象限においてy=xを下から上に突っ切るように、中心を点(a,0)の右下30°の方向にとり、円弧を描き、単位円と直交させる。
円弧の中心はy=√3(x-a)上の第4象限にあり、
直角三角形の辺の比より、円弧の半径を2、
円弧の中心のy座標を-1とすると、円弧は第1象限においてじゅうぶんな位置で単位円と直交できる。
∴a=2-√3
境界線の最小値=2a+4×(第1象限内の円弧)
=2a+4×2π×2×30/360
=2(2-√3)+4π/3
=4-2√3+4π/3
≒4.72468859
0686132人目の素数さん
垢版 |
2018/12/15(土) 23:05:59.20ID:Lt1TFZIo
nを2以上の整数とする。n個のコップが横一列に並んでいて、左端のコップにのみ水が入っている。
あなたは次の操作を好きなだけ繰り返して、全てのコップの水の量が同じにしたい。このことが可能なnを全て求めよ。
<操作>
ある右端以外で水の入っているコップを選び、それに入っている水の1/3を右隣のコップに移し、1/3を飲み、1/3をそのままにする。
0687イナ ◆/7jUdUKiSM
垢版 |
2018/12/15(土) 23:43:52.38ID:xEJOkZH0
>>686n=2がすべて。それ以上は無理。
もう飲めない。前>>684
ビーカーの底に水が少なくなって無理。
0690132人目の素数さん
垢版 |
2018/12/16(日) 03:20:32.38ID:HsDP4GrJ
>>686
右に一つ進む度にコップの水の価値は2倍あると考える。
たとえばコップが4つでそれぞれ 3,6,2,5 入ってるとき総価値は 3+6×2+2×4+5×8 = 53 である。
この状態で左から2番めのコップを動かすと 3,2,4,5 になるが総価値は 3+2×2+4×4+5×8 = 53 で変わらない。
(1/3はそのまま、1/3 飲まれてしまうが、1/3は価値が倍になるので総価値は変化しない。)
コップの数が4ので最終的に 1,1,1,1 になったとすると総価値は 15 であるから初期状態は 15,0,0,0 でなければならない。
しかし一番左のコップは各操作で 1/3 になるか変化しないかのいずれかなので操作を繰り返して 1 になることはない。
一般にコップの数が n で全部 1 にできたとすると総価値は 2^n -1 であるがこれが 3 のべきであることが必要である。
ここで n≧3 のとき 2^n - 1 ≡ 7 (mod 8)、3^m ≡ 1,3 (mod 8)により n≧3 である解はない。
よって可能な n は n=2 のみである。
0691イナ ◆/7jUdUKiSM
垢版 |
2018/12/16(日) 07:59:21.50ID:Ax8rD0fg
>>684訂正。
野球のボール→ の縫い目にΗのような境界線を描くとし、
単位円の中心から分岐点までの距離をaとする。
xy平面における点(a,0)である。
点(a,0)を通り第1象限においてy=xを下から上に突っ切るように、中心を点(a,0)の右下30°の方向にとり、円弧を描き、単位円と直交させる。
円弧の中心はy=√3(x-a)上の第4象限にあり、
直角三角形の辺の比より、円弧の半径を2、
円弧の中心のy座標を-1とすると、円弧は第1象限においてじゅうぶんな位置で単位円と直交できる。
∴a=2-√3
境界線の最小値=2a+4×(第1象限内の円弧)
=2a+4×2π×2×α/360
(α<30°)
=2(2-√3)+2πα/45
=4-2√3+2πα/45
0692132人目の素数さん
垢版 |
2018/12/16(日) 08:15:16.56ID:Q9cgcISo
>>592
交角の条件(90゚, 120゚)を満たす折れ線を考えてみた。

分岐点 (±x,0)
境界線と外円の交点 (±cosθ, ±sinθ)
屈曲点 (±(x+y/2), ±(√3)y/2)

(x+y/2)tanθ = (√3)y/2 より
 y = 2x/((√3)cotθ -1),
 z = 1 - (x+y/2)/cosθ,
 y+z = 1 - {(√3 -2sinθ)/((√3)cosθ - sinθ)}x

長さ2xの境界を共有するパーツの面積
 σ = π/2 - θ + (√3)xy/2,

4等分条件(σ = π/4) から
 (√3)xy/2 = θ - π/4,
 x = √[(θ-π/4)(cotθ -1/√3)],

L = 2x + 4(y+z)
が最小となるのは
 θ = 0.84809550
 π/3 - θ = 0.1991020512
 x = 0.13817309535
 y = 0.52395618804
 z = 0.39500533920
 y+z = 0.91896152724
 L = 2x + 4y + 4z = 3.952192299669527
0693イナ ◆/7jUdUKiSM
垢版 |
2018/12/16(日) 08:23:19.34ID:Ax8rD0fg
>>691
野球のボール→ の縫い目にΗのような境界線を描くとし、
単位円の中心から分岐点までの距離をaとする。
xy平面における点(a,0)である。
点(a,0)を通り第1象限においてy=xを下から上に突っ切るように、中心を点(a,0)の右下30°の方向にとり、円弧を描き、単位円と直交させる。
円弧の中心はy=√3(x-a)上の第4象限にあり、
直角三角形の辺の比より、円弧の半径を2、
円弧の中心のy座標を-1とすると、円弧は第1象限においてじゅうぶんな位置で単位円と直交できる。
∴a=2-√3
境界線の最小値=2a+4×(第1象限内の円弧)
=2a+4×2π×2×α/360
(α<30°)
=2(2-√3)+2πα/45
=4-2√3+2πα/45
(tanα=1/2、cosα=1/√5)
0694イナ ◆/7jUdUKiSM
垢版 |
2018/12/16(日) 08:57:59.18ID:Ax8rD0fg
>>693
野球のボール→ の縫い目にΗのような境界線を描くとし、
単位円の中心から分岐点までの距離をaとする。
xy平面における点(a,0)である。
点(a,0)を通り第1象限においてy=xを下から上に突っ切るように、中心を点(a,0)の右下30°の方向にとり、円弧を描き、単位円と直交させる。
円弧の中心はy=√3(x-a)上の第4象限にあり、
直角三角形の辺の比より、円弧の半径を2、
円弧の中心のy座標を-1とすると、円弧は第1象限においてじゅうぶんな位置で単位円と直交できる。
∴a=2-√3
境界線の最小値=2a+4×(第1象限内の円弧)
=2a+4×2π×2×α/360
(α<30°)
=2(2-√3)+2πα/45
=4-2√3+2πα/45
(tanα=1/2、cosα=1/√5)
α≒26.56505119
境界線の最小値=4-2√3+2π(26.56505119)/45
=4-2√3+π(26.56505119/22.5
=4.24507926
0695132人目の素数さん
垢版 |
2018/12/16(日) 09:25:31.95ID:WRFoa1F9
∠A=90°の直角三角形ABCにおいて、AからBCに下ろした垂線の足をDとする。線分AB, AC上にそれぞれ点E, Fを、AD=AE=AFを満たすように取る。
∠ABCの二等分線と線分DEの交点をU、∠ACBの二等分線と線分DFの交点をVとする。
また、点Eから直線DFに下ろした垂線の足をX、点Fから直線DEに下ろした垂線の足をYとする。
このとき、3直線EF, UV, XYは互いに平行であるか1点で交わることを示せ。
0697132人目の素数さん
垢版 |
2018/12/16(日) 11:41:52.76ID:WjpuEhRa
>>695
AD = 1とし∠BAD = θとおく。
BE = 1/cosθ -1、BD = tanθにより
EU/UD = EB/BD = tanθ/2。
同様にしてFV/VD = tan(π/2 - θ)/2。
∴EU/UD DV/VF = tanθ/2 cot(π/2 - θ)/2 。
∠DEF = ∠DAF/2 = π/2-θ/2、∠XEF=π/2 - ∠XFE = π/2 - θ/2より∠XED = π/4。
よってFX/XD = tan(π/2 - θ/2)/1 = cotθ/2。
同様にしてEY/YD = cot(π/2-θ)/2。
以上により
EX/XD DY/YF = EU/UD DV/VF。
よってチェバの定理の逆により主張は成立。
0698132人目の素数さん
垢版 |
2018/12/16(日) 15:19:26.92ID:WRFoa1F9
こういう問題は初等幾何的に解きたいものだね。

三角形ABCにおいて、内心をIとする。また、∠B内の傍接円と辺ACの接点、∠C内の傍接円と辺ABの接点をそれぞれD,Eとする。そして、直線BDとCEの交点をPとする。
線分AQの中点が点Iとなるように点Qを取るとき、直線PQは線分BCを二等分することを示せ。
0699イナ ◆/7jUdUKiSM
垢版 |
2018/12/16(日) 15:33:32.47ID:Ax8rD0fg
>>694
>>696下まわれると思う。
野球のボール→ の縫い目にΗのような境界線を描くとし、単位円の中心からx軸上にある分岐点までの距離をaとする。
点(a,0)を通り第1象限においてy=xを下から上に突っ切るように、中心を点(a,0)の右下30°の方向にとり、円弧を描き、単位円と直交させる。
円弧の中心はy=-(1/√3)(x-a)上の第4象限にあり、円弧の半径をr(>2)、円弧の中心のx座標をbとすると、円弧の中心のy座標は、
y=-(a-b)/√3
円弧の方程式は、
(x-b)^2+{y+(a-b)/√3}^2=r^2
境界線の最小値=2a+4×(第1象限内の円弧)
=2a+4×2πrθ/360°
接弦定理よりθ=
単位円x^2+y^2=1と円弧
(x-b)^2+{y+(a-b)/√3}^2=r^2から交点の座標は( , )
0702イナ ◆/7jUdUKiSM
垢版 |
2018/12/16(日) 17:22:20.55ID:Ax8rD0fg
(x-b)^2+{y+(a-b)/√3}^2=r^2と単位円の交点の座標を考えてみます。
>>699
1+2bx+b^2+2(a-b)y/√3+{(a-b)^2}/3=r^2
(1+2bx+b^2)√3/2(a-b)+y+(a-b)/2√3=(r^2)√3/2(a-b)
y=(r^2)√3/2(a-b)-(1+2bx+b^2)√3/2(a-b)-(a-b)/2√3
0<a<b<r
x^2+{(r^2)√3/2(a-b)-(1+2bx+b^2)√3/2(a-b)-(a-b)/2√}^2=1
解けると思うけど。
0703132人目の素数さん
垢版 |
2018/12/16(日) 18:05:56.73ID:Nny3soWk
>>698
もう初等幾何は忘れた。
Pはナゲール点で重心座標は(s-a, s-b, s-c)。
QはA(2s, 0, 0)と内心I(a,b,c)を2:1に外分する点だからQ(a-s, b,c)。
BCの中点M(0,1,1)で行列
0 1 1
s-a s-b s-c
a-s b c
の第3行を第2行に足せば第1行と第2行は平行となるため特にその行列式は0。
よってP, Q, Mは同一直線上にある。
0704イナ ◆/7jUdUKiSM
垢版 |
2018/12/16(日) 20:09:56.08ID:Ax8rD0fg
>>702図を描くと少しわかった。
円弧の中心は、
(b,(a-b)/√3)
円弧の半径rは、
r=2(b-a)/√3
b-a=r(√3)/2
円弧の方程式は、
(x-b)^2+{y-(a-b)/√3}^2=(4/3)(b-a)^2
単位円との交点は、
x^2-2bx+b^2+y^2-2(a-b)y/√3+(a-b)^2/3=(4/3)(b-a)^2
x^2-2bx+b^2+(1-x^2)-2(a-b)√(1-x^2)/√3+(a-b)^2/3=(4/3)(b-a)^2
-2bx+b^2+1-2(a-b)√(1-x^2)/√3+(a-b)^2/3=(4/3)(b-a)^2
-2bx+b^2+1+(a-b)^2/3=2(a-b)√(1-x^2)/√3+(4/3)(b-a)^2
2bx-b^2-1-{r(√3)/2}^2/3=2{r(√3)/2}√(1-x^2)/√3-(4/3){r(√3)/2}^2
2[a+{r(√3)/2}]x-[a+{r(√3)/2}]^2-1-{r(√3)/2}^2/3=2{r(√3)/2}√(1-x^2)/√3-(4/3){r(√3)/2}^2
辺々を二乗すると、
4[a+{r(√3)/2}]^2・x^2-4[a+{r(√3)/2}][1+{r(√3)/2}^2/3]x+[1+{r(√3)/2}^2/3]^2=(4/3)(3/4)r^2(1-x^2)-2・2[{r(√3)/2}√(1-x^2)/√3]・(4/3){r(√3)/2}^2+(4/3)^2{r(√3)/2}^4
(4a^2+ar√3+3r^2)x^2-4[a+r(√3)/2}][1+{r(√3)/2}^2/3]x+[1+{(3/4)r^2}^2/3]^2=(4/3)(3/4)r^2(1-x^2)-2・2[{r(√3)/2}√(1-x^2)/√3]・(4/3){r(√3)/2}^2+(4/3)^2{r(√3)/2}^4
(4a^2+ar√3+3r^2)x^2-4[a+r(√3)/2}][1+(3/4)r^2/3]x+[1+{(3/4)r^2}^2/3]^2=(4/3)(3/4)r^2(1-x^2)-2・2[{r(√3)/2}√(1-x^2)/√3]・(4/3){r(√3)/2}^2+(4/3)^2{r(√3)/2}^4
(4a^2+ar√3+3r^2)x^2-(4a+2r√3)(1+r^2/4)x+[1+{(3/16)r^4]^2=r^2(1-x^2)-2r√(1-x^2)・r^2+r^4
(4a^2+ar√3+3r^2)x^2-(4a+2r√3)(1+r^2/4)x+[1+{(3/16)r^4]^2=r^2-r^2・x^2-2r√(1-x^2)・r^2+r^4
(4a^2+ar√3+3r^2)x^2-(4a+2r√3)(1+r^2/4)x+[1+{(3/16)r^4]^2-r^2+r^2・x^2+2r√(1-x^2)・r^2-r^4
(4a^2+ar√3+4r^2)x^2-(4a+2r√3)(1+r^2/4)x+[1+{(3/16)r^4]^2-r^2+2r√(1-x^2)・r^2-r^4=0
境界線の最小値=2a+4×2πrθ/360°≒3.9……?
0706イナ ◆/7jUdUKiSM
垢版 |
2018/12/16(日) 23:07:35.77ID:Ax8rD0fg
>>704難解。
a=1/7
b=1/7+2√3
r=4
のとき、円弧の中心は、
(1/7+2√3,-2)
円弧の中心角≒42°-30°
=12°(←カン)
境界線の最小値=2/7+4・2π・4(12/360)
=2/7+16π/15
=3.63674645
0708イナ ◆/7jUdUKiSM
垢版 |
2018/12/16(日) 23:59:19.47ID:Ax8rD0fg
>>706ぐははは……
>>707そうともよ、だれもこれよりちっさぁはできんだろ。早く正解ですって言っておくれ。
ヾ、、,,゙
((~e~)え?
(っц)~ちょっと
「 ̄ ̄ ̄]ちっさ
■/_UU\■すぎたこ? でもこの長さ、この角度しかない思うんよ。
0710イナ ◆/7jUdUKiSM
垢版 |
2018/12/17(月) 02:41:44.10ID:1XQ825eN
>>708再戦。
確認だけど、>>642は正解じゃないのね?
a=1/8
b=1/8+2√3
r=4
のとき、円弧の中心は、
(1/8+2√3,-2)
円弧の中心角=43°-30°
=13°
境界線の最小値=2×(1/8)+4・2π・4(13/360)
=1/4+52π/45
≒3.88028484
0711132人目の素数さん
垢版 |
2018/12/17(月) 05:21:06.55ID:X9YWLOp7
>>710
 >>642 は正解です。

分岐点が (a,0)
円弧の中心 (b, -(b-a)/√3)
円弧の半径 r = 2(b-a)/√3,
とします。
a = 1/8, b-a = 2√3, r=4 のとき、
円弧の中心は (1/8 + 2√3, -2)

円弧と外円の交点を (cosφ, sinφ)
とすると
φ = 0.831377808560176 = 47.6344396113334°
cosφ = 0.673858391655747
sinφ = 0.738860519986776

また、θ+30°= 43.2132078553074°

円弧の中心角は
θ = 0.23061398182545 = 13.2132078553074°

(∴ 円弧と外円の交角 φ + (θ+30゚) = 90.8476474666384°)

長さ2aの境界線を共有するパーツの面積は
σ(a) = {中心角 π-2φ の扇形} + 2・(底辺bの) - 2・(三日月形D)
 = (π/2 -φ) + a・sinφ - rr(θ-sinθ)
 = 0.73941851823 + 0.09235756500 - 0.03261900525
 = 0.7991570780
 > 0.7853981634
 = π/4
となり、4等分条件を満足しません。
0713132人目の素数さん
垢版 |
2018/12/17(月) 06:13:31.90ID:BTBPoTbM
そもそもプラトーの法則ってどうやって証明するんだ?
0714132人目の素数さん
垢版 |
2018/12/17(月) 06:37:22.81ID:X9YWLOp7
境界線の長さは
L = 2a + 4rθ
 = 1/4 + 4・4・0.23061398182545
 = 3.9398237092

これは >>710 の値より長いです。

>>713
面積を保つような微小な変形を考えて、境界線長さを短かくできるかどうか見るんぢゃ?
「変分法」とかいう…

>>713
0716132人目の素数さん
垢版 |
2018/12/17(月) 07:31:23.11ID:RRU4PE62
面白い問題かどうかは怪しいけど数学コンテストの問題で想定解がわからないから投稿
出典は Rioplatense Olympiad 2018 level 3 p3
問題のURLは下記
https://artofproblemsolving.com/community/c6t177f6h1752225_abc_divides_sigma_a12_sigma_a23_sigma_a11004_coprime

このリンク先で既に誰かが回答も提示しているけど、
明らかに試験会場でできる方法じゃないから想定解じゃないハズ
(確かにグレブナー基底を計算すれば数学的には解けるんだけど 計算量的にありえない)

リンク先をみればいいけど問題を一応ここにも書いておくよ
[問題]
a+b+c が a^12+b^12+c^12, a^23+b^23+c^23, a^11004+b^11004+c^11004
の3数を同時に割り切るようなgcd(a,b,c)=1なる正の整数a,b,cの組をすべて求めよ

どなたか叡知にあふれた方・・・助けてください
0717132人目の素数さん
垢版 |
2018/12/17(月) 08:00:53.69ID:7Q25yQ56
直線解よりも円弧解が短いことはわかったけど
円弧のときが最小ってどうやって証明するんだろ?
0718132人目の素数さん
垢版 |
2018/12/17(月) 09:07:21.77ID:4q4lQ+sg
P(x)を整数係数多項式とする。任意の整数nに対して
P(n)P(n-1)=P(n^2)
が成り立つものを全て求めよ。
0719132人目の素数さん
垢版 |
2018/12/17(月) 10:14:27.76ID:RRU4PE62
>>718
P(x)P(x-1)=P(x^2) ...(♯) を満たす整数係数多項式P(x)を求めればよい
P(x)が (♯)を満たしていると仮定する.
αをPの根とすれば,α^2もPの根である.
よって,ある異なる自然数i,jが存在して,α^(2^i)=α^(2^j),
とくに α=0 または |α|=1 がいえる.
また, (α+1)^2も根であることから, α = -1 または |α+1| = 1 がいえる
つまり, αがPの根ならば, α=0, -1 さもなくば |α|=|α+1|=1 がいえる.
α = 0 のときは (0+1)^2 = 1 も根であるが, それはさっきの関係式を満たさない
α = -1 のときは α^2 = 1も根であるが, 同様の理由で不適である
また, |α+1| = |α| = 1 のときは
α =ω, ω^2 であることはすぐわかる(ω: 1の原始3乗根)
以上より, α∈{ω, ω^2} であることがわかった.
よって, P(x)=c*(x^2+x+1)^m
を満たす非負整数mおよび整数定数cが取れる.
これを(♯)に代入すれば c = c^2 ⇔ c(c-1)=0 を得る
よって求める整数係数多項式は以下の形に限る:
P(x)=0 (零多項式), P(x)=(x^2+x+1)^m (m:非負整数)
これらが条件を満たすことはすぐ確認できる
0720イナ ◆/7jUdUKiSM
垢版 |
2018/12/17(月) 10:19:53.90ID:1XQ825eN
>>710
等分条件からaなりbなりを求めるところが難解なんだと思う。正解も空白だし、いきなりb=0.122……だし。積分じゃなくて扇形から三角形を引くのかも。
そこは不確定な少数より、
1/7なり1/8なりの分数で然るべきと思った。
角度はせいぜい0.5°までだと思う。分度器持って測ってるわけじゃないもんで。
43°-30°=13°が限界と感じた。
0721132人目の素数さん
垢版 |
2018/12/17(月) 12:21:58.05ID:XTGID1Us
>>642が正解で良いんじゃないかな
自分がやってみたのは、角度条件から
b=(sin(π/3)-sin(π/3-θ))/sin(2π/3-θ)
R=(1-b*cos(π/3-θ))/sinθ
の関係式を立ててから等分条件からθの方程式を作る方法で、解いてみたらだいたい同じ結果になった
θ=0.198522403464295
b=0.122010156676718
R=4.66154271422005
L=2b+4Rθ=3.94570296726719

ちなみに、このθはおおよそ11.3745°
0722イナ ◆/7jUdUKiSM
垢版 |
2018/12/17(月) 14:00:11.86ID:1XQ825eN
>>720
少数aを探る。
2a+4・2π・4(13/360)=3.9457とすると、
a=0.157707578……
a=0.1577のとき、
2a+4・2π・4(13/360)
=2(0.1577)+4・2π・4(13/360)
=0.3154+52π/45
=0.3154+3.63028484
=3.94568484
a=0.157のとき、
2a+4・2π・4(13/360)
=0.314+52π/45
=0.314+3.63028484
=3.94428484
a=0.15のとき、
2a+4・2π・4(13/360)
=0.3+52π/45
=0.3+3.63028484
=3.93028484
0724132人目の素数さん
垢版 |
2018/12/17(月) 14:15:42.92ID:/nKYkp7v
R=4、θ=13° が出てくる理由が不明よね
ちゃんと計算してないからそうなる
0725132人目の素数さん
垢版 |
2018/12/17(月) 14:33:29.34ID:0iM+q3pB
彼の哲学は勘でも答えが出れば良い、論拠は問わないなので。
まぁ実際それでピタッと答えが合うときがあるかもしれないし。
奇跡のドンピシャを期待!
0726132人目の素数さん
垢版 |
2018/12/17(月) 15:07:11.57ID:jwfzbwkP
Wolframの入力これ間違っているかな?

最小値が出てこない

minimize(y) where (y=2*b+4*r*α and r=sin(θ)/(cos(θ)-1/2) and (pi/2-θ)/2+b*sin(θ)/2 - r^2*α/2=pi/8 and 0<b<1 and 0<θ<pi/2 and 0<α<2*pi)
0727イナ ◆/7jUdUKiSM
垢版 |
2018/12/17(月) 15:17:39.55ID:1XQ825eN
>>722最後の戦い。
a=0.15770758のとき、
2a+4・2π・4(13/360)
=2(0.15770758)+4・2π・4(13/360)
=0.31541516+52π/45
=0.31541516+3.63028484
=3.9457
端数が出ない。
近似じゃない。
これが正解だろ。
0729132人目の素数さん
垢版 |
2018/12/17(月) 21:57:12.17ID:0vY43FxT
ウルフラムってどうやってつかうの?
0731132人目の素数さん
垢版 |
2018/12/18(火) 01:49:33.01ID:6+IKkUSu
単位円の面積を5等分する曲線(分岐あり)の長さの最小値を求めよ
という発展問題を考えていて、
中心に4つの1/12周円弧で囲まれた面積π/5の図形を配置した場合の解として
4+2√((π/3+1-√3)π/5)=4.88997199712511968884147723267088412125423513580167589638622110207064013869324939879113289311720233605526039125851…
というものを得ている。
これより良い解は存在するか? というのが問題。
0732132人目の素数さん
垢版 |
2018/12/18(火) 02:02:37.93ID:TqVGX/9j
>>726

交角の条件 (120゚, 90゚) から
 α = 60°- θ  … (1)

また
 r = sinθ/(cosθ -1/2)  … (2)
 r = (1-bb)/(b√3),     (← これも使おう)
からrを消すと
 b = {1 - (√3)tan(θ/2)}/{1 + (√3)tan(θ/2)}  … (3)

・なお、
 tan(θ/2) = (1-b)/{(1+b)√3},
 sinθ = (√3)(1-bb)/{2(1+b+bb)},
 cosθ = (1+4b+bb)/{2(1+b+bb)},  >>641

4等分条件
 (π/2 -θ) + b・sinθ - rr(α-sinα) = π/4,
から
b = {θ - π/4 + rr(α-sinα)}/sinθ  … (4)

(1)〜(4)から

(1-sqrt(3)*tan(th/2))/(1+sqrt(3)*tan(th/2)) = (th-pi/4 + (sin(th)/(cos(th)-1/2))^2 *(pi/3-th-sin(pi/3-th))/sin(th)

これでやってみては?

結果
 θ= 0.8486751477323028478 = 48.6255041427025895976°
したがって
 α = π/3 - θ = 0.198522403464294898354 = 11.3744958572974104024°
 b = 0.1220101566767175917
 r = 4.6615427142200527
0733132人目の素数さん
垢版 |
2018/12/18(火) 02:25:55.31ID:TqVGX/9j
>>726

交角の条件(プラトーの法則)を使わずに解くことも可能とは思います。
ただし上の (1)〜(3) は使えません。
4等分条件(4)は必ず満たさなくてはいけないので、計算がかなり煩雑になりそうな予感…
0734132人目の素数さん
垢版 |
2018/12/18(火) 04:14:52.39ID:QUQCnHzh
>>730
この手の問題をちゃんと厳密に証明するのは相当むずい。
一般には変分法というのを用いる。
問題を
――
(f1,f2,f3,f4,g)で
・fi:[0,1] -> D、g:[0,1] -> D
・fi(0)∈∂D
・f1(1) = f2(1) = g(0)、f3(1) = f4(1) = g(1)
・fi、giはいたるところ可微分でその囲む4つの部分の面積はπ/4
をみたす5つ組の集合Xにおいて
・S=fi,gの長さの和
を最小とする(fi,g)を決定せよ。
――
とする。(もっとたくさんの分岐を考えたければそれに応じて条件緩める。今は上の分岐に限定する。)
まずXにノルムをいれてノルム空間に入れて完備化する。
今はいたるところ一階微分可能の条件下で完備化だからソボレフ空間というのを使う。これをYとする。
https://ja.wikipedia.org/wiki/%E3%82%BD%E3%83%9C%E3%83%AC%E3%83%95%E7%A9%BA%E9%96%93
p=2としておけば自己双対空間になるのでYにおいてはSは最小値をもつ。
その最小値は極小値でもあるのでオイラーラグランジュ方程式を立てることができる。
https://ja.wikipedia.org/wiki/%E3%82%AA%E3%82%A4%E3%83%A9%E3%83%BC%EF%BC%9D%E3%83%A9%E3%82%B0%E3%83%A9%E3%83%B3%E3%82%B8%E3%83%A5%E6%96%B9%E7%A8%8B%E5%BC%8F
これによって最小値を与える(f1,f2,f3,f4,g)がプラトーの法則を満たすことがわかる。
するとそこからこれは元のXの元であることがわかる。
これは膨らませた空間Yにおいて最小値を与えたのだからもとの空間Xにおいても最小である。
――
0735132人目の素数さん
垢版 |
2018/12/18(火) 04:15:12.30ID:QUQCnHzh
という流れ。
もう少し細かくいうと
――
・fi,gの端点を固定しての変分を考えるとfi,gは円弧または線分となる。
http://mathematical.jp/analytical_mechanics/isoperimetric_problem.html
・f1のみの変分を考えればf1と外周のなす角が90°がでる。
(これはこのスレで類題をだれかが出題してた。めっちゃムズイけど上よりは簡単。すでに円弧であることは示されてるので。)
・f1、f2、gのみの変分を考えればf1、f2、gのなす角が120°がでる。
(上に準ずる。)
――
数学科でしかも院生レベルか、少なくとも関数解析系の研究室でないと厳密に全部理解するのはムズイと思う。
なにせ変分原理は2,300年くらい?の昔からあるみたいだけど厳密に定式化されたのは20世紀に入ってからでHilbert空間論とか出てくる。
しかし逆に言うとsobolev空間云々をおいとけば、それ以外のオイラーラグランジュ方程式の理論とかは単に部分積分法さえ知ってれば理解できると思う。
上のHPにやり方のってます。
からのプラトーの原理出すとこもムズイけど上の話からみるとぐっと簡単になります。
前にだれかが出題したとき解答作ったんだけど恐ろしく長くなって解答書くのやめた。
さがせばネットでも解答あると思う。見つけてないけど。
0736132人目の素数さん
垢版 |
2018/12/18(火) 04:44:14.44ID:ThGE+mq7
>>735
元の問題の曲線を可微分函数なものに限定して考えるのは妥当なの?
あるいはそのようなものに限定されることを示せるの?
0737132人目の素数さん
垢版 |
2018/12/18(火) 04:49:17.26ID:TqVGX/9j
>>731
交角条件 (120゚, 90゚) も満たすから、それが最小値ぢゃね?(プラトーの法則)

分岐点 (±b,0) (0,±b)
境界円の半径r
境界円弧の中心角をαとすると

sin(α/2) = b/(r√2),
1 - cosα = (b/r)^2,
中央のパーツの面積は
σ= 2rr(α-sinα) + 2bb
 = 2bb(α-sinα+1-cosα)/(1-cosα)
 = 2rr(α-sinα+1-cosα),
5等分条件 (σ=π/5) から
 b/sin(α/2) = √{π/(5(α-sinα+1-cosα))}
 r = √{π/(10(α-sinα+1-cosα))}

この条件の下で境界線の長さ
 L = 4(1-b+rα)
が最小になるのは(αで微分して)
 α = π/6 = 30°
 sinα = 1/2,
 cosα = (√3)/2,
 sin(α/2) = (√3 -1)/√8,
したがって
 r = √{π/(5(π/3 +1-√3))} = 1.4119961813351850581103676
 b = sin(α/2)√{2π/(5(π/3 +1-√3))} = 0.516826472415296562696543
となって >>731 と一致
0738132人目の素数さん
垢版 |
2018/12/18(火) 05:05:13.77ID:62nHVSjU
>>736
微分可能なものに限定するのが妥当かどうかは微妙。
それは “長さ” の定義をどうするかに依るから。
曲線の長さを
(1) ∫√(x’^2 + y’^2)dt
(2) inf {Σ|pi - p(i+1)|}
のどっちで定めるかは議論の残るところだから。
(2)の方が広いのだけど、これは一次元でしか利用できない方法で3次元空間のなかにうめこまれた2次元の面の極小問題ですらシュバルツの反例というのがあって “部分空間の大きさ” を考える一般的な定式化としては上手くいかない。
ので普通は “部分空間の大きさ” は(1)の方の意味で測るし、当然その場合にはいたるところ可微分を要求することになる。
多分曲線の話なら(2)の意味で問題定式化できるとは思う。(と数学辞典に書いてあった記憶もある。)
でも可微分なものの全体はL^2の距離で稠密なのでそのこと利用すれば(1)の場合に結局還元できるとは思う。
でも思ってるだけでその証明は知りません。
挑戦してみては?
0740132人目の素数さん
垢版 |
2018/12/18(火) 10:13:20.52ID:QJ+226c7
このスレレベルたけえな
0742132人目の素数さん
垢版 |
2018/12/18(火) 11:34:11.21ID:s3JskfYr
>>733
4等分条件だけで変数を減らせないかなと思ってもいるんですが、やっぱり無理ですかね?
0744132人目の素数さん
垢版 |
2018/12/18(火) 11:45:23.26ID:2lmvarOu
>>743
よく見てないけどaを近似で与えている時点で答えがぴったりと主張する権利がないのでは
0745132人目の素数さん
垢版 |
2018/12/18(火) 13:17:28.40ID:wjFrEy5r
Prelude> let a = 0.15770758
Prelude> 2*a + 8*pi*4*13/360
3.9457000041482058
小数第5位までしかあいませぬ。
0748イナ ◆/7jUdUKiSM
垢版 |
2018/12/18(火) 17:15:06.24ID:wSEDrUMl
>>743
やっぱりちゃんと4分割した面積=π/4として単位円の中心と分岐点の距離aを自然に出さんといかんね。

bのy座標と円弧の半径rは自然に出た。

扇形と三角形を足して三日月形を引くのかな。もうちょい待ってよ。あきらめちゃいないから。もうちょいやればたぶん面積から妥当なaが出ると思う。
0749132人目の素数さん
垢版 |
2018/12/18(火) 19:21:10.93ID:NsuOluDp
@√XのXが平方数以外の自然数なら、√Xが無理数であることを証明せよ。
A√XのXが分数の場合、いかなる場合なら√Xが無理数になるか。またそれを証明せよ。
0751132人目の素数さん
垢版 |
2018/12/19(水) 01:56:25.45ID:5zoTD2o3
>>749
(1)
 Xは平方数でないから、或る素数pのべき指数は奇数。
いま √X が有理数だったと仮定する:
 √X = q/r (q,r∈N かつ(q,r)=1)
∴ Xrr = qq ∈ N
素数pのべき指数を見ると、左辺は奇数、右辺は偶数(0も含め) となる。
∴ Nにおける素因数分解は一意的(UDF)でないことが分かる。
0752132人目の素数さん
垢版 |
2018/12/19(水) 01:59:26.07ID:5zoTD2o3
>>743

r = 4,
α = 13°= (13/180)π = 0.226892802759262845
L = 3.9457
が正解じゃない理由

上記から
 sinα = 0.224951054343864998
 α - sinα = 0.001941748415397847
 a = 0.15770757792589724

分岐点 〜 交点 間の直線距離は
 2r sin(α/2) = 0.90562571014325

境界円と外円との交点を (cosφ, sinφ) とおく。
 φ = 0.8644882263708342 = 49.5315268097989°
 cosφ = 0.64902953921139422
 sinφ = 0.76076320706974604

境界円の中心は
 (3.7419350454197931, -1.7757571515447414)

線分aと円弧の交角@分岐点 116.355494663976463°< 120°
円弧と外円の交角 91.1129785262246°> 90°

長さ2aの境界線を共有するパーツの面積は
σ(a) = {中心角π-2φの扇形} + 2・(底辺aの) - 2・(三日月形D)
 = 0.70630810042406242 + 0.11997812276210747 - 0.03106797464636555
 = 0.79521824853980434
 > 0.78539816339744831
 = π/4,
となり、4等分条件を満足しません。
0753132人目の素数さん
垢版 |
2018/12/19(水) 02:16:11.61ID:5zoTD2o3
>>752
1行抜けてた…

σ(a) = {中心角π-2φの扇形} + 2・(底辺aの) - 2・(三日月形)
 = (π/2 -φ) + a・sinφ -rr(α-sinα)
 = …
0754132人目の素数さん
垢版 |
2018/12/19(水) 04:02:06.84ID:yZ9IGW8e
>>716
猛者の方々, この問題の謎を解いてくれませんか?私では無理そうです
これはコンテストの問題で計算機使用不可なハズなので
11004 の謎を解いて 使える恒等式をみつけないといけないとおもいます
0755イナ ◆/7jUdUKiSM
垢版 |
2018/12/20(木) 00:40:57.19ID:PIAZ12pp
>>748
そうか、r、4.66もあんのか。
~ 、、,,
(~-_-)円弧とx軸の交点
(っφ)を(a,0)として、
接線とx軸の交点(b,0)が知りたい。
扇形から2つから三角形2つを引けばπ/8になる、ってことですよね?
円弧の中心角をθとして、
60°-θ、半径1の扇形と、θ、半径rの扇形と、
(60°-θ)と(30°+θ)の直角三角形(斜辺b)と、
底辺(b-a)×高さ(r/2)の三角形。
弧度法はこんがらがるんでなしでお願いします。
0756132人目の素数さん
垢版 |
2018/12/20(木) 01:17:15.54ID:urn695Ce
>>754
できた。
――
a+b+c = MN、(N,abc) = 1、radM | abc とする。

M ≠1 とし p|M を素因子とする。 p|a としてよい。
b ≡ -c (mod p) (∵ a+b+c ≡ 0 (mod p)) により 2b^12 ≡ 0 (mod p) (∵ a^12 + b^12 + c^12 ≡ 0 (mod p))。
∴ p = 2、b≡c≡1 (mod p)。
∴ v_2(M) ≦ v_2(a^12 + b^12 + c^12) = 1。
∴ M = 2。
以上により M|2。

一方
det ([[a,b,c],[a^12,b^12,c^12],[a^23,b^23,c^23]]) ≡ 0 (mod N)。
det ([[1,1,1],[a^11,b^11,c^11],[a^22,b^22,c^22]]) = 0 (mod N)、
∴ a^11≡b^11 (mod N) としてよい。(∵ Vandermonde。)
∴ 0 ≡ a^12 + b^12 + c^12 ≡ a^11(a+b) + c^12 ≡ c(-a^11+c^11) (mod N)、
∴ a^11≡c^11 (mod N)。
∴ 0 ≡ a^11004 + b^11004 + c^11004 ≡ a^4 + b^4 + c^4 (mod N)。
∴ 0 ≡ a^12 + b^12 + c^12 ≡ (a^4 + b^4 + c^4)^3 - 6(abc)^4 ≡ -6 (abc)^4 (mod N)。
∴ 6 ≡ 0 (mod N)。∴ N|6。

以下ry
0757132人目の素数さん
垢版 |
2018/12/20(木) 01:25:41.17ID:urn695Ce
あ、
>∴ a^11≡b^11 (mod N) としてよい。(∵ Vandermonde。)
の行嘘だ。
でなおします orz。
0758132人目の素数さん
垢版 |
2018/12/20(木) 01:56:24.09ID:urn695Ce
>>756
訂正

後半のパラグラフ
p|N、v_p(N) = eとおく。
det ([[a,b,c],[a^12,b^12,c^12],[a^23,b^23,c^23]]) ≡ 0 (mod p)。
det ([[1,1,1],[a^11,b^11,c^11],[a^22,b^22,c^22]]) = 0 (mod p)。
∴(a^11-b^11)(b^11-c^11)(c^11-a^11) (nod p) (∵ Vandermonde。)
∴ a^11≡b^11 (mod p) としてよい。
∴ 0 ≡ a^12 + b^12 + c^12 ≡ a^11(a+b) + c^12 ≡ c(-a^11+c^11) (mod p)、
∴ a^11≡c^11 (mod p)。
∴ 0 ≡ a^11004 + b^11004 + c^11004 ≡ a^4 + b^4 + c^4 (mod p)。
∴ 0 ≡ a^12 + b^12 + c^12 ≡ (a^4 + b^4 + c^4)^3 - 6(abc)^4 ≡ -6 (abc)^4 (mod p)。
∴ 6 ≡ 0 (mod p)。
p = 2のときa,b,cは全て奇数よりv_2(N) ≦v_2(a^12+b^12+c^12)≦0。(∵ a,b,c ≡ 1(mod 8))∴e = 1。
p = 3のときa,b,cは全て3と互いに素よりv_3(N) ≦v_3(a^12+b^12+c^12)≦1。(∵ Z/9Zの乗法群の位数は6だからa,b,c≡1 (mod 9))∴e = 1,2。
∴ N|3。
0759132人目の素数さん
垢版 |
2018/12/20(木) 03:22:35.00ID:b5FpuB9M
>>758
なるほど単純に行列式考えればよかったですね
12, 23 から1引くと,11,22あたりになるのが臭いと感じてましたが
これはなにげに思いつきませんでした ありがとうございました!
0760132人目の素数さん
垢版 |
2018/12/20(木) 05:24:03.82ID:vAHMMPVp
5等分問題について、>>731が提示したのはこれ
http://imgur.com/91bVONA.png

分け方を変えるともう少し小さくなる
http://imgur.com/GYiVA6F.png

この図の境界は全て直線であり、節点における角度は120°にしているけど円との交点は垂直になっていない
なので、境界線を円弧で考えれば更にもう少し良くなるはず
ただ、それを計算するのは骨が折れそう
0761132人目の素数さん
垢版 |
2018/12/20(木) 10:32:32.92ID:5thDTonU
>>760

分岐点
 (a-1,0) (a-1+b/2, ±(√3)b/2)
外円との交点
 (cosφ, sinφ) (x1,y1)
とすると
 c = cosφ - (a-1+b/2)
 y1 = (c-x1)√3,
 L = a + 2(b+c+d)


a = 0.832249108
b = 0.339584229
c = 0.953736949
d = 0.735197942
x1 = -0.3655577475
y1 = 0.9307886620
φ = 0.298501778 = 17.10289204°
cosφ = 0.9557781717

a-1 +b/2 = 0.002041223

L = 4.889287345
となってチョト長い。
計算間違えたかな…
0764132人目の素数さん
垢版 |
2018/12/20(木) 10:55:50.94ID:zDwt6WPy
>>761
分岐点間にある境界線は直線?
0766132人目の素数さん
垢版 |
2018/12/20(木) 11:18:24.01ID:NkmRjcp+
>>762
f(a,b,c,n):=(1+a+b+1+c+a*b)^n/8
とおくとき
(f(1,1,1,n) + f(1,1,-1,n) + f(1,-1,1,n) + f(1,-1,-1,n)+f(-1,1,1,n) + f(-1,1,-1,n) + f(-1,-1,1,n) + f(-1,-1,-1,n))/6^n
= ((1+1+1+1+1+1*1)^n + (1+1+1+1+(-1)+1*1)^n + (1+1+(-1)+1+1+1*(-1))^n + (1+1+(-1)+1+(-1)+1*(-1))^n
+ (1+(-1)+1+1+(-1)*1)^n + (1+(-1)+1+1+(-1)+(-1)*1)^n + (1+(-1)+(-1)+1+1+(-1)*(-1))^n + (1+(-1)+(-1)+1+(-1)+(-1)*(-1))^n)/8 /6^n
=(6^n + 4^n + 3・2^n)/(8・6^n)
0767132人目の素数さん
垢版 |
2018/12/20(木) 11:25:50.88ID:aTZ4Qgc1
>>762
nが1から8までで積が平方数になる場合の数をかぞえてみた。

> sub<-function(x){
+ y=sqrt(prod(x))
+ floor(y)==y
+ }
>
> sim<-function(n){
+ arg=list()
+ for(i in 1:n) arg[[i]]=1:6
+ gr=do.call(expand.grid,arg)
+ sum(apply(gr,1,sub))
+ }
>
> sim=Vectorize(sim)
>
> sim(1:8)
[1] 2 8 38 200 1112 6368 37088 218240
>
>
0768132人目の素数さん
垢版 |
2018/12/20(木) 11:30:09.44ID:5thDTonU
>>764
そうです。まづは直線で計算しますた。
分岐点の交角は 120°
外円との交角は 90゚-φ(右)、98.5580885°(左上)、90゚(左)
0769イナ ◆/7jUdUKiSM
垢版 |
2018/12/20(木) 11:32:35.42ID:PIAZ12pp
>>755円弧とx軸の交点 を(a,0)、接線とx軸の交点を(b,0)とし、扇形から2つから三角形2つを引けばπ/8になる。
円弧の中心角をθとすると、
中心角60°-θ、半径1の扇形は、
π(60°-θ)/360°――@
中心角θ、半径rの扇形は、
πr^2・θ/360°――A
(60°-θ)と(30°+θ)の直角三角形(斜辺b)は、
(1/2)b・sin(60°-θ)――B
底辺(b-a)×高さ(r/2)の三角形は、
(b-a)r/4――C
単位円を分岐ありの最小の境界線で4分割した面積のうち、原点を含まない領域をx軸で二分した半分は、
@+A-B-C=π/8
π(60°-θ)/360°
+πr^2・θ/360°
-(1/2)b・sin(60°-θ)
-(b-a)r/4
=π/8
0770132人目の素数さん
垢版 |
2018/12/20(木) 13:10:05.45ID:dv0n+7uF
>>766
>>767

ぴったり、一致するのを確認。

> c(2, 8, 38, 200, 1112, 6368, 37088, 218240)/(6^(1:8))
[1] 0.3333333 0.2222222 0.1759259 0.1543210 0.1430041 0.1364883 0.1324874
[8] 0.1299345
> n=1:8
> (6^n + 4^n + 3*2^n)/(8*6^n)
[1] 0.3333333 0.2222222 0.1759259 0.1543210 0.1430041 0.1364883 0.1324874
[8] 0.1299345
0771132人目の素数さん
垢版 |
2018/12/20(木) 14:00:51.73ID:N8RFhZeZ
ユークリッドの公理・公準のみの状態から三平方の定理を導き出すまでに最短の経路は何か
0772132人目の素数さん
垢版 |
2018/12/20(木) 16:34:23.77ID:6fQj7XXR
>>655
>よって
N(B)=∫[1-0,B+0] x df(x)

がよく分からないが, 間違いである.
N(B)のオーダーはB^nです.
0773132人目の素数さん
垢版 |
2018/12/20(木) 16:43:59.25ID:6fQj7XXR
>>762
まさに私の想定解です.
別解を:

先ず2, 3, 6のみ出る特殊なサイコロをk回振った時, 出た目の積が平方数である確率をq[k]とせよ.
q[k]=(1/3)(1-q[k-1]) → q[k]=(-1/3)^k・(3/4)+1/4.

普通のサイコロをn回振り, 出た目の積が平方数である確率を P[n], 平方数掛ける2か3か6である確率をQ[n], 平方数掛ける5である確率をR[n]とせよ.
P[n]+Q[n]は5が偶数回出る確率なので,
P[n]+Q[n]=(1/2){(1/6+5/6)^n+(5/6-1/6)^n}
=1/2+(1/2)(2/3)^n ...@

P[n]+R[n]は出た目の積の2, 3の指数が偶数の確率なので,
P[n]+R[n]=Σ[k=0, n] (nCk)・(1/2)^n・q[k]
=(1/2)^n・{(1-1/3)^n・(3/4)+(1+1)^n・(1/4)}
=(1/3)^n・(3/4)+1/4 ...A

P[n]=(1/3)P[n-1]+(1/6)(Q[n-1]+R[n-1])なので,
@, Aより,
P[n] = (1/6)(P[n-1]+Q[n-1])+(1/6)(P[n-1]+R[n-1])
=(1/8){1+(2/3)^n+(1/3)^(n-1)}.
0774132人目の素数さん
垢版 |
2018/12/20(木) 17:34:40.77ID:cBgoWNlh
>>772
問題なんか混じってないか?

>B>0を定数とせよ.
>N(B)=#{(a_1,...,a_n):a_1,...,a_nは最大公約数が1の整数, |a_i| ≦B}
>則ちN(B)とは, 絶対値がB以下の2つの整数の組(a,b)で, a,bが互いに素なもの達全体の個数である.
>此の時極限値
>lim_{B→∞} N(B)/B^2
>の値を求めよ.

この問題でN(B)のオーダーがB^nならn>2のとき極限発散するやん。

>N(B)=∫[1-0,B+0] x df(x)
>
>がよく分からないが, 間違いである.

スチェルチェス積分。
0775132人目の素数さん
垢版 |
2018/12/21(金) 00:53:33.65ID:fiFOgETC
>>761 >>763 >>765

a = 1-b+t,
b = (2/√3)sinφ,
c = cosφ - (a-1+b/2),
d = 2(a-1+b/2-x1)

右向きのパーツの面積は
σ = φ - sinφcosφ + (2c + b/2)sinφ
  = φ - sinφcosφ + (2sinφ){cosφ -t +((√3)/2)sinφ},
これより
t = cosφ + ((√3)/2)sinφ - (π/5 -φ +sinφcosφ)/(2sinφ),
x1 = {3t - √(4-3tt)}/4,
y1 = (√3)(t-x1) = (√3){t + √(4-3tt)}/4,

左斜めのパーツの面積は
σ' = π/4 - arccos(y1) /2 + t・y1 /2 - (sinφ)^2 /√3 = π/5,
これより
t = {arccos(y1) +(2/√3)(sinφ)^2 -π/10} /y1,

これらを解くと
a = 0.827153176586368
b = 0.344680163597532
c = 0.956284913285257
d = 0.730102006651509
x1 = -0.36555774494062
y1 = 0.930788662970241
φ = 0.298501777856039 = 17.1028920483027°
cosφ = 0.955778171670391
sinφ = 0.294088569241317

a-1 +b/2 = -0.0005067416148658
a-1 +b = t = 0.1718333401839

境界線の長さは
L = a + 2(b+c+d) = 4.88928734365496

あまり変わりばえせぬ…
0776132人目の素数さん
垢版 |
2018/12/21(金) 01:21:48.65ID:wQpB1nJJ
>>775
>σ = φ - sinφcosφ + (2c + b/2)sinφ
>  = φ - sinφcosφ + (2sinφ){cosφ -t +((√3)/2)sinφ},
最後のsinφの係数が((√3)/2)になるのは何故?
0777132人目の素数さん
垢版 |
2018/12/21(金) 06:18:43.69ID:fiFOgETC
>>776
a-1+b = t,
b = (2/√3)sinφ,
c = cosφ - (a-1+b/2) = cosφ -t +b/2,
から
c + b/4 = cosφ -t +(3/4)b = cosφ -t + ((√3)/2)b,
0778132人目の素数さん
垢版 |
2018/12/21(金) 06:23:37.86ID:5Ao2DZHd
正20面体の各辺を、赤色・青色・黄色のいずれかの色で色付けする。
正20面体の全ての面について、面を構成する3辺がちょうど2色を用いて色付けされているような色付けの方法は何通りあるか。
但し、各辺は区別されているものとする。
(つまり、回転によって一致する塗り方を同一視する必要はない)
(3色全てを使っている必要はない)
0779132人目の素数さん
垢版 |
2018/12/21(金) 06:28:40.53ID:wQpB1nJJ
>>777
それだったら係数は1なのでは?
0783132人目の素数さん
垢版 |
2018/12/21(金) 16:08:23.04ID:fiFOgETC
ありゃ?どこ間違ったかな?

>>779
c + b/4 = cosφ - t + (3/4)b = cosφ - t + ((√3)/2)sinφ,
だった....orz
0784132人目の素数さん
垢版 |
2018/12/21(金) 16:20:36.01ID:pFCrsbdv
>>775
b = 0.344680163597532
φ = 0.298501777856039
って、b = (2/√3)sinφの関係になってる?
0786132人目の素数さん
垢版 |
2018/12/21(金) 17:06:58.75ID:t76SIJlt
>計算機で検算
総当たりで3^30通りを計算すると考えると非現実的だけど
2色でない面を見つけたら探索を打ち切れるから、
工夫すればもしかしたら現実的な計算量でいけるかもしれないね
0787132人目の素数さん
垢版 |
2018/12/21(金) 17:17:49.61ID:5Ao2DZHd
>>785
違います

>>786
ちなみに元ネタは某数学コンテスト(数オリ関連ではない)なので、計算機無しで解くことを想定されてます
0791132人目の素数さん
垢版 |
2018/12/21(金) 19:02:45.36ID:Czj7k4bs
>>774
lim_{B→∞} N(B)/B^2
は一般に存在せず,
lim_{B→∞} N(B)/B^n
が存在する.

>スチェルチェス積分だ

其の式の意味ではなくて, 何故N(B)がそう表せるのかの導出が不明だ, という意味である.
0792132人目の素数さん
垢版 |
2018/12/21(金) 19:05:23.33ID:Czj7k4bs
n=2の時ならN(B)のオーダーはB^2だ.

多分分かっていない気がするので, n=2の時の証明についても詳細を記述可能なら又返信して下さい.
0794132人目の素数さん
垢版 |
2018/12/21(金) 20:49:03.88ID:mDFT4dj8
>>778
答えは 3^10*2^20 だとおもう
20面体を反角柱と五角錐に分解することを考えて数えた
2つの五角錐を固定するとうまいことカウントできる
0795132人目の素数さん
垢版 |
2018/12/21(金) 21:08:04.22ID:5Ao2DZHd
>>794
正解!
そのように分けて考えるとうまくできますね
別のやり方の想定解を簡単に書いておきます

三色をF=Z/3Zの元に対応させておく
(赤=0,青=1,黄=2など)
辺,面をそれぞれ1~30及び1~20の整数でラベル付けする
辺の塗り方を決めることはF^30の元を1つとることと同じである
線形写像T:F^30→F^20を
T((x_i)_i)=(面iを構成する3辺に対応する値の和)_i
と定める
このとき,求めるものは
#T^(-1)(A)
(A={ (y_i)_i∈F^20 | ∀i y_i≠0 })
とかける

いま,Tは全射なので
(F^20の基底が像に含まれることを確認すればよい)

#T^(-1)(A)
=#ker(T)*#A
=3^10*2^20
0796132人目の素数さん
垢版 |
2018/12/21(金) 21:10:13.51ID:mDFT4dj8
>>795
AはF^20の部分空間をなしていないから
対応定理が使えない
それゆえ
#T^(-1)(A) =#ker(T)*#A
とカウントできることは保証されないハズ
0797132人目の素数さん
垢版 |
2018/12/21(金) 21:12:40.45ID:5Ao2DZHd
>>796
Tは全射な線形写像なので、一点の逆像は全てker(T)を平行移動したものになるから問題ないと思います
0799132人目の素数さん
垢版 |
2018/12/21(金) 21:19:06.27ID:mDFT4dj8
>>797
代数系にそなわっている対応定理は使えない
全射準同型(ここではベクトル空間だから全射線形写像と呼ぶが)も対応定理の条件の1つ
しかしながら今回のケースではAがF^20の部分空間をなしていない
何を使ったら,そうやってカウントできるのか説明してくれませんかね
0801132人目の素数さん
垢版 |
2018/12/21(金) 21:27:56.10ID:5Ao2DZHd
>>799
あなたのいう対応定理とは準同型定理のことですか?
厳密に書けば
#ker(T)=3^10
を求めるために使ってますね

#T^(-1)(A) =#ker(T)*#A
自体は単純に個数をカウントしてるだけであり、準同型定理は使用していません
Aの要素の数は2^20
また、F^20の任意の点に対し、その点の Tによる逆像に含まれる元の数はker(T)の位数と一致しています
だから掛け算で求められます
0802132人目の素数さん
垢版 |
2018/12/21(金) 21:28:00.09ID:bH4auQhW
でも、”3辺が2色” というのが ”足して0でない” は言われたらその通りなんだけど見逃したなぁ。
こういうパズル的なやつ楽しいよね。
0803132人目の素数さん
垢版 |
2018/12/21(金) 21:29:02.27ID:mDFT4dj8
>>797
それ以前に T^(-1)(A) 自体がF^30の部分空間をなしていない
商の濃度が便利に計算できるのは割る方にも代数構造が入っているからだし
(つまり T^(-1)(A) が単なる集合であってはならない)
だから やっぱりそのカウントは無理筋だとおもうんだよね
0804132人目の素数さん
垢版 |
2018/12/21(金) 21:32:33.78ID:5Ao2DZHd
たぶん勘違いされてますよ
もっと単純な話です

#T^(-1)(A)
=Σ_{a∈A} #T^-1(a)
=#ker(T)*#A

というだけです
0805132人目の素数さん
垢版 |
2018/12/21(金) 21:37:50.63ID:5Ao2DZHd
>>802
ぶっちゃけこの解き方ありきでつくられた問題な気がしますが、なかなか思いつかないと思います
パーツに分けて考えるやり方であれば高校生でも頑張れば解けるかもしれませんね
0806132人目の素数さん
垢版 |
2018/12/21(金) 21:44:09.69ID:bH4auQhW
>>805
でもF3使うまでは思いついたんだけどねぇ。
そこから差分とったら周期3の0,1,2からなる数列が出てきて、そこからF^30→F^60の方に気持ち持ってかれたorz。
でも像の元数が綺麗に中々出なかった。
Ker は1次元なんだけどねぇ。
た〜す〜の〜か〜www
0808132人目の素数さん
垢版 |
2018/12/21(金) 21:52:51.29ID:5Ao2DZHd
>>806
なるほど、かなり惜しいですね

>>807
真面目に数えてもいちおう解ける問題ですよ
背景に代数が絡んでいるだけです
0809132人目の素数さん
垢版 |
2018/12/21(金) 21:59:05.23ID:mDFT4dj8
>>804
なるほどねー
|f^(-1)(a)| = ker(f) が一般論としていえるという話で
たしかにfが全射だと成立しますね 知識として抜けてたからわざわざ確認作業しました
ためになりました
0810132人目の素数さん
垢版 |
2018/12/21(金) 23:01:37.47ID:fiFOgETC
>>775
またまた訂正....orz

a = 0.832249110949768
b = 0.339584229234133
c = 0.953736946103557
d = 0.735197941014908
a-1 +b/2 = 0.002041225566834
他は変わらず

>>784 ご指摘トンクス
0811132人目の素数さん
垢版 |
2018/12/21(金) 23:26:38.46ID:XyADTeab
>>お734
fの長さってL(f)=∫√(1+f'(x)^2)dxで与えられるよね

L(f_n)が一様有界だとしてもW^(1,1)の一様有界性は言えてもW^(1,2)の一様有界性は言えないんじゃない?
Y=W^(1,1)としてもp=1でヒルベルト空間にならないから弱コンパクト性使えないんじゃないの?
0812132人目の素数さん
垢版 |
2018/12/21(金) 23:28:21.96ID:XyADTeab
>>お734→>>734
0813132人目の素数さん
垢版 |
2018/12/21(金) 23:53:40.40ID:B8/MYwFX
>>811
>>734で書いたXに入ってる関数(の組)についての最小値を求めるのが元の問題。
YはXを含んでてHilbert空間になってるものが取れれば良い。
Xに入ってる関数を野放図にゆるすとYとしてHilbert空間が採れなくなってしまうけど今の場合
・求めたいのはその”関数”の表す曲線の”長さ”なので有界な関数に限定してよい。
・パラメータは[0,1]で動かすとして定速度のゲージでとれば速度ベクトルの大きさは1/L(Lは長さ)なので速度ベクトルも有界としてよい。
よってXに入ってる関数としては |||〜||_2 も ||〜’||_2 も可積分として良いのでYとしてHilbert空間が採れます。
0814132人目の素数さん
垢版 |
2018/12/21(金) 23:58:03.93ID:jIBEfQIT
>>813
いやいや
結局最小元の存在って変分の直接法使ってるんでしょ?
関数f_nのW^(1,2)ノルムがnを止めるごとに有限だとしても

sup_{n∈N} ||f_n||_W^(1,2)が有限とは限らない
0815132人目の素数さん
垢版 |
2018/12/21(金) 23:59:10.40ID:jIBEfQIT
それとも別の証明法で
ヒルベルト空間からRへの写像の最小元の存在が言えるんか?
0816132人目の素数さん
垢版 |
2018/12/22(土) 00:00:55.31ID:qY2OweNU
>>814
ごめん
関数f_n→関数列f_n
です
0817132人目の素数さん
垢版 |
2018/12/22(土) 00:03:14.90ID:NG24qIEO
>>814
もちろん最小限はYのなかでとってそこからオイラーラグランジュ方程式解くところまでYの中でやらないとだめですよ。
でYのなかで解いてみてじつは元のXに入ってることを確認します。
0818132人目の素数さん
垢版 |
2018/12/22(土) 00:03:22.49ID:qY2OweNU
>>813
あーごめん
そっかだから弧長パラメータ採用すれば一様有界言えるってことなんかな
0820132人目の素数さん
垢版 |
2018/12/22(土) 00:10:54.26ID:mbmYbufh
>>818
いやだめだ
弧長パラメータ採用したら結局積分区間がfに依存するわ
だからやっぱりp=2での一様有界性言えない気がする
0821132人目の素数さん
垢版 |
2018/12/22(土) 00:17:21.25ID:NG24qIEO
いま証明したいのは作用積分SはXのなかで最小値を持つこと。
最小値がないとして
S(f_1)>S(f_2)>…
がinfに収束するとする。
f_iのなかには先に述べた理由で余り素性のよくないものはないとしてよくYに入ってるとして良い。
YはHilbert空間だから lim f_i は Yの位相で収束するとしてよくその極限をfとする。
もちろん f はまだYの元(としかわからない)。
でもYは微積ができて部分積分もできるのでオイラーラグランジュの理論が使えて通常と同じ手順で円または線分となり、元のXに入ってるたとわかる。
なのでSobolev空間のなかでちゃんと解析学が展開できることを確認することは必要ですが、今回の場合はオイラーラグランジュ方程式だから部分積分できればいいので問題ない(ハズ)です。
参考文献など紹介できるほど詳しくないのであしからず。
0822132人目の素数さん
垢版 |
2018/12/22(土) 00:22:58.05ID:rCQxrUyA
>>821
>YはHilbert空間だから lim f_i は Yの位相で収束するとしてよくその極限をfとする。

ここ
これを言うには普通最小化列のYノルムでの一様有界性→弱コンパクト性使うと思うんだけど
0823132人目の素数さん
垢版 |
2018/12/22(土) 00:29:00.68ID:NG24qIEO
>>822
もちろん f_i は長さ(の合計)がどんどん小さくなって行ってるので>>813に書いてる理由でノルムは一様に抑えられてますよ。
0824132人目の素数さん
垢版 |
2018/12/22(土) 00:29:33.94ID:/qYpf/WQ
言いたいことをまとめます

lim(n→∞)F(x_n)=inf_{y∈Y}F(y)なる点列{x_n}_{n∈N}⊂Yを取ってくれば
収束列は有界列だからsup_{n∈N}|F(x_n)|<∞が言える

で、もしもF(x)≧||x||_Yみたいな評価があれば...(1)

sup_{n∈N}||x_n||_Y<∞も言える

つまりx_nはあるヒルベルト空間の閉球に完全に含まれてることになって、
ヒルベルト空間における閉球の弱コンパクト性から

部分列取ってx_(n_j)→x in Y なるx∈Yが存在する
であとはFの弱連続性を認めれば(これも証明するの難しいと思うけど)
lim(n→∞)F(x_n)=F(x)=inf_{y∈Y}F(y)
でx∈Yは最小元ってことで証明できるけど

今問題なのは(1)の評価がW^(1,2)にしてるせいで言えないこと
0826132人目の素数さん
垢版 |
2018/12/22(土) 00:32:24.66ID:/qYpf/WQ
>>811の話に戻るんだけど

L(f_i)が一様有界でも
せいぜい言えて||f_i||_W^(1,1)の一様有界性くらいじゃないかな
0827132人目の素数さん
垢版 |
2018/12/22(土) 00:34:15.67ID:/qYpf/WQ
つまり二乗とルートで相殺されて二乗の積分で下から抑えられないってことです
0828132人目の素数さん
垢版 |
2018/12/22(土) 00:42:10.24ID:K+wsO328
>>824
ごめん途中の
>部分列取ってx_(n_j)→x in Y なるx∈Yが存在する
の→は弱収束の意味です
0829132人目の素数さん
垢版 |
2018/12/22(土) 00:56:42.61ID:lvzJupuF
ヒカキンの年収が10億超え!?明石家さんま・坂上忍も驚愕の総資産とは??
https://logtube.jp/variety/28439
【衝撃】ヒカキンの年収・月収を暴露!広告収入が15億円超え!?
https://nicotubers.com/yutuber/hikakin-nensyu-gessyu/
HIKAKIN(ヒカキン)の年収が14億円!?トップYouTuberになるまでの道のりは?
https://youtuberhyouron.com/hikakinnensyu/
ヒカキンの月収は1億円!読唇術でダウンタウンなうの坂上忍を検証!
https://mitarashi-highland.com/blog/fun/hikakin
なぜか観てしまう!!サバイバル系youtuberまとめ
http://tokyohitori.hatenablog.com/entry/2016/10/01/102830
あのPewDiePieがついに、初心YouTuber向けに「視聴回数」「チャンネル登録者数」を増やすコツを公開!
http://naototube.com/2017/08/14/for-new-youtubers/
27歳で年収8億円 女性ユーチューバー「リリー・シン」の生き方
https://headlines.yahoo.co.jp/article?a=20170802-00017174-forbes-bus_all
1年で何十億円も稼ぐ高収入ユーチューバー世界ランキングトップ10
https://gigazine.net/news/20151016-highest-paid-youtuber-2015/
おもちゃのレビューで年間12億円! 今、話題のYouTuberは6歳の男の子
https://www.businessinsider.jp/post-108355
彼女はいかにして750万人のファンがいるYouTubeスターとなったのか?
https://www.businessinsider.jp/post-242
1億円稼ぐ9歳のYouTuberがすごすぎる……アメリカで話題のEvanTubeHD
https://weekly.ascii.jp/elem/000/000/305/305548/
世界で最も稼ぐユーチューバー、2連覇の首位は年収17億円
https://forbesjapan.com/articles/detail/14474
0830132人目の素数さん
垢版 |
2018/12/22(土) 00:58:26.63ID:NG24qIEO
>>824
>で、もしもF(x)≧||x||_Yみたいな評価があれば...(1)

あります。
いま考えてるFは長さなので ∃C1 ∀n t |x_n(t)| としてよい。
tは[0,1]しか取らないのでこれで ∃C2 ∀n ||x_n||_2 < C2 。
∫|x_n’(t)|dt は n によらず有界、かつ|x_n’(t)|は等速度ゲージ(で採っているとしてよい)なので∃C3 ∀n |x’_n| < C3 (= 1/L)。
つまり|x’_n(t)|は一様有界なので ||x’_n(t)||_2 も一様有界。

つまりでたらめにx_n(t)をとってしまうとx’_n(t)は病的なものが出てくるかもしれないけど、その場合にはLnを長さとして
σ(t) = (1/Ln)∫[0,t] |x’_n(t)| dt、σの逆関数をτとしてy_n(s) = x_n(τ(s))とすればこれはただのゲージ変換で長さも変化せず、しかも|y’_n(s)|はnによらない定数で抑えられてしまう。
よってこれ本体も2乗も一様に可積分。
でこのy_n(s)で鼻から議論すればよい。
0831132人目の素数さん
垢版 |
2018/12/22(土) 01:03:36.94ID:AgnllN11
>>830
等速度ゲージにしたら結局積分区間がxに依存するんじゃないの?

だから|x_n'(t)|が一様有界でもL(x_n)は一様有界じゃない
0832132人目の素数さん
垢版 |
2018/12/22(土) 01:05:09.47ID:uC7KiAdH
そもそも等速度で積分区間がxに依存しないなら
長さがどんな曲線でも定数になってしまって意味のない変分問題になってしまう
0833132人目の素数さん
垢版 |
2018/12/22(土) 01:07:32.54ID:aletPSph
>>830
ああL_nで割ってるのか

だとしても割った分一様有界性言えないよね
0834132人目の素数さん
垢版 |
2018/12/22(土) 01:08:53.34ID:aletPSph
>>833
ごめん これはただの勘違いです

まあどのみち>>831のせいで厳しいと思います
0835132人目の素数さん
垢版 |
2018/12/22(土) 01:15:21.98ID:NG24qIEO
x_nの積分区間は[0,1]ということにしてます。
y_n作る時はいわゆる距離ゲージでなく等速度ゲージにしてるのは区間をnごとに取り替えなくて済むようにしてます。
(まあ、距離ゲージにしてもできますけど。その場合は0≦t≦L_nから先は停止させることになります。)
区間は[0,1]なので等速度ゲージでは|x’_n(t)|は(tについての)定数L_nであり、nに依らず有界としてよい。
つまり|x_n(t)|も|x’_n(t)|もn,tに依らず一様に有界としてよい。
0836132人目の素数さん
垢版 |
2018/12/22(土) 01:17:22.95ID:ZXNTE5ED
高校数学
a,b,c∈ℝについて、関数f(x)を
f(x)=
{ (x²+ax+b)/((x-1)(x²+1)) (x≠1)
{ c (x=1)
で与える。
定積分 I=∫[b,p] f(x) dxを考える。α,β∈ℝは,
tan(α)=p, tan(β)=b を満たしている。このとき、
極限 lim[b→∞] I を求めよ。
lim[x→0] sin(x)/x =1 は証明せずに利用できるものとする。
0837132人目の素数さん
垢版 |
2018/12/22(土) 01:18:39.98ID:NG24qIEO
>>834
いまノルムは||x||_Y = ∫[0,1](|x|^2+|x’|^2)dtでとってるので|x_n(t)|と|x’_n(t)|がt,nに依らずに有界なら|| x_n ||_Y はnに依らず一様に有界です。
0838132人目の素数さん
垢版 |
2018/12/22(土) 01:52:06.18ID:/qYpf/WQ
>>835
>>837
あーなるほど...ごめんなさい
パラメータを0から1で固定しても1次元曲線なら等速度パラメータに取り直せるのか
失礼しました
多次元とかの曲面とかなら
曲面積∫_D √(1+|∇u(x)|)dxとして
積分領域固定で|∇u_n(x)|=S_n(xに依存しない)
とは出来ないのでそれで勘違いをしていました
0840132人目の素数さん
垢版 |
2018/12/22(土) 02:01:37.64ID:UpCRuaJY
>>839
いやー勉強になりました

あとはx_n→x (Y-弱収束)として

長さ汎関数の弱連続性と
等積条件が保存されるか(面積汎関数の弱連続性)

さえ言えればいい感じですかね
0842イナ ◆/7jUdUKiSM
垢版 |
2018/12/22(土) 11:07:04.64ID:h7yqlWMB
>>769
やっぱり円弧と単位円の交点の座標を求めるべきか。

(cosθ,sinθ)ともおけるが、方程式を解くとx^2の項とy^2の項が消え直線の式になる。この直線は弦か?

(cosθ,sinθ)と原点(0,0)の距離は1、
(cosθ,sinθ)と(b,0)の距離は、
√{(b-cosθ)^2+sin^2θ}
両者直交しているから、
三角形の面積は、
(1/2)√{(b-cosθ)^2+sin^2θ}
=(1/2)√(b^-2bcosθ+1)
0843イナ ◆/7jUdUKiSM
垢版 |
2018/12/22(土) 11:44:56.00ID:h7yqlWMB
>>842
三角形の面積が出てる。Bだ。
(1/2)b・sin(60°-θ)=(1/2)√(b^-2bcosθ+1)
b・sin(60°-θ)=√(b^2-2bcosθ+1)
b^2・sin^2(60°-θ)=b^2-2bcosθ+1
b^2{1-sin^2(60°-θ)}=2bcosθ-1
b^2cos^2(60°-θ)=2bcosθ-1
中心角60°-θ、半径1の扇形内の重複している三角形の面積=(1/2)√{b^2sin^2(60°-θ)-b^2cos^2(60°-θ)}
=(b/2)√{1-2cos^2(60°-θ)
0844イナ ◆/7jUdUKiSM
垢版 |
2018/12/22(土) 17:15:59.14ID:h7yqlWMB
>>843
第1象限で扇形が重なっている領域=π/8
2つの扇形は、
単位円,中心角r
π(60°-θ)/360°――@
半径r,中心角θ
πr^2(θ/360°)――A
円弧の中心は、
(a+r√3/2,-r/2)
2つ扇形から引く2つの三角形は、
bsin(60°-θ)/2,(b-a)r/4
第1象限の直角三角形の相似比より、
cos(60°-θ)=1/b
よって2つ扇形から引く2つの三角形はそれぞれ、
sin(60°-θ)/2cos(60°-θ)――B
{(1/cos(60°-θ)-a}r/4――C
@+A-B-C=π(60°-θ)/360°+πr^2(θ/360°)-sin(60°-θ)/2cos(60°-θ)-{(1/cos(60°-θ)-a}r/4=π/8
未知数がa,r,θの3つ。
rの二次式と見て、r>0の解を持つ。
0845イナ ◆/7jUdUKiSM
垢版 |
2018/12/22(土) 19:16:21.07ID:h7yqlWMB
>>844どうやってaやrやθの値を出したんだ? 計算過程を書いてほしい。

r^2-{(1/cos(60°-θ)-a}・360°r/4πθ+(60°-θ)/θ-sin(60°-θ)・360°/2πθcos(60°-θ)-360°/8θ=0

r^2-{(1/cos(60°-θ)-a}・90°r/πθ+(60°-θ)/θ-sin(60°-θ)・180°/πθcos(60°-θ)-45°/θ=0

D={(1/cos(60°-θ)-a}^2・(90°/πθ)^2 -4{(60°-θ)/θ-sin(60°-θ)・180°/πθcos(60°-θ)-45°/θ}≧0
0846イナ ◆/7jUdUKiSM
垢版 |
2018/12/22(土) 22:55:23.61ID:h7yqlWMB
>>845
r^2-{(1/cos(60°-θ)-a}・90°r/πθ+(60°-θ)/θ-sin(60°-θ)・180°/πθcos(60°-θ)-45°/θ=0

r=(1/2)(1/cos(60°-θ)-a}・(90°/πθ)±√{(1/cos(60°-θ)-a}^2・(90°/πθ)^2 -4{(60°-θ)/θ-sin(60°-θ)・180°/πθcos(60°-θ)-45°/θ}

境界線の最小値=2a+4・2πrθ/360°
=2a+πrθ/45°

πrθ/45°=(πθ/90°)(1/cos(60°-θ)-a}・(90°/πθ)±√{1/cos(60°-θ)-a}^2・(90°/πθ)^2 -4{(60°-θ)/θ-sin(60°-θ)・180°/πθcos(60°-θ)-45°/θ}
=(1/cos(60°-θ)-a}±(πθ/45°)√{(1/cos(60°-θ)-a}^2・(90°/πθ)^2 -4{(60°-θ)/θ-sin(60°-θ)・180°/πθcos(60°-θ)-45°/θ}
境界線の最小値=2a+(1/cos(60°-θ)-a±(πθ/45°)√[{1/cos(60°-θ)-a}^2・(90°/πθ)^2 -4{(60°-θ)/θ-sin(60°-θ)・180°/πθcos(60°-θ)-45°/θ}]
=a+(1/cos(60°-θ)±(πθ/45°)√[{1/cos(60°-θ)-a}^2・(90°/πθ)^2 -4{(60°-θ)/θ-sin(60°-θ)・180°/πθcos(60°-θ)-45°/θ}]
=a+(1/cos(60°-θ)±√4π^2・θ^2・[{1/cos(60°-θ)-a}^2・(90°/πθ)^2 -4{(60°-θ)/θ-sin(60°-θ)・90°/2πθcos(60°-θ)-90°/2θ}]
=a+(1/cos(60°-θ)±√[{1/cos(60°-θ)-a}^2・360°-4^2・π^2・θ^2・{(60°-θ)/θ-sin(60°-θ)・90°/2πθcos(60°-θ)-90°/2θ}]
a→0.122
θ→13°のとき、
最小値→3.9457になりそうにない。
0847イナ ◆/7jUdUKiSM
垢版 |
2018/12/23(日) 17:10:40.15ID:xaVNNkbo
>>846
このスレ、選りすぐりの数学の猛者があつまってると思ったが、だれも計算過程を書いてくれないのか。
0849イナ ◆/7jUdUKiSM
垢版 |
2018/12/23(日) 19:06:28.02ID:xaVNNkbo
>>847
>>848いきなりすぎて、
a=0.122……
b=……(a<b<r 接線のx切片は必要ないのかも)
r=4.……
θ=13.……
等積条件から妥当な数字が出ていることはわかるが、計算過程を書いてほしい。
Rとかθも度数法にしてほしい。計算過程なくただ数字が書いてあるだけでは長さなのか角度なのか単位がわからない。
0850132人目の素数さん
垢版 |
2018/12/23(日) 19:31:03.12ID:bPdU4Xdf
いきなりすぎるのは、すでにある解法を模写しているだけで
自分で考えていないからだよ
0851イナ ◆/7jUdUKiSM
垢版 |
2018/12/23(日) 19:56:29.02ID:xaVNNkbo
>>849
「等積条件から」
と書くならその式を、
○x^2+△x+□=0
とちゃんと書くべき。
いきなりa=0.122……
b=
r=4.……
θ=13.……
未知数4つならそれを決定する式なり条件なりを4つ書くのがふつうです。
式が4つなくても、a<b<rの条件と図からある程度は値が特定できますが、式を書かずに「等積条件から」と書くから、計算過程を書いてほしいと言っているんです。
等積条件から出そうとしているのは同じです。式が足りないから、等積条件の式だけしかないから未知数が特定できないでいます。
0853イナ ◆/7jUdUKiSM
垢版 |
2018/12/23(日) 21:03:40.05ID:xaVNNkbo
>>851扇形2つから三角形2つを引けばπ/8になる、ってことだと思うけど。
0854132人目の素数さん
垢版 |
2018/12/24(月) 00:58:52.90ID:8af0FAjd
高校数学までを範囲と想定した問題
nを自然数とする。数列{a_n}及びa_0を a_0 = 1 , a_n = a_n-1 +3-(-1)^n と、また数列{p_n}を、素数を小さい方から順に並べた数列と定める。
(1) a_n の一般項を1つの式で表せ。
(2) b_n = a_n / p_n と定める。lim[n→∞] b_1 * b_2 * b_3 * ... * b_n-1 * b_n の収束・発散を調べ、収束する場合はその値を求めよ。
0855132人目の素数さん
垢版 |
2018/12/24(月) 01:54:21.10ID:RHWhqRjf
>>642
では未知数はbです。
交角の条件(プラトーの法則) を使って絞り込めば R、θ をbで表わすことが可能です。
その場合には σ(b,θ,R) もbの関数になります。それを π/5 とおけば bが決まります。

しかし、プラトーを使わないと自由度が減らないので計算が煩雑になります。 >>733
0856132人目の素数さん
垢版 |
2018/12/24(月) 03:41:15.05ID:SRj8kjI3
nを自然数とする。
n枚のカードがあり、各カードには数字が一つずつ、1からnまでの数字が書かれている。
このカードをシャッフルして山札をつくり、次のようにしてカードを場に並べることを考える。
「山札の一番上からカードを一枚取り、場に並べることを繰り返す。ただし、二枚目以降は、それまでに場に出ているどのカードよりも大きい数字である場合のみ場に並べる。
そうでないとき、そのカードは場には並べず、そこで作業を終了する。」
このようにして場に並べたカードの数字の合計値をS(n)とおく。
S(n)の期待値を求めよ。
(Σや"……"を使わずに、nの式として表現すること)
0857132人目の素数さん
垢版 |
2018/12/24(月) 10:21:01.51ID:RHWhqRjf
プラトーの法則を使わない場合
たとえばラグランジュの未定乗数法を使います。
目的関数
 I(b,θ,R ;λ) = 2b + 4Rθ + λ{σ(b,θ,R) - π/4},
 σ(b,θ,R) = (θ+π/6) + b・sin(π/3-θ) - RR(θ-sinθ),
について
 ∂I/∂b = 2 + λ(∂σ/∂b) = 2 + λsin(π/3-θ) = 0,
 ∂I/∂θ = 4R + λ(∂σ/∂θ) = 4R + λ{1 -b・cos(π/3-θ) -RR(1-cosθ)} = 0,
 ∂I/∂R = 4θ + λ(∂σ/∂R) = 4θ + λ・2R(θ-sinθ) = 0,
 ∂I/∂λ = σ(b,θ,R) - π/4 = 0,
の4つの式を連立させて解きます。
ガンガレ
0858132人目の素数さん
垢版 |
2018/12/24(月) 13:41:46.21ID:RHWhqRjf
>>854
(1)
a_n = a_{n-1} +3 - (-1)^n
 = a_0 + Σ[k=1,n] {3 - (-1)^k}
 = a_0 +3n + {1 - (-1)^n}/2,
∴ a_n < 3n+2,

(2)
0<ε<1 とする。
素数定理から、ある自然数mがあって
k>m ⇒ p_k / log(p_k) > (1-ε)(3k+2)/3,
b_k = a_k / p_k < (3k+2) / p_k < 3/{(1-ε)log(p_k)} < r <1,
ratio test より収束。
b_1・b_2・・・・b_n = B・Π[k=m+1,∞] b_k < B・Π[k=m+1,∞] r = 0,
ここに B = Π[k=1,m] b_k >0
0862132人目の素数さん
垢版 |
2018/12/24(月) 18:45:00.10ID:8af0FAjd
>>858
解いていただきありがとうございます。
(1)は問題ないのですが、(2)で素数定理を使うと簡単に解けてしまうので「高校数学までを範囲と想定した問題」と表記した次第です。

>>861
すみません、この手の問題はしょうもないのかもしれません。
自分では面白いと思ったのですが……

一応(2)のヒントとしては{a_n}の性質ですかね
0863132人目の素数さん
垢版 |
2018/12/24(月) 19:05:16.25ID:rXagpuif
素数定理を1から証明してしまえば全く問題ない
以上
0864132人目の素数さん
垢版 |
2018/12/24(月) 20:41:01.09ID:SRj8kjI3
中学入試や算数の難問は好き
高校数学に限定する問題はなんか不自然な制約で好きくない
0865132人目の素数さん
垢版 |
2018/12/24(月) 21:58:13.46ID:CUMHScCx
>>864
わかりみが深い
前スレの最後らへんに投げられて結構議論された中学入試のやつが最たる例
0866イナ ◆/7jUdUKiSM
垢版 |
2018/12/25(火) 03:15:37.44ID:8C4zSqur
>>853
プラトーの法則だと思うんだけど、
半径rの円弧が、
分岐点(a,0)で、
y=√3(x-a)と接するってことで使ってますよね。
0868イナ ◆/7jUdUKiSM
垢版 |
2018/12/25(火) 08:44:18.02ID:8C4zSqur
>>866
短い境界線を2a、長い境界線を4×2πrθとして、
aやrやθをどういう計算過程をもって出したらいい?

今あるのは、rの二次式。
どうやってθを13°いくらと特定するか。解の公式で解けばいいか。
0869132人目の素数さん
垢版 |
2018/12/25(火) 08:54:23.36ID:759ch7em
とりあえず>>644の設定でいいならsの満たすべき方程式は
-((12*sin(s)^2+12*cos(s)^2−12*cos(s)+3)*sin(2*s)−24*sin(s)^3+(24*s−8*%pi+2*3^(3/2))*sin(s)^2−24*s*cos(s)^2+24*s*cos(s)−6*s)/(48*cos(s)^2−48*cos(s)+12)
= π/8。
近似解は
s=.8486751477323029=48.6255041427026°
0870イナ ◆/7jUdUKiSM
垢版 |
2018/12/25(火) 11:24:23.58ID:8C4zSqur
>>868
>>869「%pi」は文字化けですね。まだこれから式の意味を考える段階ですが、わかったとして、計算過程を書いてほしいです。

a(短い境界線の半分)とr(円弧の半径)を特定することなく、θを弧度法で出さはったということでしょうか?
0871132人目の素数さん
垢版 |
2018/12/25(火) 11:35:04.99ID:zHAIkLZ9
正の実数列a_nについて、
Σ(k=1,∞)a_k=1のとき、
Π(k=1,∞)(a_k)^(a_k/k)の最小値を求めよ
ただし、0^0:=1とする
0872132人目の素数さん
垢版 |
2018/12/25(火) 13:55:20.02ID:sb/tcBWj
>>870
面積の計算が一番面倒なのでそこはグリーンの公式でmaximaに計算してもらった結果が>>869ですが、別に2つの円弧と線分で囲まれた領域の面積なので、
三角形+三日月?+三日月でも計算できます。
興味があるならやってみたらいいと思います。
単位円に張り付いてる三日月は半径1, 中心角sなので、1/2×1^2×s-1/2×1^2×sin s。
仕切りに張り付いている方が半径r=sin s/(cos s - 1/2)、中心角5π/6-(s-π/2)。
三角形は1/2 sin s。
足せば>>869になるはずです。
で=π/8をとけば良い。
0873132人目の素数さん
垢版 |
2018/12/25(火) 16:54:18.40ID:W/KfkU2b
双対ベクトル、双対テンソルについて

ベクトルとその双対ベクトルに関し、以下は正しいですか?

1.反変ベクトルとは接空間上に与えられたベクトル。
2.共変ベクトルとは余接空間上に与えられたベクトル。
3.あるベクトルが与えられたとき、そのベクトルの双対ベクトルが必ず一つ存在する。
4.反変ベクトルと共変ベクトルは双対の関係にある。

次に上記を行列ないしテンソルに拡張するとして、以下は正しいですか?

11.反変テンソルとは接空間上に与えられたテンソル。
12.共変テンソルとは余接空間上に与えられたテンソル。
13.あるテンソルが与えられたとき、そのテンソルの双対テンソルが必ず一つ存在する。
14.反変テンソルと共変テンソルは双対の関係にある。
0874132人目の素数さん
垢版 |
2018/12/25(火) 17:22:45.77ID:h9yT/cNM
1辺の長さが1の正三角形P₀ABがある。P₀から三角形の内部にむけて光線を照射し、光線が最初に辺ABにぶつかる点をP₁、∠AP₀P₁=θとする。
また、光線は△P₀ABの辺で入射角と反射角が等しくなるように反射し、P₁で反射した光線が次に△P₀ABの辺にぶつかる点をP₂、
P₂で反射した光線が次に△P₀ABにぶつかる点をP₃、
以後光線がn回目に△P₀ABの辺にぶつかる点をPₙとする。
ただし、0°<θ<60°とし、光線が△P₀ABの頂点に入射したとき光線は反射しないものとする。
また、光線は△P₀ABの内部で直進するものとする。
△P₀P₁P₂の面積の最大値を求めよ。また、そのときのtanθの値を求めよ。
0875イナ ◆/7jUdUKiSM
垢版 |
2018/12/25(火) 18:06:03.36ID:8C4zSqur
牧嶋とかいう有能なケライがいたわけですか。
>>870なぞが解けました。
0876132人目の素数さん
垢版 |
2018/12/25(火) 20:22:42.67ID:F9g040cM
>>871
a1,a2,...を未知数, λをラグランジュの未定乗数とし条件付き最小化問題として解くと
F(a1,a2,...) = Σ[k=1,∞](ak/k)log(ak) + λ(Σ[k=1,∞]ak - 1),
∂F/∂ak = (1+log(ak))/k + λ = 0
より
ak = e^(-1-λk)

Σ[k=1,∞]ak = 1
より
λ = log((e+1)/e)

このとき
Σ[k=1,∞](ak/k)log(ak)
= Σ[k=1,∞](-1/k-λ)e^(-1-λk)
= -λ-e^(-1)Σ[k=1,∞]e^(-λk)/k
= -λ+e^(-1)log(1-e^(-λ))
= 1-((e+1)/e)log(e+1)
より最小値は
e(e+1)^(-(e+1)/e)
0877132人目の素数さん
垢版 |
2018/12/25(火) 21:07:36.44ID:Waj/1OvQ
>>876
すごすぎ 大正解です

数列空間上の変分問題なるものを無理やり作問したんだけど完全に想定通りの解法です
0879132人目の素数さん
垢版 |
2018/12/26(水) 01:12:21.79ID:CD/sRfld
>>856
できた。
カード k が総和に寄与する場合の数を考える。
このカードが i+1 番目に出てかつ1〜i+1 まで単調増大となる場合の数は C[k-1,i] (n-i-1)! だから求める場合の数は Σ[i=0,k-1] C[k-1,i] (n-i-1)!。
よって求める期待値を E とすれば
E n! = Σ[k=1,n] k Σ[i=0,k-1] C[k-1,i] (n-i-1)!
.  = Σ[i=0,n-1] Σ[k=i+1,n] k! (n-i-1)! /i! /(k-i-1)!
.  = Σ[i=0,n-1] Σ[l=0,n-i-1] (l+i+1)! (n-i-1)! /i! /l!
.  = Σ[i=0,n-1] (n-i-1)! (i+1) Σ[l=0,n-i-1] C[l+i+1,i+1]
.  = Σ[i=0,n-1] (n-i-1)! (i+1) C[n+1,i+2]
.  = Σ[i=0,n-1] (n+1)n(n-1)…(i+1) /(i+2)
.  = Σ[i=0,n-1] (n+1)n(n-1)…(i+1) /(i+2)
.  = (n+1)! - 1
により
E = ((n+1)! -1) /n!。
0881132人目の素数さん
垢版 |
2018/12/26(水) 01:30:34.65ID:YAwDN11w
>>874
これn回目とかいってるけど求めるのは
>△P₀P₁P₂の面積の最大値
ってP₃以降は関係ないの?
0882132人目の素数さん
垢版 |
2018/12/26(水) 01:36:25.87ID:jqxFYK0v
>>879
正解!
自分でつくった問題なんですが、ほぼ同じやり方で計算しました
E*n!の計算の最後3行の部分がよく分かりませんが、答えは合ってるし書き損じですかね?
答えが思いのほか綺麗になるので、もしかすると別の解釈があるのかなと思い少し考えましたが分かりませんでした
0884132人目の素数さん
垢版 |
2018/12/26(水) 01:50:53.57ID:jqxFYK0v
>>879
(i+1)!で約分してたのか、そこは理解しました
そこからの総和のスムーズな求め方はまだ分かってないけど
自分は(n+1)!をΣの外に出して(i+1)/(i+2)!=1/(i+1)!-1/(i+2)!の総和として計算してました

>>883
ありがとう!
0885132人目の素数さん
垢版 |
2018/12/26(水) 02:18:21.65ID:7uE5PrOX
>>884
(n+1)n(n-1)…(i+3)(i+2)(i+1) / (i+2) = (n+1)n(n-1)…(i+2) - (n+1)n(n-1)…(i+3) (ただし i = n-1 のときは第2項は-1と解釈する。強引だけどそれで i=n-1 でも合う。)
によりこれをi : 0〜n-1で足し合わせればi=0のときの第1項とi=n-1のときの第2項だけが残って(n+1)! - 1となります。
0887132人目の素数さん
垢版 |
2018/12/26(水) 02:25:06.28ID:jqxFYK0v
>>885
なるほど理解しました
(n+1)!を掛けているだけで、本質的には同じやり方っぽいですね
0889132人目の素数さん
垢版 |
2018/12/26(水) 03:46:23.61ID:eKrvMAjz
Aさんがある平面上に時計を2N個設置する。

そのうちN個は長針と短針が付いており、残りのN個は長針も短針も付いていない。
作動している時計の長針、短針は一般的な速度を刻むが、示している時刻が同じ保証はない。

あなたは長針と短針が付いていないN個の時計に対して、長針と短針をつけて作動させることができる。

ここで「特異な三角形」とは、「異なる3つの時計の中心を頂点に持つ三角形であり、また三角形の内部もしくは辺上に別の時計の中心が存在せず、
3辺すべてが3つの時計の長針と短針の劣角側(長針と短針がちょうど反対側に来るときは長針を上に持ってきたときの左側と定める)に存在しているもの」をいう。

Aさんはある時刻に部屋に入って、時計の中心同士を結ぶ。
ただし結んだ線分同士は全て交わらないように注意する。

あなたは任意の時刻で、「特異な三角形」をN個以下にすることが可能であることを示せ。

ただしAさんが線分を結ぶ時間、及びあなたが時計を作動させる時間は無視できる。
0890132人目の素数さん
垢版 |
2018/12/26(水) 03:46:59.70ID:euuUvasb
>>879
>E = ((n+1)! -1) /n!。

これ、E = n+1 - 1/n! としてもいいですか?
0892132人目の素数さん
垢版 |
2018/12/27(木) 01:13:07.85ID:lwT4iYS4
>>889
>ただし結んだ線分同士は全て交わらないように注意する。

これは辺さえ交差してなければ、内部に他の点を含むのはありですか?
例えば正三角形の頂点ABCとその重心Gに時計が配置されてて頂点上の針の狭角に他の全ての頂点が入っていているときに△ABCは「特異な三角形」にカウントされますか?
また内点を共有するのはありですか?
例えば正三角形の頂点ABCとその重心Gに時計が配置されてて頂点上の針の狭角に他の全ての頂点が入っていて、重心の時計の針の狭角にABが入っている場合、
△ABGと△ABCを両方「特異な三角形」とカウントするのはありですか?
0894132人目の素数さん
垢版 |
2018/12/27(木) 10:27:57.67ID:fyLTl4ik
>>892

「三角形の内部もしくは辺上に別の時計の中心が存在せず、」の条件から前者は認めません。点を共有する事は認めます。
0895132人目の素数さん
垢版 |
2018/12/27(木) 11:35:42.20ID:5E6XzOOT
特異な三角形って、作動していない時計が含まれていても意味がある概念?
(針がついていない場合は、針と辺に関する条件は満たされているものと考える、とか)
そうでなければ作動していない時計の存在意義がないと思うし
あと、特異な三角形と考えるのはAさんがその3つの時計を直線で結んでいる場合のみ?(でないと直線で結ばれている意味がないはず)

「あなたは〜することが可能であることを示せ」とあるけど、"あなた"ができるのは時計に針をつけて作動させることだけだよね?
つまり、まだ針のついていない時計の時刻をうまく決定することで条件を満たすようにできることを示す、ということでよい?
0896132人目の素数さん
垢版 |
2018/12/27(木) 11:57:48.55ID:fyLTl4ik
>893
自作です

>895
N個の時計は必ず作動させなくてはいけません。
そのあと二つは仰る通りです。
文章が分かりにくくてすいません。
0897132人目の素数さん
垢版 |
2018/12/27(木) 14:35:54.79ID:VTfJXhBm
>>874
 tanθ = t とおいて、面積Sをtで表わす。

・0<θ<30°のとき
AP1 = sinθ / sin(120゚-θ) = 2t/(√3 +t),
AP2 = AP1 sin(60゚+θ)/sin(60゚-θ)
 = AP1 sin(120゚-θ)/sin(60゚-θ)
 = sinθ / sin(60゚-θ)
 = 2t/(√3 -t),

S(t) = 儕0P1A - 僊P1P2
 = (1/2)sin(∠A) AP1 (1-AP2)
 = (1/2)((√3)/2)(2t/(√3 +t))((√3 -3t)/(√3 -t))
 = (3/2)t(1-t√3)/(3-tt),

t = 2√6 - √3 のとき極大 (3√3 -2√6)/4 = 0.074293
θ = 16.5505°


・30゚<θ<60°のとき
AP1 = sinθ / sin(120゚-θ) = 2t/(√3 +t),
BP1 = 1 - AP1 = (√3 -t)/(√3 +t),
BP2 = BP1 / AP1 = (√3 -t)/2t,
S(t) = 儕0P1B - 傳P1P2
 = (1/2)sin(∠B) BP1 (1-BP2)
 = (1/2)((√3)/2)(√3 -t)(3t-√3)/(t(√3 +t))
 = (3/8)(√3 -t)(t√3 -1)/(t(√3 +t)),

t = (2√6 +√3)/7 のとき極大 (3√3 -2√6)/4 = 0.074293
θ = 43.4495°
0898イナ ◆/7jUdUKiSM
垢版 |
2018/12/27(木) 14:45:25.75ID:8aSvA2V1
>>875午後2時x分に長針と短針が一直線になるとすると、
長針は60分で360°進むからx分で6x°進む。
短針は60分で30°進むからx分で0.5°進む。
今午後2時とすると、長針は短針より60°手前。x分後長針は6x°短針は0.5x°進んでいるから、
6x-60=0.5x+180
5.5x=240
11x=480
x=480/11
=43+2/11
2時43分10(+10/11)秒
(2時43分10秒9090……)
0902132人目の素数さん
垢版 |
2018/12/28(金) 00:26:01.00ID:YFtAo9wI
S_nをn次対称群とする。
σ∈S_nに対し
s(σ)=(σの符号)∈{-1,1}
f(σ)=(σの固定する要素の数)
と定めるとき、
Σ[σ∈S_n]s(σ)/(f(σ)+1)=(-1)^(1+n)*n/(n+1)
を示せ。
0905132人目の素数さん
垢版 |
2018/12/28(金) 09:57:30.49ID:J3JKEwNA
計算機だとどう解くのかねえ

0.初期状態を決める
1.交点の角度の小さなものを少し拡げる
2.面積の小さな領域を少し広げる
3.誤差が許容範囲以内になるまで1〜2を繰り返す

こんなところかなあ
0906132人目の素数さん
垢版 |
2018/12/28(金) 13:35:26.96ID:1iwNDVav
直線でいいならプログラムもさほど困難じゃないような気がするんが。
0908イナ ◆/7jUdUKiSM
垢版 |
2018/12/29(土) 16:46:28.57ID:y2Z7Fa5C
>>898
五角形のカブトガニ、分岐は120°だけど、周との交わりがなんか偏ってるよね。
0909132人目の素数さん
垢版 |
2018/12/29(土) 17:24:01.08ID:btK/M0Gd
■速報■

無限に続くと思われていた円周率がついに終りを迎えた
千葉電波大学の研究グループがこれまでの円周率演算プログラムに
誤りがあったことを発見
同大のスーパーコンピュータ「ディープ・ホワイト」を使って
改めて計算しなおしたところ、10桁目で割り切れたという

10桁目の最後の数字は「0」だった

千葉電波大学の研究グループの発表によると、
円周率計算に際し、改めて既存の円周率計算プログラムを
点検してみたところ、円周の誤差を修正する数値に
誤りがあることに気が付いた
この数値を正常値に直して計算しなおしてみたところ、
円周率は10桁で割り切れたという
0910132人目の素数さん
垢版 |
2018/12/29(土) 19:48:21.17ID:xdg2MoVf
原点O中心、半径√13の円に内接する正三角形ABCがある。
点Pは最初得点mを持って点Aからスタートし、次のルール(あ)、(い)で点数操作が行われる:
(あ) 点を出発する際に得点は3倍される
(い) 点に到着した際に得点は到着点のy座標の値がそのまま加算される。例えばy座標が-1であれば, -1される

いま、A→B→C→Aの移動を行うとき
(i)1周した後の得点m'の取りうる範囲を、初期の得点mを用いて表し、
(ii)得点m=1でスタートして、得点に変化がない場合、点Bの座標として考えられるものをすべて挙げよ。
0911132人目の素数さん
垢版 |
2018/12/29(土) 21:15:58.90ID:TLF1TGk4
>>902
できた。
まず基本的な公式として n≧3 のとき
Σ s(σ) = 0、Σ s(σ)f(σ) = 0、Σ[f(σ) = 0] s(σ) = (-1)^(n-1)(n-1)
が成立する。容易ゆえ証明は略。
自然数 n,i に対し
X[n,i] = Σ[n∈Sn] s(σ)/(f(σ) + i)、
Y[n,i] = (-1)^(n-1) n! (i-1)!/(n+i)/(n+i-2)!
とおく。
X[n,i] = Y[n,i] を示せば十分である。
n≦3 においては容易。
Yが漸化式
Y[n+1,i] = -(n+i)Y[n,i] + (i-1)Y[n, i-1]+ Y[n,i+1]  (i≧2)、
Y[n+1,i] = -(n+i)Y[n,i] + (-1)^(n-1)(n-1)+ Y[n,i+1] (i=1)
を満たすことは容易。
σ∈S[n+1] に対し g(σ) をその固定点数とする。
まずn≧3、i≧2 において
X[n+1,i]
= Σ[σ∈S[n],k:1〜n+1] s((k n+1)σ)/(g((k n+1)σ)+i)
= Σ[σ∈S[n]]( -(n-f(σ))s(σ)/(f(σ)+i) - f(σ)s(σ)/(f(σ)+i-1) + s(σ)/(f(σ)+i+1) )
= Σ[σ∈S[n]]( -(n+i)s(σ)/(f(σ)+i) + (i-1)s(σ)/(f(σ)+i-1) + s(σ)/(f(σ)+i+1) )
= Σ[σ∈S[n]]( -(n+i)s(σ)/(f(σ)+i) + (i-1)s(σ)/(f(σ)+i-1) + s(σ)/(f(σ)+i+1) )
= -(n+i)X[n,i] + (i-1)X[n, i-1]+ X[n,i+1]
であり、n≧3、i=1において
X[n+1,1]
= Σ[σ∈S[n],k:1〜n+1] s((k n+1)σ)/(g((k n+1)σ)+1)
= Σ[σ∈S[n]]( -(n-f(σ))s(σ)/(f(σ)+1) + s(σ)/(f(σ)+2) ) - Σ[σ∈S[n], f(σ)≠0] f(σ)s(σ)/(f(σ))
= Σ[σ∈S[n]]( -(n-f(σ))s(σ)/(f(σ)+1) + s(σ)/(f(σ)+2) ) + Σ[σ∈S[n], f(σ)=0] s(σ)
= Σ[σ∈S[n]]( -(n-f(σ))s(σ)/(f(σ)+1) + s(σ)/(f(σ)+2) ) + (-1)^(n-1)(n-1)
= -(n+i)X[n,i] + (-1)^(n-1)(n-1)+ X[n,i+1]
である。
0912132人目の素数さん
垢版 |
2018/12/30(日) 08:58:19.51ID:kTwoovIR
>>911
想定していた解とは違いますが、正しそうです
問題より強い主張を漸化式を用いて示したわけですね
i≧2の場合の議論は避けられなさそうです
詳しく書いてくださりありがとうございます

ちなみに出典は某数学コンテスト(数オリではない)です
0913132人目の素数さん
垢版 |
2018/12/30(日) 09:23:56.79ID:k0jfpvut
>>902
俺もできたけど>>911の方が美しいかな…
一応概略を書いとく

>>911と同じく Σ[σ∈S_n]s(σ)/(f(σ)+i) を求める方針で、使う漸化式が異なる。
なので記号を借りて
 X[n,i]=Σ[σ∈S_n]s(σ)/(f(σ)+i) (*)
とおく。

S_n の元を、n-1, n の行き先によって分類する。
(i)n-1, n を固定するもの。
これだけで (*) の和を考えると、X[n-2, i+2] に一致することが分かる。
以下同様に、
(ii)n-1 と n を入れ替えるもの→-X[n-2, i]
(iii)n を固定し n-1 を固定しない→X[n-1,i+1]-X[n-2,i+2]
(iv)n を n-1 に移し、n-1 を n に移さない→-X[n-1,i]+X[n-2,i+1]
(v)n-1 を固定し n を固定しない→(iii)と同じ
(vi)n-1 を n に移し、n を n-1 に移さない→(iv)と同じ
(vii)n-1,n が共に n-2 以下に移る→0 (∵σとσ(n-1 n)で打ち消しあう)

これらの総和をとって
 X[n,i]=2(X[n-1,i+1]-X[n-1,i])-(X[n-2,i+2]-2X[n-2,i+1]-X[n-2,i])
を得る。あとは推測して帰納法。
0915132人目の素数さん
垢版 |
2018/12/30(日) 09:53:01.19ID:iqGvUL00
>>902
のような問題は解けと言われれば解けるけど、なんでこんな式が成立するのか、見つけられるのかがさっぱりわがんね。
なんか背景理論的なものがあるんだろうなぁとしかわがんね。
0916132人目の素数さん
垢版 |
2018/12/30(日) 10:16:38.72ID:lxuhMzG5
>>909
虚構新聞ですね。

π =
3.1415926535 8979323846 2643383279 5028841971 6939937510
5820974944 5923078164 0628620899 8628034825 3421170679
8214808651 3282306647 0938446095 5058223172 5359408128
4811174502 8410270193 8521105559 6446229489 5493038196
4428810975 6659334461 2847564823 3786783165 2712019091
4564856692 3460348610 4543266482 1339360726 0249141273
7245870066 0631558817 4881520920 9628292540 9171536436
7892590360 0113305305 4882046652 1384146951 9415116094
3305727036 5759591953 0921861173 8193261179 3105118548
0744623799 6274956735 1885752724 8912279381 8301194912
9833673362 4406566430 8602139494 6395224737 1907021798
6094370277 0539217176 2931767523 8467481846 7669405132
0000000000 0000000000 0000000000 0000000000 0000000000
0000000000 0000000000 0000000000 0000000000 0000000000
0918イナ ◆/7jUdUKiSM
垢版 |
2018/12/30(日) 15:00:24.87ID:olbjfmVh
>>908
>>910(i)
点Aを(0,√13)とした場合、
点Bを(-a,-b)、点Cを(a,-b)とおくと、
点数はm→3m→3m-b→9m-3b→9m-4b→27m-12b→27m-12b+√13
m'=27m-12b+√13
bを求める。
AC^2=(√13+b)^2+a^2
BC^2=4a^2
AC=BCより、
(√13+b)^2=3a^2――@
点C(a,-b)は半径√13の円周上にあるので、
a^2+b^2=13――A
@Aより、
13+2b√13+b^2=3(13-b^2)
4b^2+2b√13-26=0
2b^2+b√13-13=0
b=(√13)/2
∴m'=27m-5√13
0919イナ ◆/7jUdUKiSM
垢版 |
2018/12/30(日) 15:31:48.04ID:olbjfmVh
>>918
(ii)m=m'=1とすると、
1=27-12b+√13
12b=26+√13
b=(26+√13)/12
a^2={(b+√13)^2/3}^2
a=(b+√13)/3
={(26+√13)/12+√13}/3
=(26+13√13)/36
とりうる点Bは、
((26+13√13)/36,(26+√13)/12)
((26+13√13)/36,(-26-√13)/12)
((-26-13√13)/36,(26+√13)/12)
((-26-13√13)/36,(-26-√13)/12)
0923イナ ◆/7jUdUKiSM
垢版 |
2018/12/31(月) 12:28:20.21ID:HqvNJuR/
>>921範囲か!
>>918修正。
(i)m'=m-12b+√13において0≦b≦
27m-11√13≦m'≦27m+13√13
ちがうか。
点A(√13cosθ,√13sinθ)
点B(√13cos{θ+(2π/3)},√13sin{θ+(2π/3)})
点C(√13cos{θ+(4π/3)},√13sin{θ+(4π/3)})
Aを出発するとき、
m→3m
Bに到着するとき、
3m→3m+√13sin{θ+(2π/3)}
Bを出発するとき、
3m+√13sin{θ+(2π/3)}→3[3m+√13sin{θ+(2π/3)}]
=9m+3√13sin{θ+(2π/3)}
Cに到着するとき、
9m+3√13sin{θ+(2π/3)}→9m+3√13sin{θ+(2π/3)}+√13sin{θ+(4π/3)}
=9m+3√13sin{θ+(2π/3)}+3√13sin{θ+(2π/3)}-√13sin{θ+(2π/3)}
=9m+2√13sin{θ+(2π/3)}
Cを出発するとき、
9m+2√13sin{θ+(2π/3)}→3[9m+2√13sin{θ+(2π/3)}]
=27m+6√13sin{θ+(2π/3)}
Aに到着するとき、
27m+6√13sin{θ+(2π/3)}→27m+6√13sin{θ+(2π/3)}+√13sinθ
∴m'=27m+6√13sin{θ+(2π/3)}+√13sinθ
(0≦θ≦π/2)
このとき0≦sinθ≦1
2π/3≦θ+(2π/3)≦5π/6
sin(5π/6)≦sin{θ+(2π/3)}≦sin(π/2)=1
m'のとりうる範囲は、
27m+6√13sin(5π/6)+√13sin(5π/6)≦m'≦27m+6√13sin(2π/3)
27m+6√13(√13)/2+√13(√13)/2≦m'≦27m+6√13(√13)(√3/2)
27m+39+13/2≦m'≦27m+39√3
27m+91/2≦m'≦27m+39√3
0925132人目の素数さん
垢版 |
2019/01/01(火) 10:21:41.54ID:nRsyFy0N
甲乙二人がおのおの32ピストル(当時のお金の単位)の金を賭けて勝負したとする。
 そしてどちらかが先に3点を得たものを勝ちとし、勝った方がかけ金の総額64ピストルをもらえるとする。ところが甲が1点を得ただけで、勝負が中止になってしまった。
 このとき、二人のかけ金の総額64ピストルを甲と乙にどのように分配すればよいだろうか。
 ただし二人の力は互角で、勝つ確率はそれぞれ1/2ずつだとする。
0926 【大吉】
垢版 |
2019/01/01(火) 10:59:12.75ID:lflPiwVu
>>925
甲32ピストル
乙32ピストル
∵勝負は中止になった。どちらも3点には満たない。どちらか勝ったほうが64ピストル総取りのルールだったはず。分配なら最初に言っとかないといかん。
>>923
0928132人目の素数さん
垢版 |
2019/01/01(火) 16:37:14.23ID:31JmtpX8
>>925
何をもって良い分配と見なすかによるでしょ

期待される勝率によって分けるなら
甲:44 乙:20
当初のルールを忘れて甲が勝利したと見なすなら
甲:64 乙:0
勝敗がついていない為に完全な引き分けと見なすなら
甲:32 乙:32

他にもいくらでもこじつけられそう
0929132人目の素数さん
垢版 |
2019/01/02(水) 00:21:57.02ID:GvDO1SFm
>>760 (下) すべて線分、分岐角120°
   L = 4.889287343655

>>903 (上) 線分3 + 円弧4、分岐角120゚、外円との交角90°
   L = 4.848096236

かな?
0930132人目の素数さん
垢版 |
2019/01/04(金) 15:45:31.82ID:BKdPZt1m
格子
Γ = { M*(-143+√-2) + N*(401) | M,Nは整数 }
で(原点以外の点で)最も原点に近い点を知るには、やみくもに探す以外に良い計算があるか?

どんな格子でも通用する方法ではなくて、実は格子として長方形型格子 {M+N√-2|M,Nは整数} と相似な格子という仕掛けがあります
0932132人目の素数さん
垢版 |
2019/01/07(月) 06:06:02.69ID:q8wkjRc4
四角形の二つの辺の長さがそれぞれ800,1000である。
このうち、向かい合う二つの頂点から平行にそれぞれの対辺に対して二本の線を引く。このとき2つの直線の距離は300である。
このとき引いた直線は対辺をある長さx,yに分割するが、このx,yを求めよ。
0933132人目の素数さん
垢版 |
2019/01/07(月) 06:28:09.98ID:Ye80VrF7
>>931
出題者ではないが

時計を置く先手の最善手:
針のある時計を正N角形に配置
針のない時計は外側の任意の位置に置く
針は全て同時刻に合わせ、9時の向きに
正N角形の中心が来るようにする
(針が6時を指した時、正N角形の中の
三角形がすべて有効になる)

針を追加する後手の最善手:
時刻を先手の時計の6時間後に合わせる
文字盤の向きはすべて同じ方角とする
(針のなかった時計の中心を2つ以上含む
三角形が、任意の時刻ですべて無効となる)

三角形を描く先手の最善手:
自分の作った正N角形の外周と
対角線のうちN-3本を結び、N-2個の
三角形を作る
外側は適当に結ぶ
(「特異な三角形」は、6時のとき
最大N-2個)

証明は頭のいい人に任せた
0936132人目の素数さん
垢版 |
2019/01/07(月) 11:22:10.08ID:kZqBQG7w
>>932
最初の四角形の頂点をA,B,C,Dとして、問題を書き直してはどうか
0938132人目の素数さん
垢版 |
2019/01/07(月) 11:31:52.45ID:UGEAw86a
面白い問題の予感がしない
0939132人目の素数さん
垢版 |
2019/01/07(月) 16:33:29.00ID:G1hK8Aqj
>>937
長方形です。
みなさんごめんなさい。面白くないのは承知しています。
0941132人目の素数さん
垢版 |
2019/01/07(月) 18:11:37.07ID:MyoHHzE6
test
0943132人目の素数さん
垢版 |
2019/01/08(火) 12:15:47.54ID:iVFH0oHY
左括弧"("と右括弧")"を2n個並べたとき,正しく括弧が組み合わさっている確率をP_nとする.
[例:(()),()() :正しい, )(((,)(():正しくない]

このとき,lim(n→∞)n√nP_nを求めよ.
0945132人目の素数さん
垢版 |
2019/01/13(日) 17:17:11.53ID:23+SNwHD
連休なのに過疎ってるのでネタを投下してみる

問:表面積が1であるf面体のうち、体積Vが最大であるものは何か?

これに対してメディアル多面体が解となるという予想があった。定義は以下。

26問目スレ 447
>「メディアルf面体」
>  [ 6-12/f ] 角形と [ 6-12/f ] +1 角形のみからなるf面体。
> f≧12 のときは 5角形×12,6角形×(f-12)

これについて調査したところ、以下の場合は反例がありそう。

f=11 V=0.080055026399577983 4角形×2,5角形×8,6角形×1
f=13 V=0.082432267303420834 4角形×1,5角形×10,6角形×2
f=33 V=0.089603827451613424 5角形×13,6角形×19,7角形×1

そこで、以下の問を提案する。

問「表面積が1であるf面体のうち、体積Vが最大である解」がメディアルf面体でないfはいくつあるか?

つまり、条件を満たす f は、上記 11,13,33 がすべてであるか?(そもそも上記の例は最大解と言えるか?)
0946132人目の素数さん
垢版 |
2019/01/15(火) 06:38:35.78ID:ke5su7tE
n次正方行列Aが冪零行列のとき、A^p=Oをみたす正整数pの最小値を求めよ。
0949132人目の素数さん
垢版 |
2019/01/16(水) 01:11:03.88ID:lOjtUToz
>>945
連休終わったけど…

f   V/S^{3/2}
4  0.05170027 = 1/{6√(6√3)}     正4面体
6  0.06804138 = 1/(6√6)       立方体
8  0.074488  4角形×4, 5角形×4  メディアル8面体
10  0.078740  4角形×8, 4角形×2  (シリコンフラーレン)
12  0.08168837 = φ^{4/7} /{6(√3)・5^{5/8}) 正12面体 
14  0.083365  5角形×12, 6角形×2  ねじれ重角錐台(ゴールドバーグ)
16  0.084740  ?
20  0.086610  5角形×12, 6角形×8  メディアル20面体
32  0.089493  5角形×12, 6角形×20  切頂20面体(サッカーボール)
42  0.090565  5角形×12, 6角形×30  切稜12面体
∞  0.09403160 = 1/(6√π)       球

φ = (1+√5)/2 = 1.61803398875 (黄金比)

http://www.jstage.jst.go.jp/article/tmj1911/40/0/40_0_226/_pdf
0950132人目の素数さん
垢版 |
2019/01/16(水) 07:54:30.43ID:h759beZ/
>>949
こちらの計算した値は以下:
f   V/S^{3/2}
4    0.051700269950116645   正4面体
6    0.068041381743977170   立方体
8    0.074344868093229974
10   0.078734752898039745
12   0.081688371824182551   正12面体 
14   0.083349245941114841
16   0.084742718358283536
20   0.086626966830007951   切頂20面体(サッカーボール)
32   0.089493100466131958
42   0.090574499972086386 (切稜12面体=0.090566239172274965)

正多面体とサッカーボール以外では値が多少違っているような。
精度の問題でしょうかね。
0951132人目の素数さん
垢版 |
2019/01/16(水) 08:06:26.75ID:h759beZ/
>>950
>20   0.086626966830007951   切頂20面体(サッカーボール)
>32   0.089493100466131958
サッカーボールは32面体のほうです。
20   0.086626966830007951   メディアル20面体
32   0.089493100466131958   切頂20面体(サッカーボール)

なお、f=20はメディアル20面体には違いないですが、ゴールドバーグ論文のXX-1(1,3,3,(6),3,3,1)やXX-2(1,6,6,6,1)とは異なり
たぶん 2,2,(4),(2,2),(4),2,2 で表されているものに近いのではないかと思います。
0954132人目の素数さん
垢版 |
2019/01/16(水) 15:42:04.04ID:S1Sxn+2y
>>951
化学式みたいにして多面体を表す方法があるんでしょうか
0955132人目の素数さん
垢版 |
2019/01/17(木) 01:39:36.05ID:w+hJ6CW4
>>950 >>951
 修正乙

>>949
5  0.059698329545752329 = 1/{9√(2√3)}
       正3角形プリズム、(正3角形の辺)/(高さ) = tan(π/3) = √3
7  0.071398254996602697 = (1/15)√{(5/6)cot(π/5)} 
       正5角形プリズム、(正5角形の辺)/(高さ) = tan(π/5)
9  0.076900
10  0.078734752898039745  4角形×2, 5角形×8  (Siフラーレン)
12  0.081688371824182551 = φ^{7/4} /(6(√3)・5^{5/8}) 正12面体 
17  0.085206

cot(π/5) = φ^{3/2} / 5^{1/4} = 1.37638192
φ = (1+√5)/2 = 1.61803398875
0956132人目の素数さん
垢版 |
2019/01/17(木) 02:20:36.38ID:UEm1182L
ABを直径とする半径5[km]の円形の湖がある
この湖の水質は一様ではなく、
円の中心からr[km]離れた場所では時速r[km/h]までのスピードでしか泳げない
AからBまで泳ぐときの最短時間を求めよ
0958132人目の素数さん
垢版 |
2019/01/17(木) 06:19:24.09ID:w+hJ6CW4
>>956
「もののふの矢橋の船は速けれど 急がば回れ瀬田の長橋」
                        宗長(室町時代)
0959132人目の素数さん
垢版 |
2019/01/17(木) 07:10:26.91ID:w8EhBaN/
>>957
不正解
少なくとも円周上を泳げばπ時間で泳げる
0960132人目の素数さん
垢版 |
2019/01/17(木) 08:02:19.87ID:5Ku0kPJA
>>954
ゴールドバーグの記法によると、
「まず適当な回転対称軸を取って上下方向を決め、各面の中心が位置する高さが同じものの枚数を数えて列挙する」
というやり方らしい。

この方法では、最初の軸の取り方によって、違う表現になったりするので注意が必要。
例えば、正三角柱は、三角形の面を下にして置けば 1,3,1 であるし、四角形の面を下にして置けば下から 1,2,2 となる。

また、ゴールドバーグ記法だと、数字の表しているものが何角形か明記されないところが分かりにくいと思われるので、
3〜7角形の面にt,q,p,x,hのアルファベットを付けて表してみる。特に正多角形の面は大文字で表す。
先の正三角柱の例は、T1,q3,T1 または、q1,T2,q2 のような表現となる。

f   V/S^{3/2}
4    0.051700269950116645   T1,T3 (正四面体)
5    0.059698329545752329   T1,q3,T1 (正三角柱)
6    0.068041381743977170   Q1,Q4,Q1 (立方体)
7    0.071398254996602697   P1,q5,P1 (正五角柱)
8    0.074344868093229974   q2,p2,p2,q2
9    0.076898933926867766   p3,q3,p3
10   0.078734752898039745   Q1,p4,p4,Q1
11   0.080055026399577983   x1,(q2+p4),p2,p2
12   0.081688371824182551   P1,P5,P5,P1 (正十二面体)
13   0.082432267303420834   q1,(p2+x2),p4,p2,p2
14   0.083349245941114841   X1,p6,p6,X1
16   0.084742718358283536   x1,p6,(p3+x3),p3
17   0.085264872589057683   x1,(p4+p2),(p2+x4),p2,p2
20   0.086626966830007951   x2,(p2+p4+x2),(x2+p4+p2),x2
32   0.089493100466131958   P1,x5,P5,x5,x5,P5,x5,P1 (切頂20面体)
33   0.089603827451613424   p1,x5,p5,x5,(x4+h1),p5,(x4+p2),x1
42   0.090574499972086386   P1,x5,x5,p5,x10,p5,x5,x5,P1
0961132人目の素数さん
垢版 |
2019/01/17(木) 08:04:55.11ID:5Ku0kPJA
>>960
f=32の切頂20面体の六角形の面は正六角形ではないことに注意。
2種類の長さの辺を持つ内角120°の六角形で、辺の長さの比は sin24°:sin36°=(√(3+6/√5)-1)/2:1≒0.692:1

f=33は回転対称ではない。切頂20面体のひとつの六角形を五角形2つに分割したもので、新しくできた稜の両端に接する面の角も一つずつ増える。

f=42は、切稜12面体(P1,x5,x5,P5,x10,P5,x5,x5,P1)の上半分(,P5,x5,x5,P1)を10分の1周だけ回転させて組み合わせた形になる。
0962132人目の素数さん
垢版 |
2019/01/18(金) 01:44:16.89ID:SClQ+Kbh
(f=32 の参考に)
 
フラーレン分子C_60

「C_60には90本のC-C結合があるが、そのうちPを形成している60本は 1.458Å、2つのhに共有される残り30本は 1.401Åの長さを持つことが知られている。」(比は 0.961)

藤田、吉田、大澤:炭素, No.162, p.100-109 (1994)
H. Hedberg et al.: Science, 254, p.410 (1992)
http://www.jstage.jst.go.jp/article/tanso1949/1994/162/1994_162_100/_pdf


「C_60は立方晶系(等軸晶系)で、分子内での二つのhに共有されているC-C距離は 1.391Å、P内のC-C距離 1.455Åということからすると、・・・・」(比は0.956)

日本大百科全書(ニッポニカ)
http://kotobank.jp/word/フラーレン-164308
0964132人目の素数さん
垢版 |
2019/01/18(金) 06:06:10.47ID:ZOTcJQBl
ごめん>>956の問題だと結局円周が答えになってしまってつまらないので修正します

直径AB、半径5[km]の円形の湖がある
この湖の水質は一様ではなく、
円の中心からr[km]離れた場所では時速r[km/h]までのスピードでしか泳げない
ABを3:2に内分する点をPとしたとき、
AからPまで泳ぐときの最短時間を求めよ

まあでも>>957がほぼ答えなのですが
0965132人目の素数さん
垢版 |
2019/01/18(金) 16:17:40.84ID:/XwlhYRA
>>960-961
P1,x5,x5,p5,x10,p5,x5,x5,P1と
P1,x5,x5,P5,x10,P5,x5,x5,P1
本質的に変わらんw

この書き方じゃ一意に決まらないんでは?
0966132人目の素数さん
垢版 |
2019/01/18(金) 19:17:46.00ID:SClQ+Kbh
>>964

 log(r) = log(5)(1 - θ/π)
のコースだから
 (1/r)(dr/dθ) = - log(5)/π,
 √{π^2 + log(5)^2} 時間
0967132人目の素数さん
垢版 |
2019/01/18(金) 21:14:26.42ID:Yq+h6qB0
>>966
おー正解です
対数螺旋が最短であることの証明は出来ますか?
0968イナ ◆/7jUdUKiSM
垢版 |
2019/01/19(土) 16:46:31.95ID:elTtnGN6
>>940
>>956円周を泳ぐと、
泳ぐ距離は10π/2
速さは5q/h
時間は、
(10π/2)÷5=π(時間)
ABを質量m(s)の人がまっすぐ泳ぐとき、
加速度-a(q/h)として、
エネルギー保存の法則より、
(1/2)m・5^2-m・a・5=0
a=2.5(q/h^2)
池の中心に達するまでの時間をt(h)とすると、
5t-(1/2)at^2=5
5t-1.25t^2=5
4t-t^2=4
t^2-4t+4=0
t=2
AB間は2t=4(時間)かかる。
AB間を池の端に中心とした半径5√2(q)の円弧を描くように泳ぐと、
加速度-b(q/h)として、
エネルギー保存の法則より、
(1/2)m・5^2-m・b・(2π/8)5√2=0
b=5√2(q/h^2)
中心に最接近するまでの時間をT(h)とすると、
5T-(1/2)bT^2=(2π/8)5√2
T-(1/2)(√2)T^2=(π/4)√2
T^2-T√2+π/2=0
判別式D=(√2)-4(π/2)
=2-2π<0
∴円周を泳ぐときがπ時間でもっとも速いと考えられる
0969132人目の素数さん
垢版 |
2019/01/19(土) 18:00:57.45ID:qYdfMlDa
>>968
ABをまっすぐ泳ぐ場合は等加速度運動ではないのでそうはなりません
中心部では速度0なのでAからBまでまっすぐ泳げないというのが正しいです

それとそれだけでは他のあらゆるルートより短いことの証明にはなってません
0970132人目の素数さん
垢版 |
2019/01/19(土) 18:04:54.38ID:qYdfMlDa
>>969
ごめんなさい
>中心部では速度0なのでAからBまでまっすぐ泳げないというのが正しいです
これは語弊がありました
正しくは中心部までにかかる時間が1/(5-x)の0から5までの積分で発散するので中心まで有限時間では辿り着けない
でした
0971132人目の素数さん
垢版 |
2019/01/19(土) 18:14:38.16ID:EsplZJyO
>>965
やっぱり図示したほうがいいのかな

ゴールドバーグの示した切稜12面体がこちら
http://i.imgur.com/CGlcOxY.png

で、問題の42面体がこちら。真ん中の部分を互い違いにして作る
http://i.imgur.com/k0USDtP.png

確かに表現が同じになってしまう。うまい表現方法はないものか。
0974イナ ◆/7jUdUKiSM
垢版 |
2019/01/19(土) 19:36:28.31ID:elTtnGN6
>>968
>>969中心とおるコースだと、中心で永遠に泳ぎつづけることはわかりました。

結局π時間で正解ですか?
0975132人目の素数さん
垢版 |
2019/01/19(土) 19:50:11.57ID:X9Bj3NZe
>>974
>>956の問題ならπ時間で正解です
0976イナ ◆/7jUdUKiSM
垢版 |
2019/01/19(土) 20:15:27.60ID:elTtnGN6
_____」前>>974
( -~-)正解できて
zz∪∪うれしいです。
 ̄ ̄ ̄ ̄ ̄ ̄ ̄ ̄ ̄]
0977132人目の素数さん
垢版 |
2019/01/19(土) 20:44:43.28ID:EsplZJyO
>>973
そうですね。
中心の5角形と、それに隣接する6角形×5、さらにそれらに隣接する10枚の面をまとめて180度反転して図を作っています。
0978132人目の素数さん
垢版 |
2019/01/20(日) 17:45:48.59ID:wH8qpdgE
>>971
topologicalな構造のみを考えればいいんだから、単純に前の階層に隣接する面を右回りに列挙して表示すればいいだけじゃないかな?
一意な表現にしたければ各階層の起点はこれ、と決めてしまえばいい。

たとえば、真ん中の5から始めて、それを取り囲む面を次の階層としたとき、
各階層の12時の方角にあるものを起点とすると、
上の図では、5角形、6角形、5角形、稜、6角形、稜、5角形が起点
下の図では、5角形、稜、6角形、稜、6角形、稜、5角形が起点となる

そうすると、稜を縦棒で表すとして、
上の図は 5,66666,5656565656,|6666666666,6565656565,|66666,5
下の図は 5,|66666,6565656565,|6666666666,6565656565,|66666,5
みたいにすればよくないかな?
0979132人目の素数さん
垢版 |
2019/01/21(月) 02:24:43.78ID:0Mn8D/WQ
自然数から自然数への関数 y = f(x) を
y = x + a + (~b & (b-1))
で定義する。ただし、a および b は
x = a*(2*b-1) ; aは1,2,4,8,...のように2の冪で表せる数、bは自然数
で定まるものとする。
尚、 “~” は 否定 NOT 、 “&” は 論理積 AND を表す。


f(10)=f(2*5)=10+2+(~3 & 2)=12+(~[11] & [10])=12+([00] & [10])=12
f(11)=f(1*11)=11+1+(~6 & 5)=12+(~[110]&[101])=12+([001]&[101])=13
f(12)=f(4*3)=12+4+(~2&1)=16+(~[10]&[01])=16+([01]&[01]=17=f^2(10)
f(13)=13+1+(~7&6)=14=f^2(11)
f(14)=14+2+(~4&3)=19


問題1:f^10(10)を求めよ
問題2:f^m(n)=2019 となる (m,n)のうち、m+nを最小にする(m,n)を求めよ
問題3:f^m(n)=20190121 となる (m,n)のうち、m+nを最小にする(m,n)を求めよ
0980132人目の素数さん
垢版 |
2019/01/21(月) 07:06:24.75ID:s0Etp/8a
ある私立医大の合格者の偏差値の平均値はm、標準偏差は10の正規分布であるとする。
合格者のうち成績上位70%は入学を辞退し下位30%の合格者が入学する。入学者の偏差値の平均値をmaとする。
m - maを算出せよ。
0981132人目の素数さん
垢版 |
2019/01/21(月) 15:13:14.71ID:5XDj55BD
>>980

f(x) = 1/{σ√2π)} exp[-(x-m)^2/ 2σ^2],

下位30% ・・・・ 偏差値(m-0.5244σ)以下

f(x)

ma = ∫[-∞, m-0.5244σ] x・f(x) dx } / ∫[-∞, m-0.5244σ] f(x) dx{
 = m + ∫[-∞, m-0.5244σ] (x-m) f(x) dx / 0.3
 = m + (σ/√2π)∫[-∞, -0.5244] t・exp(-tt/2) dt / 0.3
 = m + (σ/√2π) [ -exp(-tt/2) ](t=-∞, -0.5244) / 0.3
 = m + (σ/√2π) [ -0.8715 / 0.3 ]
 = m - 1.159σ
 = m - 11.59
0984132人目の素数さん
垢版 |
2019/01/21(月) 17:37:33.47ID:LMGU6ueJ
>>979
問題2は (m, n)=(155, 255) かな

f(x) は x の2進数表記と 1 の数が同じものを
小さい順に並べたとき、x の次を表す
2019=[1 11111 00011] は 1 が 8 個だから
255=[111 11111] まで遡ることができる
0986132人目の素数さん
垢版 |
2019/01/21(月) 19:52:05.09ID:0Mn8D/WQ
>>982
問題1:関数を順次作用させていくと、
10,12,17,18,20,24,33,34,36,40,48,65,66,...
という整数列が得られます。
先頭の10を第一項とすると、そこから10項目の第11項は48。正解です。
この数列を見て、何かに気づかないかな? というのが狙いでした。
問題2、3は残念ながら最小ではないので不正解です。

>>984
正解。関数の「意味」も、ご指摘の通りです。

任意の二つの自然数が、この関数で結びつくかどうかは、二進数に直して、1の数を数えれば判断できます。
結びつくことがわかった場合、その距離をどのように計るか? それを考えるための問題が2と3です。

この関数をプログラム化するのは、ビット演算可能な言語なら簡単にできます。
実際に繰り返し関数を適用すれば、問題2は簡単に答えにたどり着くだろうけど、
問題3は困難だろうと思い採用した数字でした。しかし、実際にコード化し試したところ一瞬でした。

というわけで、問題3も、プログラム的解法が可能です。が、非プログラム的解法を期待します。
0988132人目の素数さん
垢版 |
2019/01/22(火) 00:04:54.96ID:VZNVWJP7
古典クイズ

沢山の宝石がある。宝石には穴が空いており、全ての宝石は一本の長い紐に通されて一直線に並んでいる。
また、宝石はn種類あり、各種類の宝石の個数は様々であるが全て偶数個であることは分かっている。
紐を何回か切断しいくつかの塊に分けることで、紐から宝石を外すことなく2人の人間で宝石を分けることを考える。
このとき、宝石の個数や並びに関わらず、n回切断することで常に均等に宝石を分けられることを示せ。
0989132人目の素数さん
垢版 |
2019/01/22(火) 02:32:35.40ID:iC4qDyjQ
>>987
正解

20190121=[1001101000001001110101001]。二進数で25桁、1の数が11。これが、20190121の属性。
2^11-1=2047=[11111111111]を起点にすると、m+n を最小にできる。

[11111111111]を一番目とすると、24桁の最大数[11111111110000000000000]は、C[24,11]=2496144番目。
この数にもう一度関数を作用させると、25桁の最小数[1000000000000001111111111]になる。
この「何番目」という指標と、関数の作用回数は、1ずれていることに注意すると、
[11111111111]に、C[24,11]回、関数を作用させると、最上位の桁=2^24の位に1を立て、
残りの1をすべて下位の桁に押し込んだ[1000000000000001111111111]を得られることが判る。

同様に、1が10個並んだ状態[1111111111]に、C[21,10]回作用させると、22桁の最小数 
[1000000000000111111111]になる。スタート地点が[1000000000000001111111111]だったら、
[1001000000000000111111111]となるが、上位3桁に影響はない。

つまり、[11111111111]=2047を[1001101000001001110101001]に到達させるためには、
C[24,11]+C[21,10]+C[20,9]+C[18,8]+C[12,7]+C[9,6]+C[8,5]+C[7,4]+C[5,3]+C[3,2]
=3061558回、関数を作用させればよい。
http://codepad.org/d6ezVAJb
0990132人目の素数さん
垢版 |
2019/01/22(火) 02:45:12.76ID:VZNVWJP7
>>989
ほとんど同じ考え方で計算してました
さすがにC[24,11]+C[21,10]+……C[3,2]の計算はパソコンを使いましたが
0991132人目の素数さん
垢版 |
2019/01/23(水) 07:53:49.36ID:0KQkAS3a
>>955
(f=10 の参考に)

シリコンフラーレン分子Si_16
Q,p4,p4,Q

Qの1辺       2.34Å
pp境界(Qに接触) 2.25Å
pp境界       2.28Å 
各8稜

V.Kumar, C.Majumder & Y.Kawazoe: Chem. Phys. Lett., Vol.363, Iss.3/4, p.319-322 (2002/Sep)
(東北大・金材研の川添教授)

シリコン単結晶  2.3513Å
0992132人目の素数さん
垢版 |
2019/01/23(水) 10:11:16.98ID:eeoGrxts
>>991
このスレの>>32に貼られてる10面体がそれでしょうか
長さについては多少違うようですが
0993132人目の素数さん
垢版 |
2019/01/23(水) 10:34:36.27ID:EtY2cbZ8
ゼビスポモールのセールいいね
0997132人目の素数さん
垢版 |
2019/01/25(金) 02:50:08.43ID:PhzqWDq+
これ面白い

2の間隔の正方形を作り点をABCDとしたよ。
辺ABの上に点Pを取り、辺AP=BP=√2としたよ。

1)この時、Pを含めた5角形APBCDの周長さはいくつかな?

2)点Pを(1)の周長を変えないように点B点Dの延長線上となるように点Pを動かしたよ。
その結果四角形APCDとなったよ。
この時の点DP間の長さはいくつかな?

3)点Pが(2)の状態にある時、点P1とし、(1)の周長を崩さぬように点Pを動かしまた点BDの直線上に動かしその点をP2としたよ。
点P1→点P2動いた時間はいくつ?
なお、動かした時間は1につき1secとするよ。

4)動かした点Pの軌道をグラフに描く。
点P1が(0,0)を通るように上に凸で書いたよ。
次に(x,y)=(2,0)を中心とし(0,0)を通る半円を書き足したよ。
軌道と円で囲まれてる所の面積はいくつかな
0998132人目の素数さん
垢版 |
2019/01/25(金) 03:40:28.47ID:dfwh8WQW
>>945
「メディアルf面体」
  5 - 12/f < k < 7 - 12/f なるk角形からなるf面体。
  f≧12 のときは 5角形×12,6角形×(f-12)
  fが12を割り切るときは全て(6-12/f)角形 (例:正f面体)
0999132人目の素数さん
垢版 |
2019/01/25(金) 04:47:55.06ID:w6DbRDDH
>>998
5角形×12,6角形×1 で作られる多面体は、どんな形になりますか?
1000132人目の素数さん
垢版 |
2019/01/25(金) 13:18:12.07ID:fIrAiYO+
不可能
10011001
垢版 |
Over 1000Thread
このスレッドは1000を超えました。
新しいスレッドを立ててください。
life time: 88日 12時間 58分 49秒
10021002
垢版 |
Over 1000Thread
5ちゃんねるの運営はプレミアム会員の皆さまに支えられています。
運営にご協力お願いいたします。


───────────────────
《プレミアム会員の主な特典》
★ 5ちゃんねる専用ブラウザからの広告除去
★ 5ちゃんねるの過去ログを取得
★ 書き込み規制の緩和
───────────────────

会員登録には個人情報は一切必要ありません。
月300円から匿名でご購入いただけます。

▼ プレミアム会員登録はこちら ▼
https://premium.5ch.net/

▼ 浪人ログインはこちら ▼
https://login.5ch.net/login.php
レス数が1000を超えています。これ以上書き込みはできません。

ニューススポーツなんでも実況